[obm-l] Re: [obm-l] produtório(seno e cosseno)

2018-07-24 Por tôpico Rodrigo Renji
Olá! o pessoal postou várias soluções, mas motivado pelo seu email acabei
escrevendo de um outro modo e colocando num blog, se quiser dar uma olhada,
tem um material extra de produtórios para download também

Link
https://matematicapurafm.blogspot.com/2018/07/produtos-envolvendo-funcoes.html


Livre
de vírus. www.avast.com
.
<#DAB4FAD8-2DD7-40BB-A1B8-4E2AA1F9FDF2>

Em 23 de julho de 2018 18:34, Kevin Felipe Kuhl Oliveira 
escreveu:

> Veja se concorda com o seguinte raciocínio:
>
> sen(x) = 2*cos(x/2)*sen(x/2) = 2*cos*(x/2)*(2 cos(x/4)*sen(x/4))
>
> Então, teremos (pode-se provar por indução):
> sen(x) = 2^(n)*cos (x/2)*cos(x/4)*cos(x/8)*cos(x/16)*….*cos
> (x/2^n)*sen(x/2^(n))
>
> Dividindo ambos os lados da igualdade por x:
>
> (sen(x))/x = 
> 2^(n)*cos(x/2)*cos(x/4)*cos(x/8)*cos(x/16)*….*cos(x/2^(n))*sen(x/2^(n))/x
> =
> =cos(x/2)*cos(x/4)*cos(x/8)*cos(x/16)*….*cos(x/2^(n))*[
> sen(x/2^(n))/(x/2ˆ(n))]
>
> Quando n tende a infinito, sen(x/2^(n))/(x/2ˆ(n)) tende a 1.
>
> Assim, prova-se a igualdade do problema cos(x/2). cos(x/4).cos(x/8)...  =
> (sen(x))/x.
>
> Att.
>
> Kevin Kühl
> Em 23 de jul de 2018 17:24 -0300, marcone augusto araújo borges <
> marconeborge...@hotmail.com>, escreveu:
>
> Como mostrar que cos(x/2). cos(x/4).cos(x/8)...  = (senx)/x ?
>
> --
> Esta mensagem foi verificada pelo sistema de antivírus e
> acredita-se estar livre de perigo.
>
>
> --
> Esta mensagem foi verificada pelo sistema de antivírus e
> acredita-se estar livre de perigo.
>

-- 
Esta mensagem foi verificada pelo sistema de antiv�rus e
 acredita-se estar livre de perigo.



[obm-l] Re: [obm-l] Re: [obm-l] Recorrência

2016-10-16 Por tôpico Rodrigo Renji
Olá pessoal : )
Estou escrevendo um material, se quiserem dar uma olhada, os links deixo
abaixo

►(9.14) equações de diferenças ( recorrências lineares) I
https://dl.dropboxusercontent.com/u/21174119/compartilhar/equacoesdiferencas.pdf
►(9.15) equações de diferenças ( recorrências lineares) II
https://dl.dropboxusercontent.com/u/21174119/compartilhar/equacoesdediferenas2.pdf

Em 16 de outubro de 2016 20:51, Jeferson Almir 
escreveu:

> Principles and Techniques in Combinatorics
> ( Chen chuan-chong ) acredito ser intermediário pra Phoda
> Aí desses pesados existe o Introduction to Combinatorics e o
> Problems in Combinatorics and Graph Theory ambos do renomado IOAN TOMESCU
>
> Em domingo, 16 de outubro de 2016, Esdras Muniz 
> escreveu:
>
>> Olá amigos, gostaria que me passassem eferências de livros ou artigos que
>> falem sobre recorrência. Dês de já obrigado.
>>
>> --
>> Esdras Muniz Mota
>> Mestrando em Matemática
>> Universidade Federal do Ceará
>>
>>
>>
>> --
>> Esta mensagem foi verificada pelo sistema de antivírus e
>> acredita-se estar livre de perigo.
>
>
> --
> Esta mensagem foi verificada pelo sistema de antivírus e
> acredita-se estar livre de perigo.
>

-- 
Esta mensagem foi verificada pelo sistema de antiv�rus e
 acredita-se estar livre de perigo.



Re: [obm-l] Soma

2015-09-29 Por tôpico Rodrigo Renji
Deixo um vídeo com a dedução da fórmula da soma de k=1 até infinito de k
a^k (que dá 1/ (1-a) ).

Daí parece tranquilo obter a que deseja tomando a=e^{-0,08} .

https://www.youtube.com/watch?v=yBRAIuUyM1I=5=PLmT_L9MZaC2mX4fmZwFRuz6RwM8GGNPcS

Em 29 de setembro de 2015 15:38, João Sousa 
escreveu:

> Alguém poderia me passar a fórmula geral para
>
> sum_{k=1}^{\infty} k*exp(-0,08*k)
>
> Abs
> João
>
> --
> Esta mensagem foi verificada pelo sistema de antivírus e
> acredita-se estar livre de perigo.
>

-- 
Esta mensagem foi verificada pelo sistema de antiv�rus e
 acredita-se estar livre de perigo.



[obm-l] Re: [obm-l] Re: [obm-l] Re: Princípio da indução finita

2014-07-24 Por tôpico Rodrigo Renji
Se quiser escrevi uma tentativa aqui também, página 19

►(4.3)números naturais, axiomas de peano

https://www.dropbox.com/s/h5i3mhuno663pzo/numerosnaturaisaxiomasdepeano.pdf


Em 24 de julho de 2014 22:01, Cassio Anderson Feitosa 
cassiofeito...@gmail.com escreveu:

 http://ellalves.net.br/textos/conteudo/37/inducao_matematica_parte_i

 --
 Esta mensagem foi verificada pelo sistema de antivírus e
 acredita-se estar livre de perigo.


-- 
Esta mensagem foi verificada pelo sistema de antiv�rus e
 acredita-se estar livre de perigo.



[obm-l] Re: [obm-l] Uma fórmula

2014-06-10 Por tôpico Rodrigo Renji
Olá, aqui escrevi várias fórmulas para soma de termos dessa forma, só que
não estão muito didáticos =x

*►Somatórios*

►(9.1)texto I

https://www.dropbox.com/s/ra4g9mghzgmpvk1/sum1-def-bern-euler-inter-stir.pdf

►(9.2) Texto

https://www.dropbox.com/s/okrvri90pbq0so3/sum2-poli-inver-harm-gamma.pdf

Aqui em um blog escrevi uma maneira usando derivadas

http://bmpa.wordpress.com/2012/02/26/soma-de-potencias/

( se quiser mais textos de somas, produtórios etc tenho esses aqui
http://bmpa.wordpress.com/2012/04/29/minhas-anotacoes/ )

abraço


Em 10 de junho de 2014 22:01, marcone augusto araújo borges 
marconeborge...@hotmail.com escreveu:

 Obrigado a todos qe se manifestaram.
 Eu sei que tem uma fórmula para 1^m + 2^m + ...+ n^m
 Outra coisa: na verdade eu gostaria de saber como fica o desenvolvimento de
 (a1 + a2 + a3 ...+ a10)^10
 Desde já agradeço.

 --
 Esta mensagem foi verificada pelo sistema de antivírus e
 acredita-se estar livre de perigo.


-- 
Esta mensagem foi verificada pelo sistema de antiv�rus e
 acredita-se estar livre de perigo.



[obm-l] Re: [obm-l] Fórmula fechada(?)

2014-03-10 Por tôpico Rodrigo Renji
Pode não ter fechada, mas tem em função dos números harmônicos

Sendo S_n= 1 + (1+1/2) + (1 + 1/2 +1/3) + ... +(1+1/2+... +1/n)
e H_n= 1+...+1/n

então S_n= (n+1)H_n -(n+1)

Uma maneira de demonstrar é usando soma por partes

(tenho feito aqui na página 29 se quiser ver, mas já usando soma por
partes, a técnica de soma por partes eu falo no inicio do texto)

https://www.dropbox.com/s/luxel9a8fc57g6g/sum3-partes-fato-harmo.pdf


Em 10 de março de 2014 08:11, marcone augusto araújo borges 
marconeborge...@hotmail.com escreveu:

 Existe uma formula fechada para H_n = 1 + (1+1/2) + (1 + 1/2 +1/3) + ...
 +(1+1/2+... +1/n)?

 --
 Esta mensagem foi verificada pelo sistema de antivírus e
 acredita-se estar livre de perigo.


-- 
Esta mensagem foi verificada pelo sistema de antivírus e
 acredita-se estar livre de perigo.



[obm-l] Re: [obm-l] Re: [obm-l] Ajuda em recorrência!!

2013-12-15 Por tôpico Rodrigo Renji
Valeu! qualquer coisa só falar :) !


Em 15 de dezembro de 2013 07:42, douglas.olive...@grupoolimpo.com.brescreveu:

  Obrigado meu camarada  vou ler com atenção!!





 Em 14.12.2013 12:23, Rodrigo Renji escreveu:

 Faz
 f(n)+2= g(n+1)/g(n) =  1/ (f(n)+2) =  g(n) /  g(n+1) , (que vamos usar )

 daí f(n)-1 =g(n+1)/g(n) -3 =  [g(n+1) -3g(n) ]  / g(n)


 e  f(n+1) =g(n+2)/g(n+1)  -2 = [g(n+2)- 2g(n+1) ] /  g(n+1)

 por isso substituindo tudo em f(n+1)=(f(n)-1)/(f(n)+2) , segue que

 [g(n+2)- 2g(n+1) ] /  g(n+1)  =[g(n+1) -3g(n) ]  / g(n) .   g(n) /
  g(n+1)

 cancelando todas coisas canceláveis, segue que

 g(n+2)- 2g(n+1)  = g(n+1) -3g(n)

 o que implica

 g(n+2)= 3 g(n+1)-3g(n)

 que é uma recorrência de segunda ordem com solução conhecida , depois só
 ajustar as condições iniciais


 eu tenho um texto (ruim) falando sobre caso geral disso, se quiser dar uma
 olhada

 https://www.dropbox.com/s/0h6sfpe6p33vu76/equacoesdiferencas.pdf

 lá pela página 35 .

 Como transforma recorrência do tipo f(n+p)= (af(n)+ b)/ (c f(n) +d) ,
 caindo em uma outra recorrência que teoricamente sabemos resolver


 Em 14 de dezembro de 2013 08:56, 
 douglas.olive...@grupoolimpo.com.brescreveu:

  Olá amigos preciso de uma ajudinha para resolver um problema estava
 muito interessado em resolver a seguinte recorrência

 f(n+1)=(f(n)-1)/(f(n)+2) com f(1)=3 para n natural

 Qualquer ajuda será bem vinda.

 Att. Douglas Oliveira


 --
 Esta mensagem foi verificada pelo sistema de antivírus e
 acredita-se estar livre de perigo.


 --
 Esta mensagem foi verificada pelo sistema de antivírus e
 acredita-se estar livre de perigo.




 --
 Esta mensagem foi verificada pelo sistema de antivírus e
 acredita-se estar livre de perigo.


-- 
Esta mensagem foi verificada pelo sistema de antivírus e
 acredita-se estar livre de perigo.



[obm-l] Re: [obm-l] Ajuda em recorrência!!

2013-12-14 Por tôpico Rodrigo Renji
Faz
f(n)+2= g(n+1)/g(n) =  1/ (f(n)+2) =  g(n) /  g(n+1) , (que vamos usar )

daí f(n)-1 =g(n+1)/g(n) -3 =  [g(n+1) -3g(n) ]  / g(n)


e  f(n+1) =g(n+2)/g(n+1)  -2 = [g(n+2)- 2g(n+1) ] /  g(n+1)

por isso substituindo tudo em f(n+1)=(f(n)-1)/(f(n)+2) , segue que

[g(n+2)- 2g(n+1) ] /  g(n+1)  =[g(n+1) -3g(n) ]  / g(n) .   g(n) /
 g(n+1)

cancelando todas coisas canceláveis, segue que

g(n+2)- 2g(n+1)  = g(n+1) -3g(n)

o que implica

g(n+2)= 3 g(n+1)-3g(n)

que é uma recorrência de segunda ordem com solução conhecida , depois só
ajustar as condições iniciais


eu tenho um texto (ruim) falando sobre caso geral disso, se quiser dar uma
olhada

https://www.dropbox.com/s/0h6sfpe6p33vu76/equacoesdiferencas.pdf

lá pela página 35 .

Como transforma recorrência do tipo f(n+p)= (af(n)+ b)/ (c f(n) +d) ,
caindo em uma outra recorrência que teoricamente sabemos resolver


Em 14 de dezembro de 2013 08:56, douglas.olive...@grupoolimpo.com.brescreveu:

  Olá amigos preciso de uma ajudinha para resolver um problema estava
 muito interessado em resolver a seguinte recorrência

 f(n+1)=(f(n)-1)/(f(n)+2) com f(1)=3 para n natural

 Qualquer ajuda será bem vinda.

 Att. Douglas Oliveira


 --
 Esta mensagem foi verificada pelo sistema de antivírus e
 acredita-se estar livre de perigo.


-- 
Esta mensagem foi verificada pelo sistema de antivírus e
 acredita-se estar livre de perigo.



[obm-l] Re: [obm-l] Dízimas periódicas

2013-08-13 Por tôpico Rodrigo Renji
Olá! Luiz.

Essa questão eu vejo ser bastante frequente ( por exemplo, só dar uma
olhada em dúvidas que postam na internet como em redes sociais, por exemplo
lista de discussão de matemática no facebook) .

Eu penso que para entender realmente, o que acho ser a essência disso é
necessário algum tipo de entendimento de processo limite ( ou processos
equivalentes ) .

Digo isso pois, 0,999 no fim é o limite de uma soma  , isto é, uma
série .
0,999... e o  número do qual se aproxima a sequência
0,9
0,99
0,999
etc

que é o número 1 , 0,999... sendo apenas uma outra representação do
número .

Nisso tem que definir o que significa repetir infinitos decimais como em
0,999..., acho que isso é feito com mais clareza usando séries, que é
algo não abordado em geral fora da faculdade , então acho meio complicado
 explicar em essência o que é, porém acho que é possível dar ideias
intuitivas sobre o processo .

Estamos em um certo grupo criando um arquivo de dúvidas frequentes de
matemática e fazendo alguns vídeos também, se quiser dar uma olhada
►Perguntas frequentes e erros comuns
https://www.dropbox.com/s/c8e49mdirkijft0/perguntasfreqerros.pdf

e uma lista de reprodução com vídeos
http://www.youtube.com/playlist?list=PLmT_L9MZaC2n0r7BnN3ileCQsKSGwxHoJ

abraço!


Em 13 de agosto de 2013 14:19, Luiz Antonio Rodrigues rodrigue...@gmail.com
 escreveu:

 Olá, pessoal!
 Tudo bem?
 Na semana passada eu propus a seguinte discussão para os meus alunos: se
 0,111... + 0,888... = 0,999... então 1/9 + 8/9 = 0,999...   o que implica
 que 1= 0,999...
 Consegui despertar a curiosidade dos alunos, mas muitos deles não
 aceitaram o que acabamos concluindo. Alguém poderia me ajudar com algum
 argumento interessante sobre a estranha conclusão?
 Obrigado e um abraço!
 Luiz

 --
 Esta mensagem foi verificada pelo sistema de antivírus e
 acredita-se estar livre de perigo.

-- 
Esta mensagem foi verificada pelo sistema de antiv�rus e
 acredita-se estar livre de perigo.



[obm-l] Re: [obm-l] Re: [obm-l] SOMATÓRIO

2013-08-03 Por tôpico Rodrigo Renji
Um outro modo

usa a fatoração y²-1=(y-1) (y+1) com y=2 ^(2^k) simplifica a fração usando
isso e cai numa soma telescópica ( os termos vão se anulando conforme vai
somando), com isso dá para achar a fórmula da soma finita, depois tomar o
limite .

Dá para estudar essa questão com x^{2^k} no lugar de 2 ^(2^k) o processo é
o mesmo.


O caso geral com x, faz a série convergir para (x+1)/(x²+1)  se |x|1 .

Tenho essa questão escrita em um pdf, com outras somas também, se quiser
dar uma olhada, página 69
https://www.dropbox.com/s/okrvri90pbq0so3/sum2-poli-inver-harm-gamma.pdf


Em 3 de agosto de 2013 12:04, Pacini Bores pacini.bo...@globo.comescreveu:

 Seja S o valor do somatório .
 Tente mostrar que :

 1 - 1/(2^(2^n))  S  1/2+1/4+1/8+1/16+...

 Pacini




 Em 3 de agosto de 2013 11:26, Bob Roy bob...@globo.com escreveu:

 Olá,
 só consegui fazer limitações e não consegui determinar o valor do
 somatório abaixo .

 Alguém me ajuda ?

 somatório de zero ao infinito de  (2^(2^n))/((2^(2^(n+1))-1) .

 abs

 Bob



 --
 Esta mensagem foi verificada pelo sistema de antivírus e
 acredita-se estar livre de perigo.



 --
 Esta mensagem foi verificada pelo sistema de antivírus e
 acredita-se estar livre de perigo.


-- 
Esta mensagem foi verificada pelo sistema de antivírus e
 acredita-se estar livre de perigo.



Re: [obm-l] somatorio formula em f(n)

2013-07-10 Por tôpico Rodrigo Renji
Opa, valeu por postar o link do Fatos matemáticos (recomendo o blog),

As versões mais recentes dos textos, vou colocar em alguns links abaixo

E uma lista de reprodução de vídeos no youtube com teoria básica
http://www.youtube.com/playlist?list=PLmT_L9MZaC2kzEosTUaAOjjrymbGy84W5

Somatórios
texto I Definição, números de Euler, bernoulli, stirling
https://www.dropbox.com/s/ra4g9mghzgmpvk1/sum1-def-bern-euler-inter-stir.pdf

Texto 2 soma de polinomios ( vários fórmulas), inversos, soma harmonica,
soma usando função gamma
https://www.dropbox.com/s/okrvri90pbq0so3/sum2-poli-inver-harm-gamma.pdf

Texto 3 Soma por partes (parecida com integração por partes), soma com
fatorial, mais harmonicos
https://www.dropbox.com/s/luxel9a8fc57g6g/sum3-partes-fato-harmo.pdf
Texto 4 soma e integral, truque de gauss, soma usando derivada, soma pelo
metodo da função indeterminada
https://www.dropbox.com/s/q1pn22ryghm4tfw/sum4-inte-gaus-deri-indet.pdf
texto 5 desigualdades, função beta, função piso
https://www.dropbox.com/s/4kbect7p8xsz1o8/sum5-confin-beta-piso-desi.pdf
Texto 6 soma de binomiais
https://www.dropbox.com/s/71hegdmg97d0661/sum6-binomiais.pdf
Texto 7 soma de trigonométricos
*https://www.dropbox.com/s/88rnq00mh5zb8yk/sum7-trigonometricos.pdf
*Texto 8 soma usando combinatória
https://www.dropbox.com/s/6ux9jnju0z8j4cj/sum8-combinatoria-divi.pdf


abraço

Rodrigo



Em 10 de julho de 2013 13:30, Rígille Scherrer Borges Menezes 
rigillesbmene...@gmail.com escreveu:

 Já vi em um site, fatos matemáticos, alguns materiais sobre somatórios.
 Não li ainda, mas talvez você ache útil:

 http://fatosmatematicos.blogspot.com.br/2010/10/tecnicas-para-somatorios.html


 Em 7 de julho de 2013 09:54, terence thirteen 
 peterdirich...@gmail.comescreveu:

 27, integrais discretas por Eduardo Poço
 29, Algoritmo de Gosper, por Humberto Naves

 O segundo é um artigo um tanto elaborado, merece uma leitura bem detida.


 Em 6 de julho de 2013 18:15, Hermann ilhadepaqu...@bol.com.br escreveu:

 **
 Agradeço a ajuda, serrá que o Eureka tem um super indice como na RPM?
 Pelo visto é realmente complicado e tem a ver com experiências do aluno.
 Valeu
 Hermann

 - Original Message -
 *From:* terence thirteen peterdirich...@gmail.com
 *To:* obm-l obm-l@mat.puc-rio.br
 *Sent:* Saturday, July 06, 2013 4:49 PM
 *Subject:* Re: [obm-l] somatorio formula em f(n)

  Em geral isto depende muito dos termos dentro do somatório. Às vezes
 estas somas são chatas pra caramba, em outros são fáceis. Por exemplo, no
 se caso, você poderia pensar que a soma dos quadrados se comporta como um
 polinômio.

 Mas, em geral, isto tem a ver com funções hipergeométricas. Tem um
 artigo na Eureka! sobre isto, vou caçar!




 Em 6 de julho de 2013 11:33, Hermann ilhadepaqu...@bol.com.brescreveu:

 **
 Meus amigos gostaria de uma (+1) ajuda:

 Qual o metodo ou raciocinio para: dado um somatorio deixá-lo em função
 de n

 exemplo S,i=1 a n, (i-1)^2

 como chego emn(2n^2-3n+1)/6

 obrigado
 Hermann

 --
 Esta mensagem foi verificada pelo sistema de antivírus e
 acredita-se estar livre de perigo.




 --
 /**/
 神が祝福

 Torres

 --
 Esta mensagem foi verificada pelo sistema de antiv�rus e
 acredita-se estar livre de perigo.


 --
 Esta mensagem foi verificada pelo sistema de antivírus e
 acredita-se estar livre de perigo.




 --
 /**/
 神が祝福

 Torres

 --
 Esta mensagem foi verificada pelo sistema de antivírus e
 acredita-se estar livre de perigo.



 --
 Esta mensagem foi verificada pelo sistema de antivírus e
 acredita-se estar livre de perigo.


-- 
Esta mensagem foi verificada pelo sistema de antiv�rus e
 acredita-se estar livre de perigo.



Re: [obm-l] Binomais de m+n tomados p a p

2012-05-15 Por tôpico Rodrigo Renji
Olá!

além da soluções que postaram (se entendi bem a identidade), tinha
escrito algumas outras nessa página de um blog
http://bmpa.wordpress.com/2011/05/29/demonstracao-da-convolucao-de-vandermonde-relacao-de-euler/
( escritas em tex), se quiserem dar uma olhada :)

abraço!

=
Instruções para entrar na lista, sair da lista e usar a lista em
http://www.mat.puc-rio.br/~obmlistas/obm-l.html
=


Re: [obm-l] Arquivos fontes - Material do IME

2012-02-15 Por tôpico Rodrigo Renji
Olá!

Sérgio parabéns pela concretização do livro!

Bardonista

O que podemos fazer é outro material gratuito, realmente no creative
commons, com fonte livre , que possa até ser editado por quem desejar
( citando a fonte principal pelo menos). Estou querendo tentar fazer
um material desse e deixar online, pedir ajuda de quem puder
contribuir com soluções também, tentando resolver as provas de física
e matemática do IME e ITA e alguns textos com teoria anexada.

abraço

Em 15 de fevereiro de 2012 13:38, Bardonista Magista
bardoni...@gmail.com escreveu:
 Aproveito para enviar uma cópia do licenciamento para uso não
 comercial e pessoal do PDF, que constava do site
 http://www.lps.ufrj.br/~sergioln/ime/ em maio/2010:

 PROVAS DO VESTIBULAR DO IME

      Arquivo de provas de Matemática do Vestibular do Instituto
 Militar de Engenharia (IME).
      A versão 16 contém 116 provas e 56 soluções em cerca de 3 MB.

      Download:

      O uso particular ou individual, sem fins lucrativos, deste
 material é livre.
      O uso institucional, com ou sem fins lucrativos, deve ser
 solicitado por email ao autor: sergi...@lps.ufrj.br.

      Você pode contribuir com a elaboração deste material das seguintes 
 formas:
      (a) fornecendo enunciado de provas ainda não disponíveis;
      (b) enviando correções, sugestões ou comentários;
      (c) complementando as resoluções das provas antigas.

      Infelizmente, não tenho provas de outras disciplinas do IME e
 nem provas de Matemática do ITA. Para isto, posso sugerir:
      http://www.sassabetudo.cjb.net
      http://www.ime.eb.br (comece por Mapa do Site  Vestibular)
      http://www.estudemais.com.br (algumas pagas)
      http://www.net-rosas.com.br/~cesario
      http://www.rumoaoita.com (muito ativo, com simulados e apostilas)



 Em 15 de fevereiro de 2012 08:14, Bardonista Magista
 bardoni...@gmail.com escreveu:
 Saudações,

  Uso linux a muitos anos e é mais natural para mim o LaTeX e
 ferramentaria Open Source. Já há um tempo não está mais disponível os
 arquivos fontes LaTeX das provas de matemática IME disponibilizadas
 sob licença de uso pessoal livre e não comercial. Alguém que tenha
 salvado o arquivo IME_v14.zip ou IME.zip poderia me enviar por e-mail:
 bardoni...@gmail.com ? Gostaria muito de estudar a fonte LaTeX sua
 estrutura e verificar como ficou leve e com diagramação excelente os
 PDF que eram disponibilizados...

 Agradecido,

 Bardonista


 Re: [obm-l] Arquivos fontes - Material do IME

 Saulo
 Fri, 09 Jan 2009 19:53:22 -0800

 Realmente é de grande louvor mesmo.
 Eu tenho resoluções das provas do IME de 1982 até 2000 ou 2001 se
 não me engano.
 Tenho provas da década de 70 tbm, um material muito rico com questões
 do IME e ITA. Algumas questões resolvidos já.



    Mensagem Original:
    Data: 17:11:30 09/01/2009
    De: Paulo Santa Rita paulo.santar...@gmail.com
    Assunto: Re: [obm-l] Arquivos fontes - Material do IME

    Ola Sérgio,

    Parabéns ! Este seu trabalho é digno de louvor em diversos
 sentidos.Tenha certeza que ele já é e será útil em diversos níveis e
 paradiversos tipos de estudantes. Se precisar de ajuda na solução
 dealguma questão, conte comigo.

    Um AbraçãoPSR, 60901091709

    2009/1/9 Sergio Lima Netto sergi...@lps.ufrj.br: Caros colegas
 da lista, Estou disponibilizando no site
 www.lps.ufrj.br/~sergioln/ime TODOS os arquivos-fonte de LaTeX (.tex
 para os textos e .eps para as figuras) do material com as provas de
 matematica do vestibular do IME. O uso pessoal/individual deste
 material eh livre (para edicao, correcao, complementacao, divulgacao
 etc.) O uso institucional deve ser solicitado por email. Considero
 que este foi um grande projeto, feito com muito carinho. Gostaria
 muito de ve-lo continuado, expandido, melhorado, divulgado, ou seja,
 usado, por varias outras pessoas. Abraco, sergio
 =
 Instruções para entrar na lista, sair da lista e usar a lista em
 http://www.mat.puc-rio.br/~obmlistas/obm-l.html
 =

    =
    Instru??es para entrar na lista, sair da lista e usar a lista em
    http://www.mat.puc-rio.br/~obmlistas/obm-l.html
    =

 =
 Instruções para entrar na lista, sair da lista e usar a lista em
 http://www.mat.puc-rio.br/~obmlistas/obm-l.html
 =

=
Instruções para entrar na lista, sair da lista e usar a lista em
http://www.mat.puc-rio.br/~obmlistas/obm-l.html
=


[obm-l] Re: [obm-l] Função Injetora

2011-12-13 Por tôpico Rodrigo Renji
Olá joão!


Isso não vale em geral em conjuntos infinitos

considere por exemplo

f: N em N com
f(n) =n+1

a função é injetora, porém não é sobrejetora.

nenhum elemento é enviado no número 0 ( com N= {0,1,2,3,} )

=
Instruções para entrar na lista, sair da lista e usar a lista em
http://www.mat.puc-rio.br/~obmlistas/obm-l.html
=


[obm-l] Re: [obm-l] Re: [obm-l] RE: [obm-l] Solução da série \sum_{i=1}^n 2^{n-i}i^2

2011-09-16 Por tôpico Rodrigo Renji
Olá!

então umas maneiras de calcular a soma

 \sum_{i=1}^n 2^{n-i}i^2

pode pensar no caso geral

\sum_{i=1}^n x^{i}i^2

você sabe

\sum_{k=0}^n x^{k} = [x^(n+1) -1] / [x-1]

se você deriva essa identidade em relação a x, tem

\sum_{k=0}^n k x^{k-1} = D [x^(n+1) -1] / [x-1]

onde D é a derivada
multiplique acima por x

\sum_{k=0}^n k x^{k} = xD [x^(n+1) -1] / [x-1]

agora aplique o procedimento novamente, derive e multiplique por x,
resultando em

\sum_{k=0}^n k² x^{k} = xD ( xD [x^(n+1) -1] / [x-1] )


basta então ver o que é xD ( xD [x^(n+1) -1] / [x-1] ), isto é,
derivar e multiplicar por x duas vezes a expressão [x^(n+1) -1] /
[x-1]
( acho chato calcular essas derivadas)


Tem outro método chamado soma por partes (parecido com integração por
partes porém mais fácil ), com o qual é possível deduzir fórmula
fechada para esse tipo de soma também

em geral dá para deduzir expressão pra soma

\sum_{k=0}^n k^p x^{k}, com p natural
uma representação dessa fórmula aparece um tipo de número especial
chamado números de stirling do segundo tipo.

Vou deixar um material gratuito para download onde escrevi sobre isso

http://www.4shared.com/folder/dumYzksM/Somatrios.html
no texto 3 falo um pouco sobre soma por partes


abraço!

=
Instruções para entrar na lista, sair da lista e usar a lista em
http://www.mat.puc-rio.br/~obmlistas/obm-l.html
=


[obm-l] Re: [obm-l] RE: [obm-l] Re: [obm-l] Re: [obm-l] dúvida sobre séries

2011-06-07 Por tôpico Rodrigo Renji
Olá!

Então acho bem bacana esse também ( e nem é tão complicado de
demonstrar, eu acho )


Esse critério pode ser usado para estudar a convergência de [ SOMA de
1/ k^p  ]  também

pois  [ SOMA de 2^k / 2^(kp)  ]  =  [ SOMA de 2^(k (1-p))  ]

se 1 - p 0, isto é 1 p a série converge por série geometrica

se 1-p  0 , 1  p a série diverge de novo por série geometrica .

=
Instruções para entrar na lista, sair da lista e usar a lista em
http://www.mat.puc-rio.br/~obmlistas/obm-l.html
=


Re: [obm-l] Probleminha....

2011-06-06 Por tôpico Rodrigo Renji
Uma tentativa por modo indireto ( não sei se foi assim que fez xD)

abc. Prove que a equação 1/(x-a)  + 1/(x-b)  + 1/(x-c)=0 (I) ,
possui exatamente duas raízes, x1 e x2 , que satisfazem a condição
ax1bx2c.


em (b,c) a função é contínua com lim x-b pela direita dando  +
infinito e limite x-c pela esquerda dando - infinito
logo existe raiz em (b, c) por continuidade

da mesma maneira existe raiz em ( a, b) por continuidde.


multiplicando por (x-a)(x-b) (x-c), temos uma equação  de grau 2, que
só pode ter no máximo duas raizes reais. ( a multiplicação fornece uma
equivalência pois x não pode ser b, a ou c)

=
Instruções para entrar na lista, sair da lista e usar a lista em
http://www.mat.puc-rio.br/~obmlistas/obm-l.html
=


Re: [obm-l] PG de segunda ordem(?)

2011-06-06 Por tôpico Rodrigo Renji
Então, primeiro tem que definir o que é uma p.g de segunda ordem.

Uma PG é uma sequência x_n onde x(n+1)/x(n) =c  uma constante
podemos denotar x(n+1)/x(n) como Q x(n), Q é um operador que faz o
quociente de termos consecutivos da sequência

uma p.g de segunda ordem, seria uma sequência em que se aplica o
operador Q duas vezes e a sequência resulta numa constante

Q² x(n) =c

Q [ Q x(n)] =c

Q y(n) =c

logo y(n) =T .c^n para alguma constante ;t

por fórmula de p.g

substituíndo
Q x(n) = y(n) temos
Q x(n) =  T c^n
aplique o produto com k variando de 1 até n-1 em Q x(k) =  T c^k,
perceba que os termos no primeiro produtório vão se anulando
no segundo cai numa soma no expoente, o resultado fica como

x(n)=x (1). T^(n-1) c^((n-1)(n-2)/2 )

então por essa definição uma p.g de ordem 2 seria algo do tipo

c_1 .c_2^(n-1) . c_3 ^(n-1)(n-2)/2 =x_n

e essa sequência do email não seria uma pg de ordem 2.


Em geral podemos definir uma p.g de ordem p, como uma sequência x_n
que satisfaz


Q^p  x(n) =c, para alguma constante c

onde Q^p  é aplicar aquele operador quociente p vezes

a fórmula geral de uma p.g de ordem p é da forma

c_1. c_2^(n-1) .c_3 ^ ((n-1)(n-2)/2) .   c(p+1) ^ ((n-1)(n-2)... (n-p)/p!)

valeu! \o\

=
Instruções para entrar na lista, sair da lista e usar a lista em
http://www.mat.puc-rio.br/~obmlistas/obm-l.html
=


[obm-l] Re: [obm-l] Re: [obm-l] dúvida sobre séries

2011-06-06 Por tôpico Rodrigo Renji
Olá!

Uma outra maneira  ( além da que os colegas enviaram antes), para
mostrar que a série  não converge, tem um critério de convergência que
acho legal, Critério de condensação de Cauchy:


 Se x_k é uma sequência decrescente de termos positivos ( como é o caso de 1/k )


então a série [ SOMA de x_k]  converge , se e somente se , a série [
SOMA de 2^k  x_(2^k) ]  converge.

Aplicando isso para a série do email

temos com a_k= 1/k


[ SOMA de 2^k  x_(2^k) ]  =  [ SOMA de 2^k  ,  1/ (2^k)   ] =  [ SOMA 1 ]

que diverge, pois somando de 1 até n resulta em n, com n indo pro
infinito , diverge  : ) Pode não ajudar muito, mas acho esse critério
legal

abraço

=
Instruções para entrar na lista, sair da lista e usar a lista em
http://www.mat.puc-rio.br/~obmlistas/obm-l.html
=


Re: [obm-l] Fwd: Identidade de Euler (OFFTOPIC)

2011-05-29 Por tôpico Rodrigo Renji
Oi, eu tinha postado uma tentativa por interpolação de newton, mas fica ruim
de ler no email por falta dos caracteres matemáticos. Então fiz como o
dados e postei num blog essa demonstração

http://bmpa.wordpress.com/


e escrevi em um texto em formato pdf, junto com outras coisas, se alguém
quiser ver

http://www.4shared.com/folder/2XMFl46c/coeficiente_binomial.html

texto sobre coeficiente binomial


Abraço!


Re: [obm-l] Fwd: Identidade de Euler (OFFTOPIC)

2011-05-29 Por tôpico Rodrigo Renji
Olá, de novo

É possível demonstrar de outra maneira usando derivadas.

usando (1+x)^r (1+x)^s = (1+x)^(r+s)
aplique a n -ésima derivada de ambos lados
no primeiro lado use a regra de leibniz para derivada do produto de
duas funções, no outro lado apenas a derivada polinomial
, aplique a derivada em x=0, divida ambos os membros por n! e o resultado saí .

http://bmpa.wordpress.com/2011/05/29/demonstracao-da-convolucao-de-vandermonde-relacao-de-euler/

Abraço!

=
Instruções para entrar na lista, sair da lista e usar a lista em
http://www.mat.puc-rio.br/~obmlistas/obm-l.html
=


Re: [obm-l] Fwd: Identidade de Euler

2011-04-29 Por tôpico Rodrigo Renji
Olá!

Só por curiosidade, acho que consegui uma outra demonstração dessa
identidade ( usando interpolação de Newton)

A interpolação 'diz' o seguinte

f(n+x)= somatório ( de k=0 até n) C(n, k ) D^k f(x)

onde  D^k f(x) é a 'k' -ésima diferença em x (tomar diferença de
termos consecutivos 'k' vezes).

Tomando f( x ) = C(x, p) temos D^k C(x+n, p) = C(x+n, p- k ) (pela
relação de stifel aplicada várias vezes)

daí D^k C(x, p)=C(x, p- k ) e o resultado segue

C(x+n, p)= somatório ( de k=0 até n) C(n, k ) C(x, p- k )

abraço!

=
Instruções para entrar na lista, sair da lista e usar a lista em
http://www.mat.puc-rio.br/~obmlistas/obm-l.html
=


[obm-l] Re: [obm-l] Re: [obm-l] Re: [obm-l] Re: [obm-l] Re: [obm-l] FRAÇÕES - conceito

2011-03-28 Por tôpico Rodrigo Renji
Ainda sobre o 0^0, acho que a princípio não se deve  levar em conta
limites para decidir uma definição aritmética, ainda mais quando
existem identidades aritméticas que apontam que seria melhor definir
0^0 como 1.

Para limites não importa a definição da função no ponto, e se for
analisar continuidade, as funções a princípio também não precisam ser
contínuas ( daí não importaria definir 0^0 como um certo valor).

acho que escrever p(x)=SUM [(n=1 a M) a_n x^n] + a_0 é feio =p~
uma das utilidades de se ter 0^0 como 1 é ter fórmulas fechadas
compactas para diversas identidades

abraço

=
Instruções para entrar na lista, sair da lista e usar a lista em
http://www.mat.puc-rio.br/~obmlistas/obm-l.html
=


[obm-l] Re: [obm-l] Re: [obm-l] Re: [obm-l] Re: [obm-l] Re: [obm-l] FRAÇÕES - conceito

2011-03-25 Por tôpico Rodrigo Renji
Olá

Também acho natural ter o 0 em N, mesmo para contagem, pois podemos
associar |vazio|=0
(número de elementos do conjunto vazio associado ao zero), como o Rogério falou.


Sobre 0^0, eu também uso que seja 1. A noção de 'indeterminação' eu
uso apenas para limites e não para operações aritméticas.
(escrevi algo sobre isso)
http://www.4shared.com/dir/HLZtU_v7/zeroazero.html


Achei muito interessante o que Professor Ralph disse sobre frações,
achei legal : )

abraço

=
Instruções para entrar na lista, sair da lista e usar a lista em
http://www.mat.puc-rio.br/~obmlistas/obm-l.html
=


[obm-l] Re: [obm-l] Demonstração de somatório

2011-03-04 Por tôpico Rodrigo Renji
Olá
Então , nessa última perceba que

k.(k!)= (k+1)!-k!

aplique a soma de ambos os lados a soma no segundo termo é telescópica
( os termos vão se anulando)

=
Instruções para entrar na lista, sair da lista e usar a lista em
http://www.mat.puc-rio.br/~obmlistas/obm-l.html
=


[obm-l] Re: [obm-l] RE: [obm-l] Re: Demonstração de somatório

2011-03-04 Por tôpico Rodrigo Renji
Olá Henrique

Então pode ser feito  assim mesmo como você percebeu, os termos vão se anulando

essa técnica de soma telescópica talvez seja a mais importante para
demonstração\ dedução ( não indutiva) de fórmula para somatórios

em geral vale o seguinte

Soma telescópica
somatório ( de k=1 até n) de g (k+1) - g ( k )=  g(n+1)-g(1)

daí aplicando isso sai direto ( não acho é tanta coisa)

de
k.(k!)= (k+1)!-k!
aplicando a soma
somatório ( de k=1 até n) de k.(k! ) = somatório ( de k=1 até n)
(k+1)!-k! =  (n+1)!-1
por soma telescópica.

Abraço

=
Instruções para entrar na lista, sair da lista e usar a lista em
http://www.mat.puc-rio.br/~obmlistas/obm-l.html
=


[obm-l] Re: [obm-l] Re: [obm-l] RE: [obm-l] Fórmula fechada para somatório

2011-02-22 Por tôpico Rodrigo Renji
Olá

*notação
coeficientes binomiais  c ( k , p ) : = k! / ( p! (k-p)! )


Coeficientes binomiais são  fáceis de se calcular a soma, por causa da
relação de stiefel


c(k+1 ,  p+1 ) - c (k,  p+1 ) = c ( k, p )
 aplicamos a soma de ambos lados, a soma é telescópica

soma  (de  k=0 até  n ) c ( k, p ) =  c ( n+1, p+1 )
uma maneira de decorar isso é que, quando você soma c ( k, p )  de 0 até n
o resultado é só somar +1 em cima e em embaixo no coeficiente binomial
soma de c ( k, p ) é c ( n+1, p+1 ) .



1 + 3 + 6 + ... + n(n+1)/2 = n(n+1)(n+2)/6.

Então, nesse caso dá para ver com o coeficiente binomial

o termo somado é  k(k-1)/2 de k=0 até n+1

k(k-1)/2 ´pode ser escrito como c(k, 2 )

ai temos a soma
soma (de k=0 até  n+1 )  c(  k , 2 ) =  c (n+2 , 3) = (n+2) (n+1) (n)/6

Valeu

=
Instruções para entrar na lista, sair da lista e usar a lista em
http://www.mat.puc-rio.br/~obmlistas/obm-l.html
=


Re: [obm-l] Mais uma soma

2011-02-16 Por tôpico Rodrigo Renji
Olá

pode tentar fazer um caso geral

toma 1/ (ak +b)
mostra que

1/(a(k+1)+ b) - 1/ (ak +b) = -a /  ( (ak+b+a) (ak+b) )

aplica a soma de ambos lados, que é telescópica
assim você tem a fórmula da soma de termos do tipo
 -a /  ( (ak+b+a) (ak+b) )

depois só colocar os valores de a e b específicos
no caso do seu problema é a=3 , b=1 a soma começando de k=0 e indo até 65

a fórmula fechada da soma é
(n+1) / (3n+4)

=
Instruções para entrar na lista, sair da lista e usar a lista em
http://www.mat.puc-rio.br/~obmlistas/obm-l.html
=


Re: [obm-l] Mais uma soma

2011-02-16 Por tôpico Rodrigo Renji
Aproveitar também pra divulgar um material de somatório,  versão
completa de download gratuito (porém não tão bom)

nessa pasta do 4 shared
http://www.4shared.com/dir/dumYzksM/Somatrios.html
tem uns 7 pdf's

no texto 2 no finalzinho tem uma parte de soma de inversos que tem
esse e o caso geral desse

abraço

=
Instruções para entrar na lista, sair da lista e usar a lista em
http://www.mat.puc-rio.br/~obmlistas/obm-l.html
=


[obm-l] Re: [obm-l] outra soma de série

2011-01-21 Por tôpico Rodrigo Renji
Olá!

Então, isso é uma soma telescópica

seja f(x) = 1/(-p)   1/((x+1)... (x+p))


mostre que f(x+1) -f(x)= 1/ ((x+1)... (x+p+1))

aplique a soma de ambos lados com x variando de 0 até infinito

o resultado dá 1/(p .p!)


[obm-l] Re: [obm-l] Re: [obm-l] soma de série

2011-01-17 Por tôpico Rodrigo Renji
olá

Mas essa série nem é telescópica não?

ai teria que ter frações parciais mais alguma coisinha, pois o
resultado dá irracional

por exemplo em manipulação ingênua

sum (k=1 até infinito ) 1/(4k+1)-1/(4k+3) = sum (k=1 até infinito )
integral (0 até 1) x^{4k} -x^{4k+2}dx=

supondo que pode trocar ordem de integral com série e usando série geométrica

=integral (0 até 1) (1-x^2)/(1-x^4)dx= integral (0 até 1) 1/(1+x^2)dx
= arctg(1)-arctg(0) =pi/4

esse resultado é 2s, logo a soma que pediu é pi/8

*tinha visto essa manipulação numa prova da obmu

(se quiser um texto com várias manipulações dessas, posso enviar o
link de um pdf para download gratuito)

abraço

=
Instruções para entrar na lista, sair da lista e usar a lista em
http://www.mat.puc-rio.br/~obmlistas/obm-l.html
=


[obm-l] Re: [obm-l] Re: [obm-l] Re: [obm-l] construir bijeção

2011-01-15 Por tôpico Rodrigo Renji
Olá, tentei generalizar um pouquinho a demonstração

Seja C um conjunto infinito, construir uma bijeção entre C e
C\{a1,..., a_p}
(quer dizer, bijeção entre C e C menos um número p de pontos )

Tomamos
A={ a_{p+1}, a_{p+2},... } subconjunto de C (pode ser feito, pois todo
conjunto infinito possui subconjunto enumerável )

(**)Definimos B={a_1,a2,...,a_p} u A= {a_1,a2,...,a_p, a_{p+1},...}

f restrita à B como
f(a_1)=a_{p+1}, f(a_2)=a_{p+2},.., f(a_t)= a_{p+t}
tal restrição é injetiva e sua imagem é A

(**)
Definimos agora
f restrita à C\B como
f(x)=x, ela é injetiva e sua imagem é C\B

logo fica definida f de C\B U B=C
com imagem C\B U A =C\ {a1,..., a_p}

sendo injetiva e sobrejetiva, logo bijetiva .


Com isso conseguimos bijeção entre
C e C\{a_1,...,a_p}
onde C é infinito

Quer dizer, podemos picotar o conjunto C tirando um número finito
de pontos e temos essa bijeção construída

Por exemplo
bijeção entre
[0,1] e (0,1), nesse caso tiramos 0 e 1

bijeção entre

[0,1] e (0,1], tiramos 0

Bijeção entre
[0,1] e (0, 1/2 ) U (1/2 , 1)
tiramos três pontos 0, 1/2 e 1

Valeu ;*

=
Instruções para entrar na lista, sair da lista e usar a lista em
http://www.mat.puc-rio.br/~obmlistas/obm-l.html
=


[obm-l] Re: [obm-l] construir bijeção

2011-01-14 Por tôpico Rodrigo Renji
Olá

Li uma vez sobre esse modo de construir bijeção entre [0,1] e  (0,1 )

tome o conjunto A={1/2 , 1/3,  1/4,..., 1/(n+1), }

e B={0,1} u A

definimos f [0,1]  em (0,1) como

f restrita à B como f(0)=1/ 2 , f(1) =1/3 ,  e f( 1/ (n+1)  ) =1/ (n+3)

*(1)* logo a imagem dessa restrição é o conjunto A .

definimos agora a função f restrita à (0,1 ) \ A
como f(x)=x .

Temos que
**Ambas restrições são injetoras .

**A união desses conjuntos em que definimos a função dá o intervalo = [0,1]
=
[0,1] =(0,1)\A u {0,1} u A

*** a função é sobrejetiva, pois se y em (0,1 ) e A então por *(1)* existe x
em B tal que f(x)=y

se y em (0,1)\ A então  basta tomar o mesmo valor no dominio pois temos
f(y)=y nesse conjunto .

Então a função definida dessa maneira é sobrejetiva e injetiva, logo bijeção
entre os conjuntos .


[obm-l] Re: [obm-l] RE: [obm-l] Indução?

2010-12-15 Por tôpico Rodrigo Renji
Olá,  outra maneira

Primeiro demonstre a recorrência que cosseno satisfaz

cos [(n+1)a] =2cos (n a) .cos (a) -cos [(n-1)a]


usando indução de segunda forma . Para n=1 ok a propriedade vale,
supondo que vale para todo 0k n+1 vamos mostrar que vale para n+1

por hipótese de indução
2 cos [(n)a ] 2 cos(a) =  (x^n+1/x^n) (x+1/x)
que multiplicando dá
x^(n+1) +1/ x^(n+1) + x^(n-1)+ 1/x^(n-1)  onde por hipótese de indução
esse último termo é
2cos ((n-1)a)

disso segue que x^(n+1) +1/ x^(n+1)  =2 ( cos [(n)a ] 2 cos(a) -cos ((n-1)a))

logo pela primeira recorrência segue que
x^(n+1) +1/ x^(n+1)  =cos [(n+1)a] .

=
Instruções para entrar na lista, sair da lista e usar a lista em
http://www.mat.puc-rio.br/~obmlistas/obm-l.html
=


[obm-l] Re: [obm-l] RES: Sequência

2010-11-19 Por tôpico Rodrigo Renji
Olá  ( de novo :) vim só comentar uma curiosidade

Por coincidência ontem vi um problema que só consegui resolver usando
esse resultado  (Stolz-Cesáro)

era assim, calcular o limite

lim [2ln (2) +...+n ln (n) ]  /n² ln(n)

ai com o resultado transforma a soma em uma sequência e depois calcula o limite
(pensei que poderia sair daquele resultado de transformar em integral,
mas não consegui )


abraço

Em 18 de novembro de 2010 11:06, Rodrigo Renji
rodrigo.uff.m...@gmail.com escreveu:
 Fiz um pequena besteira no email anterior =/ ( colei de novo a mesma
 mensagem, pois tinha tirado a formatação ), então estou enviando de
 novo com alguns comentários adicionais

 Olá :)
 Tem um resultado de onde sai com facilidade essa propriedade da média
 de Cesáro , a seguinte( cuja demonstração não é complicada )


 (Stolz-Cesàro)
 Seja (y_n) uma sequência crescente e ilimitada ( logo lim y_n= infinito)

 (Vou denotar D x(n)= x(n+1) -x(n))

 Se vale lim (D x(n) ) / ( D y(n)) =L então vale lim  x(n)/y(n) = L

 Demonstração, para n grande vale (nn_0)
  L-eD x_n /Dy_n  L+e
 com D y(n) 0 pois y(n) é crescente e y(n)0 pois y(n) tende  infinito

 logo podemos multiplicar por D y(n)

 (L-e)Dy_n D x_n  (L+e) Dy_n somamos de n_0+1 até n de cada lado ( as
 somas são telescópicas), de onde segue

 (L-e)(y(n+1)- y(n_0+1 ))  x_(n+1) -x(n_0+1)  (L+e) (y(n+1) - y(n_0 +1))

 podemos dividir por y(n+1)0

 (L-e)(1- y(n_0+1 )/y(n+1)) +x(n_0+1) /y(n+1) x_(n+1)/y(n+1)  (L+e)
 (1- y(n_0 +1)/y(n+1)) +x(n_0+1) /y(n+1)


 para n grande ( como lim yn = infinito ) segue dessa desigualdade que


 (L-e) x_(n+1)/y(n+1)  (L+e)

 logo lim x(n)/ y(n) =L .


 Agora como corolário
 (média de cesàro)
 Se lim x_n= a então lim ( x_1 ++x_n)/ n= a

 tomando s_n= x_1 ++x_n e b_n=n tem-se Ds_n= x(n+1) e D b_n=1

 logo pela hipótese lim D s_n /D b_n = lim x(n+1) =a
 como, b_n=n é crescente e ilimitada então vale Stolz-Cesáro

 logo
 lim s_n/ n=  lim  ( x_1 ++x_n)/ n =a .


 Esse resultado de Stolz-Cesàro, pode ser visto como análogo a regra de
 L hospital para sequências . O operador D ( que normalmente se escreve
 como um Delta ), pode ser pensado como o análogo de derivada para
 sequências ( tem varias analogias entre esses operadores.

 Ah, esse resultado tem no livro do elon de análise, como exercício .


 Pra limite infinito também vale a mesma propriedade

  (Stolz-Cesáro  para limite infinito)
 Se b_n é crescente e ilimitada e vale
 lim D a_n /Db_n = infinito
 então lim a_n /b_n = infinito

 demonstração no mesmo esquema
 para nn_0 vale e qualquer A0 vale
  D a_n /Db_n A daí podemos multiplicar por D b_n pois Db_n0 ( b_n é
 crescente)
 D a_n  A .D b_n
 somamos de n_0+1 até n com o indice variando e usando a propriedade 
 telescópica

 a(n+1)-a(n_0 +1)  A (b(n+1) - b(n_0+1))

 a(n+1)  A (b(n+1) - b(n_0+1)) +a(n_0 +1)

 dividimos por b_(n+1)0
 a(n+1) /b(n+1)  A (1 - b(n_0+1) /b(n+1)) +a(n_0 +1)/b(n+1)

 como lim b(n+1) = infinito, para n grande vale

 a(n+1) /b(n+1)   A

 logo lim a(n) /b(n)= infinito

 daí de novo o resultado que gerou esse email sai como corolário .

 Se lim a_n= infinito então lim (a1 +...+an) /n= infinito

 tomamos s_n=a1+...+an, vale D s_n= a(n+1) e b_n=n, b_n é crescente e ilimitada
 e vale
 lim D s_n / D b_n = lim a(n+1) /1= infinito
 logo lim s_n /n = lim (a1 +...+an) /n = infinito

 abraço


=
Instruções para entrar na lista, sair da lista e usar a lista em
http://www.mat.puc-rio.br/~obmlistas/obm-l.html
=


[obm-l] Re: [obm-l] RES: Sequência

2010-11-18 Por tôpico Rodrigo Renji
Olá :)
Tem um resultado de onde sai com facilidade essa propriedade da média
de Cesáro , a seguinte( cuja demonstração não é complicada )

(Stolz-Cesàro)
Seja (y_n) uma sequência crescente e ilimitada ( logo lim y_n= infinito)

(Vou denotar D x(n)= x(n+1) -x(n))

Se vale lim (D x(n) ) / ( D y(n)) =L então vale lim  x(n)/y(n) = L

Demonstração, para n grande vale (nn_0)

 L-eD x_n /Dy_n  L+e

com D y(n) 0 pois y(n) é crescente e y(n)0 pois y(n) tende  infinito

logo podemos multiplicar por D y(n)

(L-e)Dy_n D x_n  (L+e) Dy_n somamos de n_0+1 até n de cada lado ( as
somas são telescópicas), de onde segue

(L-e)(y(n+1)- y(n_0+1 ))  x_(n+1) -x(n_0+1)  (L+e) (y(n+1) - y(n_0 +1))

podemos dividir por y(n+1)0

(L-e)(1- y(n_0+1 )/y(n+1)) +x(n_0+1) /y(n+1) x_(n+1)/y(n+1)  (L+e)
(1- y(n_0 +1)/y(n+1)) +x(n_0+1) /y(n+1)


para n grande ( como lim yn = infinito ) segue dessa desigualdade que

(L-e) x_(n+1)/y(n+1)  (L+e)
logo lim x(n)/ y(n) =L .

Agora como corolário

Olá :)
Tem um resultado de onde sai com facilidade essa propriedade da média
de Cesáro , a seguinte( cuja demonstração não é complicada )


(Stolz-Cesàro)
Seja (y_n) uma sequência crescente e ilimitada ( logo lim y_n= infinito)

(Vou denotar D x(n)= x(n+1) -x(n))

Se vale lim (D x(n) ) / ( D y(n)) =L então vale lim  x(n)/y(n) = L

Demonstração, para n grande vale (nn_0)
 L-eD x_n /Dy_n  L+e
com D y(n) 0 pois y(n) é crescente e y(n)0 pois y(n) tende  infinito

logo podemos multiplicar por D y(n)

(L-e)Dy_n D x_n  (L+e) Dy_n somamos de n_0+1 até n de cada lado ( as
somas são telescópicas), de onde segue

(L-e)(y(n+1)- y(n_0+1 ))  x_(n+1) -x(n_0+1)  (L+e) (y(n+1) - y(n_0 +1))

podemos dividir por y(n+1)0

(L-e)(1- y(n_0+1 )/y(n+1)) +x(n_0+1) /y(n+1) x_(n+1)/y(n+1)  (L+e)
(1- y(n_0 +1)/y(n+1)) +x(n_0+1) /y(n+1)


para n grande ( como lim yn = infinito ) segue dessa desigualdade que


(L-e) x_(n+1)/y(n+1)  (L+e)

logo lim x(n)/ y(n) =L .


Agora como corolário
(média de cesàro)
Se lim x_n= a então lim ( x_1 ++x_n)/ n= a

tomando s_n= x_1 ++x_n e b_n=n tem-se Ds_n= x(n+1) e D b_n=1

logo pela hipótese lim D s_n /D b_n = lim x(n+1) =a
como, b_n=n é crescente e ilimitada então vale Stolz-Cesáro

logo
lim s_n/ n=  lim  ( x_1 ++x_n)/ n =a .


Esse resultado de Stolz-Cesàro, pode ser visto como análogo a regra de
L hospital para sequências . O operador D ( que normalmente se escreve
como um Delta ), pode ser pensado como o análogo de derivada para
sequências ( tem varias analogias entre esses operadores)

=
Instruções para entrar na lista, sair da lista e usar a lista em
http://www.mat.puc-rio.br/~obmlistas/obm-l.html
=


[obm-l] Re: [obm-l] RES: Sequência

2010-11-18 Por tôpico Rodrigo Renji
Fiz um pequena besteira no email anterior =/ ( colei de novo a mesma
mensagem, pois tinha tirado a formatação ), então estou enviando de
novo com alguns comentários adicionais

Olá :)
Tem um resultado de onde sai com facilidade essa propriedade da média
de Cesáro , a seguinte( cuja demonstração não é complicada )


(Stolz-Cesàro)
Seja (y_n) uma sequência crescente e ilimitada ( logo lim y_n= infinito)

(Vou denotar D x(n)= x(n+1) -x(n))

Se vale lim (D x(n) ) / ( D y(n)) =L então vale lim  x(n)/y(n) = L

Demonstração, para n grande vale (nn_0)
 L-eD x_n /Dy_n  L+e
com D y(n) 0 pois y(n) é crescente e y(n)0 pois y(n) tende  infinito

logo podemos multiplicar por D y(n)

(L-e)Dy_n D x_n  (L+e) Dy_n somamos de n_0+1 até n de cada lado ( as
somas são telescópicas), de onde segue

(L-e)(y(n+1)- y(n_0+1 ))  x_(n+1) -x(n_0+1)  (L+e) (y(n+1) - y(n_0 +1))

podemos dividir por y(n+1)0

(L-e)(1- y(n_0+1 )/y(n+1)) +x(n_0+1) /y(n+1) x_(n+1)/y(n+1)  (L+e)
(1- y(n_0 +1)/y(n+1)) +x(n_0+1) /y(n+1)


para n grande ( como lim yn = infinito ) segue dessa desigualdade que


(L-e) x_(n+1)/y(n+1)  (L+e)

logo lim x(n)/ y(n) =L .


Agora como corolário
(média de cesàro)
Se lim x_n= a então lim ( x_1 ++x_n)/ n= a

tomando s_n= x_1 ++x_n e b_n=n tem-se Ds_n= x(n+1) e D b_n=1

logo pela hipótese lim D s_n /D b_n = lim x(n+1) =a
como, b_n=n é crescente e ilimitada então vale Stolz-Cesáro

logo
lim s_n/ n=  lim  ( x_1 ++x_n)/ n =a .


Esse resultado de Stolz-Cesàro, pode ser visto como análogo a regra de
L hospital para sequências . O operador D ( que normalmente se escreve
como um Delta ), pode ser pensado como o análogo de derivada para
sequências ( tem varias analogias entre esses operadores.

Ah, esse resultado tem no livro do elon de análise, como exercício .


Pra limite infinito também vale a mesma propriedade

 (Stolz-Cesáro  para limite infinito)
Se b_n é crescente e ilimitada e vale
lim D a_n /Db_n = infinito
então lim a_n /b_n = infinito

demonstração no mesmo esquema
para nn_0 vale e qualquer A0 vale
  D a_n /Db_n A daí podemos multiplicar por D b_n pois Db_n0 ( b_n é
crescente)
D a_n  A .D b_n
somamos de n_0+1 até n com o indice variando e usando a propriedade telescópica

a(n+1)-a(n_0 +1)  A (b(n+1) - b(n_0+1))

a(n+1)  A (b(n+1) - b(n_0+1)) +a(n_0 +1)

dividimos por b_(n+1)0
a(n+1) /b(n+1)  A (1 - b(n_0+1) /b(n+1)) +a(n_0 +1)/b(n+1)

como lim b(n+1) = infinito, para n grande vale

a(n+1) /b(n+1)   A

logo lim a(n) /b(n)= infinito

daí de novo o resultado que gerou esse email sai como corolário .

Se lim a_n= infinito então lim (a1 +...+an) /n= infinito

tomamos s_n=a1+...+an, vale D s_n= a(n+1) e b_n=n, b_n é crescente e ilimitada
e vale
lim D s_n / D b_n = lim a(n+1) /1= infinito
logo lim s_n /n = lim (a1 +...+an) /n = infinito

abraço

=
Instruções para entrar na lista, sair da lista e usar a lista em
http://www.mat.puc-rio.br/~obmlistas/obm-l.html
=


[obm-l] Re: [obm-l] Re: [obm-l] [obm-l] Sequência

2010-11-16 Por tôpico Rodrigo Renji
Olá, tentei escrever uma solução de maneira diferente ( só não sei se está
certa)( mas acho que no fim segue a mesma linha da solução do hugo)
podemos considerar a sequência como de termos positivos, pois para n grande
x_nA0 e se lim x(n+p) = infinito então lim x (n)= infinito.

Então para qualquer A0 existe n_0 natural tal que  kn_0 implica x_k2A
tomamos n2n_0 ( daí  (n -n_0)/n 1/2 )

logo
(x_1 +...+x_n)/n   (x_(n_0+1)+...+x_n)/n (n-n_0)/n   (2A) 1/2  .  2.A =A

logo s_n:= (x_1 +...+x_n)/nA, com A0 arbitrário então lim s_n= infinito


Re: [obm-l] Para mim, 0^0=1

2010-09-23 Por tôpico Rodrigo Renji
Olá :)

Existem várias identidades que dão  certo quando se tem 0^0=1

**Binômio de Newton
por exemplo, o binômio de newton funciona em casos triviais ( usando c(n,k)
pro coeficiente binomial )

(1+x)^n = soma (k= 0 até n) c( n, k) x^k

tomando x=-1 tem-se

(0)^n = soma (k= 0 até n) c( n, k) (-1)^k

tomando n=0
soma (k= 0 até 0) c( 0, k) (-1)^k = c( 0, 0) (-1)^0=1

além do caso
(y+x)^n = soma (k= 0 até n) c( n, k) x^k y^(n-k)

se x=0, caso trivial, deve dar y^n

soma (k= 0 até n) c( n, k) 0^k y^(n-k)
só resta o termo em k=0

que é c( n, 0) 0^0 y^(n) para isso dar y^n devemos ter
0^0=1 pois c(n,0)=1.

** Soma geométrica
A fórmula da soma geométrica funciona em todos casos quando temos 0^0=1


soma (k=0 até n) x^k =  (x^{n+1} -1) / (x-1)

para x diferente de 1

se x=0
soma (k=0 até n) 0^k =  (0^{n+1} -1) / (0-1) = -1/-1 =1

do lado esquerdo só resta o termo 0^0


** série exponencial

e^x = soma (k=0 até infinito ) x^k/k!
que converge para todo x real( 0 incluído)
se x=0
e^0=1

e^0 = soma (k=0 até infinito ) 0^k/k!

mais uma vez só resta o termo com k=0

0^0/k! =e^0, 0^0=1

sempre quando usamos notação compacta


** Produtório

Definimos
a^n= produto (k=1 até n) a
com a real e n natural (0 incluído) .

definimos produtório

produto (k=a até a) f(k) = f(a) ( condição inicial )
e com a propriedade de abrir um produtório em dois

produto (k=a até b) f(k) = produto (k=a até p) f(k) . produto (k=p+1 até b)
f(k) ( recorrência)

podemos com isso dar sentido a coisas como

produto (k=a até a-1) f(k), quando o limite superior é menor que o inferior,
pela relação acima tomando p=a-1 tem-se

*produto (k=a até b) f(k)* = produto (k=a até a-1) f(k) . *produto (k=a até
b) f(k)*
*
*
as partes em negrito são iguais, então é necessário que
produto (k=a até a-1) f(k) seja o elemento neutro do produto, então produto
(k=a até a-1) f(k)=1
que podemos chamar de produto vazio

logo voltando a definição de potência por produtório

a^0= produto (k=1 até 0) a =1

independente do valor de a, incluíndo 0, pois caímos num produto vazio.

O mesmo pode ser usado para definir fatorial de forma compacta

produto (k=1 até n)  k =n!
para todo n natural, caso n=0 temos um produto vazio logo
0!=1

tentei fazer um pdf mostrando alguns desses casos, se alguém quiser dar uma
olhada
http://www.4shared.com/dir/HLZtU_v7/zeroazero.html

Abraço

Rodrigo














Em 16 de setembro de 2010 21:50, Johann Dirichlet
peterdirich...@gmail.comescreveu:

 Nessas horas eu me pergunto: por que existem tantas arestas
 não-aparadas na matemática?

 A aresta mais pontuda, na minha opinião, é o paradoxo de
 Banach-Tarski: é possível desmontar uma bolinha de gude e juntar os
 pedaços de modo a se obter uma bola do tamanho do sol.

 Em 16/09/10, Ralph Teixeiraralp...@gmail.com escreveu:
  Eu sou um dos defensores de 0^0=1. Apresento dois motivos:
 
  i) Se f(x) e g(x) sao analiticas em 0 com f(x),g(x)-0 quando x-a, entao
  f^g - 1 quando x- a (bom, desde que f^g faca sentido em volta de x=a).
 A
  *unica* excecao a esta regra eh o caso em que f eh identicamente nula,
  quando o limite dah 0 (se f^g faz sentido) ou nao existe (se g0 ali por
  perto de x=a).
  Isto explica porque 99.9% dos exercicios de limite que ficam da forma
 0^0
  acabam dando 1 como resposta!
 
  Acho que isto tambem explica porque eu nao faria 0/0=1 ou algo assim --
 nao
  ha teorema semelhante para 0/0.
 
  ii) Como escrever um polinomio generico de grau 17 usando somatorios?
 Acho
  que muita gente concorda que uma boa representacao eh:
  p(x) = SUM (n=0 a 17) a_n x^n
  onde os a_n sao coeficientes arbitrarios. Agora eu pergunto -- quanto
 vale
  p(0)?
 
  Com a convencao 0^0=1, nada especial precisa ser feito, eh soh substituir
  x=0 no somatorio.
 
  Com a convencao 0^0 nao existe bom, ai a nossa representacao por
  somatorio ficaria tecnicamente errada. Teriamos que escrever:
 
  p(x) = SUM (n=0 a 17) a_n x^n, se x0
  p(x) = a_0, se x=0
 
  ou entao tirar o x^0 do somatorio:
 
  p(x) = a_0 + SUM (n=1 a 17) a_n x^n
 
  (e se voce acha que esta ultima eh bem razoavel -- escreva p'(x). Separou
 o
  a1? Argh!)
 
  Como eu nao tenho paciencia de ficar escrevendo este a_0 separado toda
 hora,
  prefiro logo pensar que 0^0=1 e resolvo meus problemas com um somatorio
 soh.
  :)
 
  Isto tudo dito, claro que eh soh uma convencao, questao de gosto. Mas eu
  *gosto* de 0^0=1. :)
 
  Abraco,
   Ralph
 
  2010/9/16 Jorge Luis Rodrigues e Silva Luis jorgelrs1...@hotmail.com
 
  Olá, Pessoal! Vale lembrar que o símbolo do nada está entre as mais
  importantes descobertas feita pelo homem. É difícil acreditar que os
  homens
  levaram 5 mil anos entre escrever números e conceber o nosso sistema de
  numeração posicional, ponto crucial num desenvolvimento sem o qual o
  progresso da ciência moderna seria inconcebível. Hoje parece simples,
 mas
  a
  mentalidade concreta dos antigos gregos, não podia conceber o vazio, o
  nada,
  como um número. Apreciaremos ainda mais a grandeza dessa 

[obm-l] Re: [obm-l] CONCEITOS E CONTROVÉRSIAS!

2010-09-16 Por tôpico Rodrigo Renji
...O curioso é que os defensores de 0^0=1 não reivindiquem o mesmo direito
para 0/0. Algum colega saberia o motivo?...

Acho que o motivo de associar 0^0 com 0/0 é errado, se fosse associar 0^0
com 0/0 isso também poderia ser feito com o 0, por causa do seguinte


o pessoal argumenta pela regra de expoentes, supondo que fosse verdadeira
para base 0

que  0^0 = 0^ (1 -1) = 0^1 /0^1 = 0/0


porém isso pode ser feito com o 0 também

0= 0^ (2-1) = 0^2/0^1 =0/0

então o 0 também seria  0/0 ? e seria então indefinido?

o erro é usar isso com base  0, o que não pode ser feito . Quando se
demonstra essa proprieda a^( b-c )= a^b/a^c , por exemplo, definindo
potenciação pra expoente natural e base real e depois expoente inteiro, essa
propriedade se demonstra, deixando fora o caso em que a=0 .


[obm-l] Tex na lista de discussão

2010-04-03 Por tôpico Rodrigo Renji
Olá : ), então, existem alguns plugins para os navegadores firefox e
chrome, que permitem visualização de comandos em tex no navegador

Para o navegador FIREFOX
usando o firefox[http://baixaki.ig.com.br/download/Mozilla-Firefox.htm]
e a extensao Greasemonkey[https://addons.mozilla.org/pt-BR/firefox/addon/748]
mais um script http://thewe.net/tex/textheworld6.user.js

CHROME
Para o navegador CHROME
https://chrome.google.com/extensions/detail/mbfninnbhfepghkkcgdnmfmhhbjmhggn

os códigos em tex devem ser colocados entre  [;   e  ;]
por exemplo o código
[; \int^x_a f(z) dz ;]


é visualizado como integral no navegador com os plugins instalados
como na imagem
http://i317.photobucket.com/albums/mm387/matcult/u2.gif

Bem não sei se vocês achariam interessante ter esses plugins e usar na
lista de discussão, acho que melhoraria bem mais a visualização das
expressões matemática.

Abraço
Rodrigo

=
Instruções para entrar na lista, sair da lista e usar a lista em
http://www.mat.puc-rio.br/~obmlistas/obm-l.html
=


[obm-l] Re: Re: [obm-l] Re: [obm-l] Re: [obm-l] Número Harm ônico

2010-03-31 Por tôpico Rodrigo Renji
Tem outra maneira de achar uma fórmula fechada não elementar para os
números harmônicos. Usando a função gamma, que satisfaz

Gamma (x+1) =x Gamma (x)

tomando o logaritmo de ambos lados segue

 ln gamma (x+1) = \ln x + \ln gamma (x)

derivando

gamma ' (x+1)/ gamma (x+1) = 1/x + gamma' (x) / gamma(x) **(1)**


Podemos definir com isso a função digamma ( as vezes simbolizada como
psi também ), da seguinte maneira
psi (x) = gamma ' (x) / gamma (x)

observe em  **(1)** que
psi (x+1) = 1/x +psi (x)

e daí

psi (x+1) - psi (x) =1/x
psi (k+1) - psi (k) =1/k

aplique a soma na expressão acima com k variando de 1 até n , a soma
do lado esquerdo é telescópica ( os termos vão se anulando) e sobra
apenas

psi (n+1)-psi (1) =H_n= soma [de k=1 até n] 1/k

é possível mostrar que
-psi (1) = gamma onde gamma é a constante de Euler- Mascheroni, é um
problema aberto saber se essa constante é racional ou irracional ainda
(desde a época de Euler)
Alguns links
http://www.math.upenn.edu/~wilf/AeqB.html
Livro A=B, gratuito (download legal),

quero deixar também o link da minha pasta no 4shared, onde tenho
escrito em pdf essas coisas que escrevi no email, só que com mais
detalhes
http://www.4shared.com/dir/10890465/8ca60bc6/Somatrios.html
Abraço

=
Instruções para entrar na lista, sair da lista e usar a lista em
http://www.mat.puc-rio.br/~obmlistas/obm-l.html
=


[obm-l] Re: Re: [obm-l] Re: [obm-l] Re: [obm-l] Número Harm ônico

2010-03-31 Por tôpico Rodrigo Renji
Acho que não existe fórmula fechada em termos de funções elementares
para o n-ésimo número harmônico
H_n=1+...+1/n

(H_n acho que é o simbolo usado pelo knuth no concrete mathematics)
(assim como não existem primitivas elementares para algumas funções)

quando isso acontece podemos tentar escrever fórmulas usando outros
tipos de funções, por exemplo, representar H_n por meio de uma
integral.

Acho que Euler foi o primeiro a fazer isso, talvez da maneira como segue

H_n= soma [de k=1 até n] 1/k
que é o mesmo que
 [ k=0 até n-1] 1/ (k +1)

mas observe que 1/(k+1) pode ser escrito como uma integral
1/(k+1)= integral [de 0 até 1] x^k dx, escrevendo isso na soma fica

H_n = soma [de k=0 até n-1] integral [de 0 até 1] x^k dx
como a soma é finita, podemos trocar a ordem com a integral, ficando

H_n =integral [de 0 até 1] soma [de k=0 até n-1] x^k dx

aqui soma [de k=0 até n-1] x^k temos uma soma geométrica que é (x^n -1) /(x-1)

então chegamos na expressão

H_n = integral [de 0 até 1] (x^n -1) /(x-1) dx

isso vale para n natural
mas podemos usar isso para fazer uma extensão dos números harmônicos
para valores reais de n pela integral acima
Perceba que para n natural vale
H_(n+1) =H_n +1/(n+1)
isso é também vale para alguns valores reais de n
por meio da extensão da integral acima
por exemplo, um resultado de série por meio de números harmônicos
soma [ de k=1 até infinito ] 1/ (k)(k+s) =H_s /s

por exemplo com a integral podemos mostrar que
H(1/2) =2-2ln( 2)
e com isso
soma [ de k=1 até infinito ] 1/ (k)(k+1/2)  =H(1/2).2=
4-4ln 2, dando o resultado correto da série

=
Instruções para entrar na lista, sair da lista e usar a lista em
http://www.mat.puc-rio.br/~obmlistas/obm-l.html
=


[obm-l] Série

2009-02-03 Por tôpico rodrigo renji abarai
Achar o valor (número fechado?) para o qual converge a série

0^p /0! +1^p/1! +...+n^p/n!+...

em função de p um número natural.

=
Instruções para entrar na lista, sair da lista e usar a lista em
http://www.mat.puc-rio.br/~obmlistas/obm-l.html
=


Re: [obm-l] Dúvidas de Recorrencias

2008-09-07 Por tôpico Rodrigo Renji
T(1)=1
T(n)=T(n-2) + 2n + 1

essa primeira
faça na recorrencia n+2 ao inves de n, ficando com
T(n+2)=T(n+2-2) + 2(n+2) + 1
T(n+2)=t(n)+2n+4+1=t(n)+2n+5
temos então  a recorrencia
t(n+2)=t(n)+2n+5
seja E^k o operador que faz E^k f(n)=f(n+k), escrevemos
essa recorrencia como

(E^2-1) t(n)=2n+5
(E-1)(E+1)t(n)=2n+5
o operador (E-1) abaixa grau de polinomio e o operador (E+1) anula
termo do tipo c(-1)^n
então a solução tem que ser do tipo
t(n)=an^2+bn+c+d(-1)^n
agora achando os coeficientes, aplicando (E+1) o termo d(-1)^n some
agora temos que aplicar (E-1)(E+1) no polinomio, aplicando primeiro
(E-1) (esses operadores comutam)

temos
(E-1)t(n)=a(2n+1)+b
aplicando (E+1) agora
(E+1)(E-1)t(n)=a(2(n+1)+1)+b+a(2n+1)+b=a(2n+2+1)+2b+a(2n+1)=a(2n+3)+2b+a(2n+1)=
a(4n+4)+2b=4n.a+4a+2b que tem que ser igual a 2n+5
temos então 4n.a+4a+2b=2n+5
dai tiramos 4a=2, a=1/2
4/2 +2b=5 , 2+2b=5   2b=3, b=3/2

então t(n), fica da forma
n^2 /2 +3/2n+c+d(-1)^n=t(n)

usando a condição inicial
t(1)=1

temos
1/2 +3/2 +c -d=1
2 +c-d=1
1+c=d
assim a solução fica

n^2 /2 +3/2n+c+(1+c)(-1)^n=t(n)
n^2/2 +3/2 n +c + (-1)^n +c (-1)^n=t(n)

perceba que se n é impar (-1)^n=-1
então t(n) fica da forma
n^2/2 +3/2 n +c -1 -c=n^2/2 +3/2 n -1=t(n)
agora para os pares
n^2/2 +3/2 n +c +1+c=t(n)
mostrando que precisamos de mais uma condição inicial para determinar
o coeficiente c


2008/9/6 Rafael Ando [EMAIL PROTECTED]:
 Para a segunda, olha só... T(2) = T(1)+2 = 1+2 = 3. A sua fórmula dá T(2) =
 (2²-1)/2 = 3/2, então não está certo não :(

 Podemos fazer assim:

 T(n) = n + T(n-1)
 = n + (n-1 + T(n-2)) = ... = n + (n-1) + (n-2) + ...+ 1.

 Logo T(n) = n*(n+1)/2, ou (n² + n) /2.

 Para a primeira T é uma função definida apenas nos valores impares? Com
 os dados apresentados T poderia ser qualquer coisa nos pares...

 On Fri, Sep 5, 2008 at 11:04 PM, Venildo Amaral [EMAIL PROTECTED]
 wrote:

 Estou com uma dúvida em como resolver essas duas recorrências, cheguei a
 um ponto que não consigo achar a forma fechada das mesmas.

 T(1)=1
 T(n)=T(n-2) + 2n + 1 ???

 outra

 T(1)=1
 T(n)=T(n-1) + n, essa aqui cheguei na forma fechada de (n^2-1)/2, mas não
 sei se esta certo.


 Atenciosamente,
 Venildo Junio do Amaral
 [EMAIL PROTECTED]
 www.venildo.mat.br
 http://venildo.dv01.discovirtual.ws - Diretório Virtual
 Home Work
 (11) 4748-0159 / (11) 9167-1450



 --
 Rafael


=
Instruções para entrar na lista, sair da lista e usar a lista em
http://www.mat.puc-rio.br/~obmlistas/obm-l.html
=


Re: [obm-l] Número de quadrados

2008-07-10 Por tôpico Rodrigo Renji
Se quiserem alguns números dessa sequência, tem aqui nesse link
http://www.research.att.com/~njas/sequences/?q=1%2C3%2C7%2C13%2C22%2C34%2C50amp;amp;sort=0fmt=0language=englishamp;go=Search

 eu cheguei na formula n³/12 +3n²/8+5n/12 +1/16 -1/16 (-1)^n =f(n)


2008/7/10 Wanderley Guimarães [EMAIL PROTECTED]:
 Eu resolvi um pouco diferente.

 Quantos quadrados 1x1 podemos formar?
 (n+1 escolhe 2)

 Quantos quadrados 2x2 podemos formar?
 (n escolhe 2)

 ...

 Então temos Somatorio de i = 2 até n + 1 de (i escolhe 2) = 2^(n+1)

 Errei em algum canto?

 On Wed, Jul 9, 2008 at 6:51 PM, Felipe Diniz [EMAIL PROTECTED] wrote:
 Para uma escada de tamanho n, seja F(n) o numero de quadrados
 temos que
 F(n)=quadrados que nao englobem a primeira coluna + quadrados que englobem a
 primeira coluna.
 quadrados que nao englobem a primeira coluna  = F(n-1)

 para n par:
 quadrados que englobem a primeira coluna:
  1 + 2 + 3 + 4+... + k+k+ (k-1)+(k-2)+...+1 = 2(1+2+3..+k) = k(k+1), onde k
 eh o maior inteiro tal que 2k+1=n, como n e' par n=2m
 2k+1=2m = k= m+1/2, logo k = m= n/2
 para n impar:
 quadrados que englobem a primeira coluna:
  1 + 2 + 3 + 4+... + k (k-1)+(k-2)+...+1 = 2(1+2+3..+k-1)+k = k^2, onde k eh
 o menor inteiro tal que 2k+1n, como n e' impar n=2m+1
 2k+12m+1 = k m, logo k = m+1= (n+1)/2

 Assim F(n)= Somatorio de k=2 ate n  de A(k) + F(1)
 onde A(n)= n/2 ( n/2 + 1) se n e` par, e [(n+1)/2]^2 se n e` impar.
 Assim:
 F(2n) = n(n+1)+somatorio de k=1 ate n-1   A(2k)+A(2k+1) + F(1) = 1 + n(n+1)+
 somatorio de k=1 ate n-1 de 2k^2 + 3k+1 =
 1+ n(n+1)+ n-1 + 3(n-1)n/2 + 2 (n-1)n(2n-1)/6

 F(2n+1)= somatorio de k=1 ate n   A(2k)+A(2k+1) + F(1) = 1 + Somatorio de
 k=1 ate n de 2k^2 + 3k+1 =
 1 + n + 3n(n+1)/2 + 2n(n+1)(2n+1)/6

 fiz meio rapido espero estar certo...


 Felipe Diniz

 On Wed, Jul 9, 2008 at 12:05 PM, Rodrigo Renji [EMAIL PROTECTED]
 wrote:

 Na seguinte figura (link no photobucket)


 http://s317.photobucket.com/albums/mm387/matcult/?action=viewcurrent=quadrados2.jpg


 Queremos saber o número máximo de quadrados  de qualquer tamanho
 formados pelos quadrados unitários, numa escada com n degrais

 =
 Instruções para entrar na lista, sair da lista e usar a lista em
 http://www.mat.puc-rio.br/~obmlistas/obm-l.html
 =





 --
 Wanderley Guimarães

 =
 Instruções para entrar na lista, sair da lista e usar a lista em
 http://www.mat.puc-rio.br/~obmlistas/obm-l.html
 =


=
Instruções para entrar na lista, sair da lista e usar a lista em
http://www.mat.puc-rio.br/~obmlistas/obm-l.html
=


[obm-l] Número de quadrados

2008-07-09 Por tôpico Rodrigo Renji
Na seguinte figura (link no photobucket)

http://s317.photobucket.com/albums/mm387/matcult/?action=viewcurrent=quadrados2.jpg


Queremos saber o número máximo de quadrados  de qualquer tamanho
formados pelos quadrados unitários, numa escada com n degrais

=
Instruções para entrar na lista, sair da lista e usar a lista em
http://www.mat.puc-rio.br/~obmlistas/obm-l.html
=


Re: [obm-l] FATORIAL DE ZERO

2008-06-20 Por tôpico Rodrigo Renji
Olá ^.^
eu costumo pensar no fatorial como uma função definida por recorrência
uma função f(n) que satisfaz a equação funcional ou recorrencia
f(n+1)=f(n).(n+1) para n natural
sendo uma recorrência de  ordem 1 precisa de uma condição inicial
que tomamos f(0)=1, porém se tomarmos outra condição inicial como f(1)=1
podemos deduzir f(0)=1 atraves da recorrencia, pois

f(1)=1.f(0), f(1)=f(0), logo f(0)=1

se definirmos outra condição inicial f(2), ao inves de f(1) ou f(0), por exemplo
definindo f(2)=2, podemos deduzir que f(1)=1
pois

f(2)=2.f(1)=2, f(1).2=2 logo f(1)=1

e assim por diante, se definimos f(n) pra algum número natural n0,
podemos achar os outros números a partir da recorrência, pois sendo
uma recorrência de ordem 1 ela fica definida por apenas 1 condição
inicial que pode ser qualquer.

a função f(n) que satisfaz f(n+1)=f(n).(n+1) pra n natural, e a condição inicial
f(0)=1, podemos definir como fatorial de n, f(n)=n!

sendo f(n) diferente de zero pra todo n, podemos tomar

f(n+1)/f(n) =(n+1)
e tomar o produtorio de ambos lados, fazendo o n variar de n=0 até p-1

Prod [n=0,p-1] f(n+1)/f(n)= Prod [n=0, p-1](n+1)
no primeiro lado, temos uma propriedade semelhante a soma telescópica,
o resultado ficando apenas
f(p+1)/f(0) =Prod [n=0, p-1](n+1)

assim

f(p+1)=f(0).Prod [n=0, p-1](n+1)=f(0) . Prod [n=1, p](n)
isto é
f(p+1)=f(0) . Prod [n=1, p](n)
depende do valor que damos para f(0)
que é nossa condição inicial pro fatorial.


2008/6/18 Vitor Tomita [EMAIL PROTECTED]:
 Há quem diga que 0!=1 porque só há uma maneira de se permutar 0
 objetos: não fazer nada. Recorrer ao gama de Euler não permite deduzir
 isso, é uma espécie de postulado, que funciona bem na prática e por
 isso foi postulado.

 On Wed, 18 Jun 2008 20:05:12 +0100
 Bruno França dos Reis [EMAIL PROTECTED] wrote:

 Segundo o Google:
 Constante de Artur = ln(1 + arctan(e^2 - 3,79)^pi)) + cosh(pi^e+
 e^(1,21*pi)) = 7.80040173 * 10^28

 Não resisti...
 Bruno


 2008/6/18 Artur Costa Steiner [EMAIL PROTECTED]:

  Acho que nao eh um postulado, mas sim uma definicao. Da mesma forma
  que, por definicao, a^n = a**a (n vezes) para n inteiro
  positivo. Da mesma forma que, por definicao, Gama(x) = Integral (0
  a oo) e^(-t) t^(x -1) dx
 
  Se eu fosse um cara prepotente, poderia definir número de Artur
  como ln(1 + arctan(e^2 - 3,79)^pi)) + cosh(pi^e^+ e^(1,21pi.
  Contrariamente a outras cosntantes, nao serve para nada, uma
  definicao idiota, as seria uma definicao, nao um postulado.
 
  Artur
 
  -Mensagem original-
  De: [EMAIL PROTECTED] [mailto:[EMAIL PROTECTED]
  nome de Paulo Santa Rita
  Enviada em: quarta-feira, 18 de junho de 2008 13:59
  Para: obm-l@mat.puc-rio.br
  Assunto: Re: [obm-l] FATORIAL DE ZERO
 
 
  Ola Jorge e demais colegas
  desta lista ... OBM-L,
 
  Nao ha o que justificar ...  0! = 1 e um POSTULADO : tao POSTULADO
  quanto o quinto postulado de Euclides. E - assim como o famoso
  postulado euclidiano tambem foi - ele e ainda hoje um dos alicerces
  da nossa maneira de contar, pois, se o negarmos, as consequencias
  que dai advem parecem nao corresponder com a realidade com que
  estamos acostumados a lidar
 
  Mas nada pode tolher a nossa liberdade de imaginacao.
 
  Quando o Lobachevski negou o quinto postulado de Euclides e afirmou
  que por um ponto fora de uma reta era possível traçar não uma, mas
  várias retas paralelas a reta inicial dada, ele chamou os
  desenvolvimento desta LOUCA HIPOTESE de GEOMETRIA IMAGINARIA
  simplesmente porque achava que a realidade se conformava com a
  geometria de Euclides, nao com a Geometria que ela estava
  descobrindo. Entretanto, com o passar do tempo, ficamos sabendo que
  a realidade e muito provavelmente NAO-EUCLIDIANA mais provavel que
  a realidade se
 
   Jorge Paulino wrote:
Provavelmente esse tópico já foi criado em algum
   momento. Mesmo assim, como sou novo por aqui, gostaria de alguma
   contribuição.
  
   Sem recorrer à função gama, usando como recurso
   apenas a interpretação através da problemas de contagem, como
   justificar
  que
   0!=1??
  
   Eu conheço apenas a interpretação vinculada ao
   número de subconjuntos. Como Cn,p é igual ao número de
   subconjuntos de p elementos de um conjunto de n elementos,  então
   Cn,0 = 1 indica o número
  de subconjuntos de 0 elementos, a saber, o
   vazio.
  
   Porém, se C8,3 indica o número de comissões
   de 3 pessoas num grupo de 8, como aceitar que o número de
   comissões de
  zero
   pessoas é igual C8,0=1?
  
   Se A5,3 fornece o número de senhas de 3 letras
   distintas a partir de um universo de 5, como aceitar que deste
   mesmo
  universo é
   possível obter uma senha de zero letras, isto é, A5,0 = 1?
  
Grato,
Jorge
 
  =
  Instruções para entrar na lista, sair da lista e usar a lista em
  http://www.mat.puc-rio.br/~obmlistas/obm-l.htmlhttp://www.mat.puc-rio.br/%7Eobmlistas/obm-l.html
  

Re: [obm-l] Soma !!!

2008-04-09 Por tôpico Rodrigo Renji
Um metodo que eu conheço pra fazer esses somatorios é o seguinte

vou escrever o somatorio de f(k) com k variando de a até b como
(com a e b inteiros, b=a)
soma [k=a,b] f(k)

seja D o operador que faz Df(k)=f(k+1)-f(k)   [ normalmente escrevo o
D como o simbolo delta mas com  aqui nao tem opção escrevo D mesmo]
temos que
soma [k=a,b] Df(k)= f(b+1)-f(a) conhecida como soma telescópica

com isso podemos fazer o seguinte  , definino o somatorio indefinido
soma f(k) =g(k) se e somente se Dg(k)=f(k), que é util pois se voce
sabe uma função cujo D aplicado de f(k) voce resolve o somatorio ,
pois se Dg(k)=f(k) temos
soma [k=a,b] f(k)= soma [k=a,b] Dg(k) =g(b+1)-g(a) por soma telescopica
e se voce tem a soma indefinida
soma f(k) =g(k)  voce pode aplicar os limites do somatorio depois, ficando assim
se soma f(k) =g(k)  então soma[k=a,b] f(k) =g(b+1) -g(a)

entao a principio vou trabalhar os somatorios sem limites superior e
inferior achando a primitiva finita  (x) deles , começando com
somatorio de termos do tipo a^k, com a fixo e k variando, a soma de
termos da p.g

começo aplicando D em a^k, Da^k =a^(k+1)-a^(k) =a.a^k -a^k =a^k(a-1)
logo Da^k=(a-1)a^(k)
aplicando o somatorio de ambos lados temos
soma Da^k= soma (a-1)a^(k)
mas como soma Da^k=a^k, temos
a^k= soma (a-1)a^(k), se a diferente de 1, podemos dividir ambos lados
por (a-1) ficando

soma a^k = (a^k)/(a-1), onde voce aplica os limites inteiros que
quiser depois *(sempre que eu falar isso é com a condição que o limite
superior seja inteiro maior ou igual ao limite inferior (caso
contrario defina o somatorio como somatorio sobre conjunto vazio sendo
zero))

agora pra calcular soma k.a^k, eu costumo usar a técnica de soma por
partes (analogo a integração por partes) que segue do seguinte

D[g(k).f(k)] =g(k+1).f(k+1)-g(k).f(k)

somando e subtraindo f(k+1).g(k) temos

D[g(k).f(k)] =g(k+1).f(k+1)-f(k+1).g(k)+f(k+1).g(k)-g(k).f(k)

colocando f(k+1) em evidencia no primeiros 2 termos e g(k) em
evidencia nos dois segundos temos

D[g(k).f(k)] = f(k+1)[g(k+1)-g(k)] +  g(k) [f(k+1)-f(k) ]
vendo que aparece g(k+1)-g(k) =Dg(k) e  f(k+1)-f(k)  =Df(k) escrevemos

D[g(k).f(k)] = f(k+1)[Dg(k)] +  g(k) [Df(k) ] que é a formula analoga
a derivação de produto (caso finito)

aplicando o somatorio de ambos lados temos
soma D[g(k).f(k)] = soma f(k+1)[Dg(k)] +  g(k) [Df(k) ]  e pela
linearidade do somatorio e pelo telescopico

g(k).f(k)= soma  f(k+1)[Dg(k)]  +soma g(k) [Df(k)]
assim
soma  g(k) [Df(k)] = g(k)f(k) -  soma  f(k+1)[Dg(k)]

e vou usar isso pra calcular soma k.a^k

 soma k.a^k, vou tomar g(k)= k entao Dg(k)=g(k+1)-g(k)=x+1-x=1
e tomar Df(k)=a^k então f(k)= a^(k)/ (a-1)
com isso temos pela formula

soma k.a^k = k.a^(k)/(a-1) - soma a^(k+1)/(a-1)  =
=k.a^(k)/(a-1) - a/(a-1)soma a^(k), e como sabemos que  soma
a^(k)=a^(k)/(a-1) [ que foi feito antes), temos que

soma k.a^k =k.a^(k)/(a-1) - a.a^(k)/(a-1)^2

onde voce aplica os limites depois, sendo g(k)=k.a^(k)/(a-1) - a.a^(k)/(a-1)^2
voce tem o somatorio com os limites
soma[k=c,b] k.a^k =g(b+1)-g(c)

usando isso pra resolver o problema temos

soma [k=1,n]  [k.10^k -k] = soma [k=1,n] k.10^k -soma [k=1,n] k =
=soma [k=1,n] k.10^k -(n)(n+1)/2  no primeiro somatorio usando a
formula acima  ficamos com
soma [k=1,n]  [k.10^k -k] =10^(n+1)[9n-1]/81 +10/81 -n(n+1)/2

e finalmente multiplicando por 1/9 que era a constante que ficava pra
fora do somatorio temos
1/9 [10^(n+1)[9n-1]/81 +10/81 -n(n+1)/2]

os metodos que usei acima podem ser usado pra resolver outros
somatorios, como por exemplo c(k,p)a^k onde c(k,p) é o coeficiente
binomial e k varia no somatorio,  somatorio de seno e cosseno, esses
metodos + numeros de stirling do segundo tipo pra resolver somatorio
de potencias (base variando) (k^p, k variando, p natural), hum...







Em 09/04/08, Pedro Júnior[EMAIL PROTECTED] escreveu:
 Eita mundão da matemática...

 Rapaz 1ª vez que vi esta fórmula, nossa, mas faz sentido claro...

 vou verificar valeu mesmo, só uma perguntinha, onde vc encontrou essa
 questão mesmo?

 pois encontrei numa lista de exercício por aí, e coloquei na minha porém não
 havia resolvido antes.

 resultado nome da questão: UM PROFESSOR EM APUROS!!!

 KKK

 Bom, agradeço bastante a colaboração e vou apicar indução afim de verificar
 se vale para todo n.

 abraços

 E a caminhada continua!

 2001/11/1 Pedro [EMAIL PROTECTED]:

 
 
  Essa questão deu muito trabalho à tres semana, mais no fim deu certo.
 
Seja S_n = 1.11^0 + 2.11^1 +3.11^2 +...+n.111 rescrever
 de uma maneira para facilitar a solução:
 
   S_n = 1.(10^1 - 1)/9 +2.(10^2 - 1)/9 ++n.(10^n - 1)/9
 
 S_n = 1/9.[ (1.10^1 +2.10^2+...+n.10^n) - (1+2+3+...+n)]
 
Esta parte que eatá em negrito é : Série aritmético - geométrica. Você
 aplica a sguinte fórmula:
 
S_n=[ a_1(1 - q^n)/1- q]   + rq[1 - nq^(n - 1) +(n - 1).q^n]/(1
 - q)^2
 
  obs:a_0=0 , a_1=1 e q=10
 
Portanto,
 
S_n= 1/9 {10/81( 

Re: [obm-l] RES: [obm-l] RES: [obm-l] Indicação de livros (Cálculo numérico e Equações diferenciais)

2008-03-26 Por tôpico Rodrigo Renji
De equações diferenciais(livros) eu conheço poucos, mas dos poucos que
conheço um que me agradou foi o livro, William E. Boyce e Richard C.
Diprima, Equações diferenciais elementares e problemas de valores de
contorno, um de cálculo númerico que me agradou foi Cálculo númerico
de Victor Mirshawka

Em 26/03/08, Paulo - Uniredes[EMAIL PROTECTED] escreveu:


 Olha João,

 Provavelmente tem uns amigos aí chateados comigo, pois eu ainda não estava
 preparado para os pedidos que iam chegar. Meu pequeno sebo tem muitos livros
 legais mas eu precisava de uma loja eletrônica, para organizar as coisas.
 Ela está em http://uniredes.org/loja01. É ainda incipiente mas vai ajudar
 muito. Estou migrando o cadastro das obras que tenho. Não é um esquema
 rigorosamente comercial. Quando acabar o estoque particular acabou, mas eu
 quero atender direitinho, deixando tudo documentado. Assim posso indicar no
 ano que vem quem tem o que para vender. Será muito legal e um serviço de
 utilidade.

 Os preços que você deseja estarão disponíveis na loja, amanhã de manhã,
 prometo.

 Espero que todos gostem e já peço desculpas pela demora.



 ---
 Paulo C. Santos (PC)
 e-mail: [EMAIL PROTECTED]
 Homepage: http://uniredes.org
 Tel.: (21) 2510.8783 - Cel.: (21) 8753-0729
 
 MS-Messenger: [EMAIL PROTECTED]

  

 De: [EMAIL PROTECTED] [mailto:[EMAIL PROTECTED] Em nome
 de Joao Victor Brasil
 Enviada em: quarta-feira, 26 de março de 2008 08:31
 Para: obm-l@mat.puc-rio.br
 Assunto: Re: [obm-l] RES: [obm-l] Indicação de livros (Cálculo numérico e
 Equações diferenciais)



 Paulo,

 Quanto você quer neste livros? Os do Piskonov, Apostol e Pacitti.

 Joao Victor


 On 3/26/08, Paulo - Uniredes [EMAIL PROTECTED] wrote:
 
 
  Oi Cesar,
 
  Para cálculo I e II, eu recomendo o Piskunov:
 http://www.traca.com.br/seboslivrosusados.cgi?mod=LV62211origem=resultadodetalhada
  Eu tenho os volumes I e II em espanhol no meu sebo. São muito legais. O
 volume II é ótimo em equações diferenciais. A obra, em 2 volumes, foi usada
 na UFRJ.
 
  Tem também o do Tom M. Apostol em português...:
 
 
 http://www.livrariadafisica.com.br/produto_detalhe.asp?id_produto=19209
 
  Estudei pelo Apostol e a qualidade é muito boa. Apesar de eu ser meio
 burro (Só passei em cálculo I após fazer um curso de verão na UFRJ. Muito
 legal...mas meu negócio é redes de computadores).
 
  Faço preço camarada nos 3 livros. Camarada mesmo !
 
 
  []s
 
 
 
  ---
  Paulo C. Santos (PC)
  e-mail: [EMAIL PROTECTED]
  Homepage: http://uniredes.org
  Tel.: (21) 2510.8783 - Cel.: (21) 8753-0729
  
  MS-Messenger: [EMAIL PROTECTED]
 
  
  De: [EMAIL PROTECTED] [mailto:[EMAIL PROTECTED] Em nome
 de César Santos
  Enviada em: terça-feira, 25 de março de 2008 16:10
  Para: obm-l@mat.puc-rio.br
  Assunto: [obm-l] Indicação de livros (Cálculo numérico e Equações
 diferenciais)
 
 
  Poderiam me indicar bons livros de cálculo numérico e de equações
 diferencias acessíveis para iniciante?
 
  
  Abra sua conta no Yahoo! Mail, o único sem limite de espaço para
 armazenamento!
 
 
 
 
 
 



=
Instruções para entrar na lista, sair da lista e usar a lista em
http://www.mat.puc-rio.br/~obmlistas/obm-l.html
=


Re: [obm-l] Fw: [obm-l] soma de série

2008-03-11 Por tôpico Rodrigo Renji
mostrar que 1+1/2+1/3+...+1/n não é inteiro pra qquer N1.

a questão original é uma soma finita e não uma série
a série sabemos que diverge, o que se queria  na demonstração e que a
soma finita acima
nunca é inteira
soma [k=1, n] 1/k

Em 11/03/08, saulo nilson[EMAIL PROTECTED] escreveu:
 1+1/2+1/3+1/4=(24+12+8+6)/24=par /par


 2008/3/11 Luiz Alberto Duran Salomão [EMAIL PROTECTED]:


 
 
 
 
  - Original Message -
  From: Luiz Alberto Duran Salomão
  To: obm-l@mat.puc-rio.br
  Sent: Tuesday, March 11, 2008 10:21 AM
  Subject: Re: [obm-l] soma de série
 
 
  Caros amigos,
  Seja n um inteiro, com n1. O que se quer provar é que
  1+1/2+1/3+ . . . +1/n   não é inteiro.
  Seja 2^a a maior potência de 2 tal que 2^a é menor do que ou igual a n.
  Assim, 1/2^a   aparece no somatório acima mas 1/2^(a+1)  não aparece.
  Observe que o mínimo múltiplo comum dos denominadores dos
  termos do somatório tem a potência 2^a como fator. Agora, no
  numerador de cada  fração, já com denominador igual ao mínimo
  múltiplo comum, temos sempre um número par, com exceção do
  termo correspondente a 1/2^a. Logo, a soma dos numeradores
  é ímpar o que nos leva a concluir que o somatório não é  um
  número inteiro.
  Abraços,
  Luiz Alberto
 
 
  - Original Message -
  From: MauZ
  To: obm-l@mat.puc-rio.br
  Sent: Monday, March 10, 2008 6:13 PM
 
  Subject: [obm-l] soma de série
 
  mostrar que 1+1/2+1/3+...+1/n não é inteiro pra qquer N1.
 
  Obrigado!
 
 
 
  

 
  No virus found in this incoming message.
  Checked by AVG.
  Version: 7.5.518 / Virus Database: 269.21.7/1324 - Release Date: 10/3/2008
 19:27
 
 
 
 
 



=
Instruções para entrar na lista, sair da lista e usar a lista em
http://www.mat.puc-rio.br/~obmlistas/obm-l.html
=


Re: [obm-l] RES: [obm-l] RES: [obm-l] [obm-l] Questão de indução matemática

2008-03-05 Por tôpico Rodrigo Renji
agora me lembrei de outra coisa tb... tem um tempo eu fiz uma página
de internet com propriedades básicas de somatório (bem básicas mesmo!!
hehehe)  o link abaixo  se quiser ver
http://iishp.5gbfree.com/matematica/soma/somas.html

depois vou escrever mais (incluindo aplicação nos problema desse
email) e colocar em outra página

abraços o/

Em 05/03/08, Rubens Kamimura[EMAIL PROTECTED] escreveu:
 Renji,

  1. grato pelo retorno, valeu.

  Sds

  Rubens

  -Mensagem original-

 De: [EMAIL PROTECTED] [mailto:[EMAIL PROTECTED] Em nome

 de Rodrigo Renji
  Enviada em: terça-feira, 4 de março de 2008 18:03

 Para: obm-l@mat.puc-rio.br

 Assunto: Re: [obm-l] RES: [obm-l] [obm-l] Questão de indução matemática


  bem saiu um simbolo errado no texto que eu escrevi
  ao inves de

  soma [k=1, n+1] f(k)=soma [k=1, n] f(k)= +f(n)
  com
  soma [k=1, 1] f(k)= f(1)


  é


  soma [k=1, n+1] f(k)=soma [k=1, n] f(k) +f(n)
  com
  soma [k=1, 1] f(k)= f(1)

  apareceu um = a mais na primeira que não era pra ter

  eu costumo usar sempre essa definição acima, para demonstrar problemas
  de somatórios
  acho a notação de pontinhos (a1+...+an) informal, tento não usar
  apesar de dar uma visão as vezes do que esta acontecendo com o
  somatorio, mas acho que se pode desenvolver as técnicas de somatorio o
  suficiente para nao precisar abrir em pontinhos =P (abrindo as vezes,
  o primeiro, ou ultimo termo apenas nas demonstrações)

  se precisar de mais alguma ajuda só postar
  abraços o/
  Em 04/03/08, Rubens Kamimura[EMAIL PROTECTED] escreveu:
  
  
  
  
   Olá Marcelo Salhab,
  
  
  
   Muito grato.
  
  
  
   Sds
  
  
  
   Rubens Kamimura
  
   Assistente Técnico III - CREA/SP 5062246285
  
   CESP - Companhia Energética de São Paulo
  
   OMPTD - Capacitação e Desenvolvimento
  
   Caixa Postal, 58 - CEP 15385-000
  
   Ilha Solteira/SP - Brasil
  
   Tel. +55-18-3704-4240 ramal 136/137
  
   Tel./Fax +55-18-3704-6800
  
   www.cesp.com.br
  
   email: [EMAIL PROTECTED]
  
   Mens In Corpore Tantun Molen Regit
  
   UNYK : 132 XOU
  
   P Antes de imprimir pense em sua responsabilidade e compromisso com o MEIO
   AMBIENTE.
  
  
  
  
  
  
  
  
   De: [EMAIL PROTECTED] [mailto:[EMAIL PROTECTED] Em nome
   de Marcelo Salhab Brogliato
Enviada em: terça-feira, 4 de março de 2008 14:29
Para: obm-l@mat.puc-rio.br
Assunto: Re: [obm-l] [obm-l] Questão de indução matemática
  
  
  
  
   Olá Rubens,
  
Essas demonstrações seguem todas a mesma idéia...
vou fazer apenas o 2.1
veja que para n=1 é válido
  
vamos supor valido para K e vamos mostrar que vale para K+1.
isto é:
Hipótese: 1^2 + 2^2 + ... + k^2 = [k(k+1)(2k+1)]/6
Tese: 1^2 + 2^2 + ... + k^2 + (k+1)^2 = (k+1)(k+2)(2k+3)/6
  
Demo:
Sabemos que: 1^2 + 2^2 + .. + k^2 = k(k+1)(2k+1)/6
somando (k+1)^2 em ambos os lados, temos:
1^2 + 2^2 + ... + k^2 + (k+1)^2 = k(k+1)(2k+1)/6 + (k+1)^2
  
vamos apenas fatorar o lado direito, e mostrar que ele é igual a
   (k+1)(k+2)(2k+3)/6 = (k+1)(2k^2 + 7k + 6)/6
k(k+1)(2k+1)/6 + (k+1)^2 = (k+1)[ k(2k+1)/6 + (k+1) ] = (k+1)[ 2k^2 + k +
   6k + 6 ]/6 = (k+1)(2k^2 + 7k + 6)/6
  
logo, está provado por indução.
  
abraços,
Salhab
  
  
  
  
  
   2008/3/3 Rubens Kamimura [EMAIL PROTECTED]:
  
   Olá turma da LISTA!!!
  
Alguém desta LISTA, se habilitariam em me elucidar tal questão?
  
1. Sabendo, por definição, que: a^0=1 e a^1=a, como poderemos provar por
indução matemática sobre n, que a^m.a^n = a^(m+n), para qualquer m,n
pertencente ao conjunto dos números naturais?
  
2. Como podemos provar por indução matemática:
2.1. 1^2+ 2^2+...+n^2 = [n(n+1)(2n+1)]/6, (n maior igual 1);
2.2. 1^3+ 2^3+...+n^3 = (1+2+...+n)^2, (n maior igual 1);
2.3. 1.2+2.3+...+n(n+1) = [n(n+1)(n+2)]/3, (n maior igual 1);
  
Abraços
  
leigo e neófito...
  
  
   =
Instruções para entrar na lista, sair da lista e usar a lista em
http://www.mat.puc-rio.br/~obmlistas/obm-l.html
   =
  
  
  
--
Esta mensagem foi verificada pelo sistema de antivírus e
acredita-se estar livre de perigo.

  =
  Instruções para entrar na lista, sair da lista e usar a lista em
  http://www.mat.puc-rio.br/~obmlistas/obm-l.html
  =

  --

 This message has been scanned for viruses and
  dangerous content by MailScanner, and is
  believed to be clean.



  =
  Instruções para entrar na lista, sair da lista e usar a lista em
  http://www.mat.puc-rio.br/~obmlistas/obm-l.html

Re: [obm-l] [obm-l] Questão de indução matemática

2008-03-03 Por tôpico Rodrigo Renji
oi, vou tentar te ajudar com esses problemas




vou usar uma notação simplificada, a de somatorio, (temo que isso
dificulte sua leitura =/)
vou escrever o somatorio como
soma [k=1, n] f(k) que é o mesmo que informalmente a

soma [k=1, n] f(k)= f(1)+f(2)++f(n-1)+f(n)
que definido por recorrencia por

soma [k=1, n+1] f(k)=soma [k=1, n] f(k)= +f(n)
com
soma [k=1, 1] f(k)= f(1)
e
soma [k=1, n] f(k)= 0 se  n1, isto é, se o limite superior é menor
que o inferior
mas o que voce precisa saber realmente é que

soma [k=1, n+1] f(k)=soma [k=1, n] f(k)= +f(n)
com
soma [k=1, 1] f(k)= f(1)

soma [k=1, n] f(k)= f(1)+f(2)++f(n-1)+f(n)

isto é, que esse simbolo
soma [k=1, n] f(k)= equivale a toma a soma de f(k), com k variando de 1 até n

f(1)+f(2)++f(n-1)+f(n)

o que vai ser usado nas demonstrações vai ser isso
soma [k=1, n+1] f(k)=soma [k=1, n] f(k)= +f(n).
(eu considero mais formal e compacto usar a notação de somatorio)
agora vamos pra primeira

2.1. 1^2+ 2^2+...+n^2 = [n(n+1)(2n+1)]/6, (n maior igual 1);

com a notação de somatorio isso se escreve
soma [k=1, n] k^2=[n(n+1)(2n+1)]/6

para n=1 temos
soma [k=1, n] k^2= 1^2=1
e no outro lado temos
[1(1+1)(2+1)]/6= 1(2)(3)/6=1, entao a base da indução esta provada

agora vamos tomar por hipotese de que
soma [k=1, n] k^2=[n(n+1)(2n+1)]/6

e provar para (n+1)

soma [k=1, n+1] k^2=[(n+1)(n+2)(2n+3)]/6
mas primeiro vamos expandir  [(n+1)(n+2)(2n+3)]/6, pois provavelmente nao vamos
chegar na forma fatorada bunitinha como esta acima
 [(n+1)(n+2)(2n+3)]/6=1+13n/6 +3n^2 /2 +n^3/3

vamos expandir tb [n(n+1)(2n+1)]/6
[n(n+1)(2n+1)]/6= n/6 +n^2/2 +n^3/3

agora vamos pra demonstração
soma [k=1, n+1] k^2=soma [k=1, n] k^2 + (n+1)^2 (pela definição de
somatorio), mas pela hipotese temos
soma [k=1, n] k^2 =[n(n+1)(2n+1)]/6 que expandido é = n/6 +n^2/2 +n^3/3
somando com (n+1)^2 = n^2 +2n+1, ficamos com
n/6 +n^2/2 +n^3/3 + n^2 +2n+1=n^3/3+ 3n^2/2 +13n/6 +1= [(n+1)(n+2)(2n+3)]/6

=x, tenta fazer o resto se nao conseguir ou nao entender algo, tenta responder

o/
2.2. 1^3+ 2^3+...+n^3 = (1+2+...+n)^2, (n maior igual 1);
2.3. 1.2+2.3+...+n(n+1) = [n(n+1)(n+2)]/3, (n maior igual 1);

Em 03/03/08, Rubens Kamimura[EMAIL PROTECTED] escreveu:
 Olá turma da LISTA!!!

  Alguém desta LISTA, se habilitariam em me elucidar tal questão?

  1. Sabendo, por definição, que: a^0=1 e a^1=a, como poderemos provar por
  indução matemática sobre n, que a^m.a^n = a^(m+n), para qualquer m,n
  pertencente ao conjunto dos números naturais?

  2. Como podemos provar por indução matemática:
  2.1. 1^2+ 2^2+...+n^2 = [n(n+1)(2n+1)]/6, (n maior igual 1);
  2.2. 1^3+ 2^3+...+n^3 = (1+2+...+n)^2, (n maior igual 1);
  2.3. 1.2+2.3+...+n(n+1) = [n(n+1)(n+2)]/3, (n maior igual 1);

  Abraços

  leigo e neófito...


  =
  Instruções para entrar na lista, sair da lista e usar a lista em
  http://www.mat.puc-rio.br/~obmlistas/obm-l.html
  =


=
Instruções para entrar na lista, sair da lista e usar a lista em
http://www.mat.puc-rio.br/~obmlistas/obm-l.html
=


Re: [obm-l] Diferença finita ( de novo)

2008-02-29 Por tôpico Rodrigo Renji
agora sobre o somatório dessas p.as de outras ordens,elas saem fácil
sabendo a propriedade de somatorio de coeficiente binomial

somatorio [x=0 até b] de c(x+c, k) = c(b+c+1, k+1)

dai temos
somatorio [x=0 até n] de c(x-1, k) = c( n-1+1, k+1) =c( n,k+1)

sobre a sequencia {  3, 0, 5, 34 , 135, 452}
já tinham colocado em outro email que, apenas os primeiros termos não
definem uma sequencia, pois existem infinitas fórmulas que os geram,
que tem esses termos em comum, porém uma formula simples pra esses
termos é f(n)=2.3^(n)   -7.n   +1, a partir de f(0), ai o somatorio
nao sairia pelo metodo do seu email, mas sim o somatorio de termos de
uma p.g 3^n,  e de uma p.a -7n +1,

o/

Em 28/02/08, Rodrigo Renji[EMAIL PROTECTED] escreveu:
 agora sobre dua dúvida,, sobre o operador E (de expansão?)
  o operador E, quando aplicado numa função f(x), faz ela ser deslocada,
  sendo tomada f(x+1)
  isto é a definição dele é Ef(x)= f(x+1)
  as potencias maiores, podem ser definidas
  E^k f(x)= f(x+k), k pode ser qualquer real, mas no cálculo de
  diferenças finitas estamos apenas interessados quando k é inteiro.
  exemplos

  E 2^(x) = 2^(x+1)

  Esen(x) = sen(x+1)

  ai temos o operador delta que vou escrever como D, ele se definine como
  Df(x)= f(x+1)-f(x), mas sabemos que f(x+1)= Ef(x), assim escrevemos
  Df(x)= Ef(x)-f(x), como seria interessante colocar f(x) em evidencia em
  Ef(x)-f(x), definimos a soma de dois operadores (A+B)f(x)=Af(x)+Bf(x)
  (e produto pode ser definido tb, bla bla bla, tem que demonstrar um
  monte de coisas nisso)
  ai tomamos
  Df(x)=(E-1) f(x) e dizemos que o operador D=E-1, pois para toda função
  em que são aplicadas eles fazem a mesma coisa, tomar f(x+1)-f(x).
  sobre os metodos, todos eles são da teoria de diferenças finitas.
  abraços o/

  Em 28/02/08, Rodrigo Renji[EMAIL PROTECTED] escreveu:

  ah sim uma dedução que pode ser feita pra deduzir a formula das p.a de
ordem superior
para as progressões aritmeticas podemos escrever
  
an=a1+ (n-1).r = c(n-1,0) a1 + c(n-1,1) r
  
vamos deduzir agora a formula da p.a de ordem 2, montando o seguinte 
 esquema
o primeiro termo da p.a de ordem 2, vou chamar de b1, o n-esimo termo
vou chamar de bn
então podemos montar o esquema
b1b2--b3b4
b1-b1+a1---b1+2a1+r---b1+3a1 +3r
-a1--a1+r---a1+2r
rr
  
temos entao
b1=b1
b2=b1+a1
b3=b1+2a1+r
b4=b1+3a1+3r
dai percebemos que nesse casos, aparece sempre b1, com coeficiente 1
entao poderiamos deduzir
bn =b1
analisando os coeficientes de a1, nos bn, temos
b1=0a1
b2=1a1
b3=2a1
b4=3a1
perceba que nos casos tomados, bn, o coeficiente de a1, é sempre (n-1)
então ate agora temos bn =b1+ (n-1)a1
agora vamos deduzir o coeficiente de r
b1=0r
b2=0r
b3=1r
b4=3r
observe que para n=1 e e n=2, os coeficientes de r são zero,
poderiamos escrever entao
(n-1)(n-2).k como coeficiente de r, só que tem que ser válido quando
n=3, se n=3, temos
o coeficiente igual a 1, logo (3-1)(3-2).k=1 2.1.k=1, logo k=1/2
temos então o coeficiente de r igual a (n)(n-1)/2
a forma fica então
  
bn=b1+(n-1)a1+(n-1)(n-2).r/2
escreva agora essa expressão com coeficiente binomial
bn=c(n-1,0)b1+c(n-1,1)a1+c(n-1,2)r
e compare com a p.a escrita com coeficiente binomial
an=c(n-1,0)a1+c(n-1,1)r
  
as duas juntas
bn=c(n-1,0)b1+c(n-1,1)a1+c(n-1,2)r
an=c(n-1,0)a1+c(n-1,1)r
olhando esse padrão ja daria pra deduzir a formula da p.a de terceira 
 ordem
  
cn=c(n-1,0)c1+c(n-1,1)b1+c(n-1,2)a1 +c(n-1,3)r
onde c1, seria a o primeiro termo da p.a de ordem 3
  
e a de quarta ordem
dn=c(n-1,0)d1+c(n-1,1)c1+c(n-1,2)b1 +c(n-1,3)a1 +c(n-1,4)r
  
Em 25/02/08, Rodrigo Renji[EMAIL PROTECTED] escreveu:
  
acho que nao mandei o email inteiro da outra vez, aqui vai completo:


  Pedro vou tentar explicar melhor o que eu sei de p.a's de outras
  ordens (seguindo uma maneira informal e tentando deduzir a formula
  geral)

  primeiro as notações que vou usar, o coeficiente binomial  n!/k!(n-k)
  vou escrever como c(n,k)

  vou começar fazendo o seguinte, analisando um polinomio do primeiro
  grau, por exemplo...
  y=2x+1. tomando valores, de x variando de 0, depois 1, depois 2, vamos
  ver o que acontece

  x=0 y=2.0+1 =1
  x=1 y=2.1+1=3
  x=2 y=2.2+1=5
  x=3 y=2.3+1=7

  vamos alinhar os valores de y em sequencia
  1-3---5-7, tirando as diferenças
  ---2--2---2, ela é sempre constante igual a 2, então temos uma 
 p.a

  mas o que aconteceria se tomassemos diferenças em um polinomio de grau
  2? vamos ver o que acontece?, vamos tomar um polinomio simples de grau
  2, por exemplo y=x^2
  e tomar valores partindo de zero, e pulando de 1 em 1, vamos lá

Re: [obm-l] Diferença finita ( de novo)

2008-02-28 Por tôpico Rodrigo Renji
ah sim uma dedução que pode ser feita pra deduzir a formula das p.a de
ordem superior
para as progressões aritmeticas podemos escrever

an=a1+ (n-1).r = c(n-1,0) a1 + c(n-1,1) r

vamos deduzir agora a formula da p.a de ordem 2, montando o seguinte esquema
o primeiro termo da p.a de ordem 2, vou chamar de b1, o n-esimo termo
vou chamar de bn
então podemos montar o esquema
b1b2--b3b4
b1-b1+a1---b1+2a1+r---b1+3a1 +3r
-a1--a1+r---a1+2r
rr

temos entao
b1=b1
b2=b1+a1
b3=b1+2a1+r
b4=b1+3a1+3r
dai percebemos que nesse casos, aparece sempre b1, com coeficiente 1
entao poderiamos deduzir
bn =b1
analisando os coeficientes de a1, nos bn, temos
b1=0a1
b2=1a1
b3=2a1
b4=3a1
perceba que nos casos tomados, bn, o coeficiente de a1, é sempre (n-1)
então ate agora temos bn =b1+ (n-1)a1
agora vamos deduzir o coeficiente de r
b1=0r
b2=0r
b3=1r
b4=3r
observe que para n=1 e e n=2, os coeficientes de r são zero,
poderiamos escrever entao
(n-1)(n-2).k como coeficiente de r, só que tem que ser válido quando
n=3, se n=3, temos
o coeficiente igual a 1, logo (3-1)(3-2).k=1 2.1.k=1, logo k=1/2
temos então o coeficiente de r igual a (n)(n-1)/2
a forma fica então

bn=b1+(n-1)a1+(n-1)(n-2).r/2
escreva agora essa expressão com coeficiente binomial
bn=c(n-1,0)b1+c(n-1,1)a1+c(n-1,2)r
e compare com a p.a escrita com coeficiente binomial
an=c(n-1,0)a1+c(n-1,1)r

as duas juntas
bn=c(n-1,0)b1+c(n-1,1)a1+c(n-1,2)r
an=c(n-1,0)a1+c(n-1,1)r
olhando esse padrão ja daria pra deduzir a formula da p.a de terceira ordem

cn=c(n-1,0)c1+c(n-1,1)b1+c(n-1,2)a1 +c(n-1,3)r
onde c1, seria a o primeiro termo da p.a de ordem 3

e a de quarta ordem
dn=c(n-1,0)d1+c(n-1,1)c1+c(n-1,2)b1 +c(n-1,3)a1 +c(n-1,4)r

Em 25/02/08, Rodrigo Renji[EMAIL PROTECTED] escreveu:
 acho que nao mandei o email inteiro da outra vez, aqui vai completo:


  Pedro vou tentar explicar melhor o que eu sei de p.a's de outras
  ordens (seguindo uma maneira informal e tentando deduzir a formula
  geral)

  primeiro as notações que vou usar, o coeficiente binomial  n!/k!(n-k)
  vou escrever como c(n,k)

  vou começar fazendo o seguinte, analisando um polinomio do primeiro
  grau, por exemplo...
  y=2x+1. tomando valores, de x variando de 0, depois 1, depois 2, vamos
  ver o que acontece

  x=0 y=2.0+1 =1
  x=1 y=2.1+1=3
  x=2 y=2.2+1=5
  x=3 y=2.3+1=7

  vamos alinhar os valores de y em sequencia
  1-3---5-7, tirando as diferenças
  ---2--2---2, ela é sempre constante igual a 2, então temos uma p.a

  mas o que aconteceria se tomassemos diferenças em um polinomio de grau
  2? vamos ver o que acontece?, vamos tomar um polinomio simples de grau
  2, por exemplo y=x^2
  e tomar valores partindo de zero, e pulando de 1 em 1, vamos lá

  x=0, y=0
  x=1, y=1
  x=2, y=4
  x=3, y=9
  x=4, y=16
  x=5, y=25
  vamos tomar entao os resultados y, em sequencia
   0-14---916-25
  e se tiramos as diferenças, o que acontece?, vamos lá
   0-14916-25
  1-35---7--9
  aparece nas diferenças, uma sequencia que não é constante, vamos então
  tirar a diferença dessa sequencia que apareceu (diferença das
  diferenças) (observe que as diferenças são os termos da primeira
  sequencia considerada, a que surgiu de y=2x+1)
  tomando as diferenças temos entao
  1-35---7--9
  --2--22---2
  uma constante 2, vimos então que, tomando um polinomio de grau 2 (o
  y=x^2), e tomando as diferenças, temos uma sequencias onde a diferença
  é uma p.a e a segunda diferença é constante, e o que aconteceria com
  um polinomio de grau 3? (vou fazer só mais esse caso)

  exemplo y=x^3, tomando valores, começando do zero, temos
  x=0 y=0
  x=1, y=1
  x=2, y=8
  x=3, y=27
  x=4 , y=64
  x=5, y=125
  pondo em ordem e tirando as diferenças temos

  0---18-27-64125
  ---1---719-37--61
  --6---1218--24
  -6-6--6
  a primeira diferença nao é constante, a segunda diferença não é
  constante, porém a terceira diferença é constante

  com isso podemos perceber algumas coisas, como, nos casos analisados,
  a n diferença de um polinomio de grau n é constante, e como a
  diferença de constante é zero, temos a n+1 diferença de um polinomio
  de grau n é zero.
  dos exemplos, diferença de termos no polinomio de grau 1, 2x+1 ´e constante,
  a 2 segunda diferença dos termos de um polinomio de grau 2 é constante
  (no caso testado x^2)
  a terceira diferença de um polinomio de grau 3 é constante( do caso x^3)

  mas como definir essas sequencias?
  a sequencia cuja segunda diferença é constante, é uma p.a de ordem 2,
  a sequencia cuja terceira diferença é constante é uma p.a de ordem 3,
  a sequencia onde a n esima diferença é constante, é uma p.a de ordem n
  ( sendo as constantes diferentes de zero)

  no proximo email uma dedução das primeiras formulas de p.a e
  extrapolação pra todos outros casos

Re: [obm-l] Diferença finita ( de novo)

2008-02-28 Por tôpico Rodrigo Renji
agora sobre dua dúvida,, sobre o operador E (de expansão?)
o operador E, quando aplicado numa função f(x), faz ela ser deslocada,
sendo tomada f(x+1)
isto é a definição dele é Ef(x)= f(x+1)
as potencias maiores, podem ser definidas
E^k f(x)= f(x+k), k pode ser qualquer real, mas no cálculo de
diferenças finitas estamos apenas interessados quando k é inteiro.
exemplos

E 2^(x) = 2^(x+1)

Esen(x) = sen(x+1)

ai temos o operador delta que vou escrever como D, ele se definine como
Df(x)= f(x+1)-f(x), mas sabemos que f(x+1)= Ef(x), assim escrevemos
Df(x)= Ef(x)-f(x), como seria interessante colocar f(x) em evidencia em
Ef(x)-f(x), definimos a soma de dois operadores (A+B)f(x)=Af(x)+Bf(x)
(e produto pode ser definido tb, bla bla bla, tem que demonstrar um
monte de coisas nisso)
ai tomamos
Df(x)=(E-1) f(x) e dizemos que o operador D=E-1, pois para toda função
em que são aplicadas eles fazem a mesma coisa, tomar f(x+1)-f(x).
sobre os metodos, todos eles são da teoria de diferenças finitas.
abraços o/

Em 28/02/08, Rodrigo Renji[EMAIL PROTECTED] escreveu:
 ah sim uma dedução que pode ser feita pra deduzir a formula das p.a de
  ordem superior
  para as progressões aritmeticas podemos escrever

  an=a1+ (n-1).r = c(n-1,0) a1 + c(n-1,1) r

  vamos deduzir agora a formula da p.a de ordem 2, montando o seguinte esquema
  o primeiro termo da p.a de ordem 2, vou chamar de b1, o n-esimo termo
  vou chamar de bn
  então podemos montar o esquema
  b1b2--b3b4
  b1-b1+a1---b1+2a1+r---b1+3a1 +3r
  -a1--a1+r---a1+2r
  rr

  temos entao
  b1=b1
  b2=b1+a1
  b3=b1+2a1+r
  b4=b1+3a1+3r
  dai percebemos que nesse casos, aparece sempre b1, com coeficiente 1
  entao poderiamos deduzir
  bn =b1
  analisando os coeficientes de a1, nos bn, temos
  b1=0a1
  b2=1a1
  b3=2a1
  b4=3a1
  perceba que nos casos tomados, bn, o coeficiente de a1, é sempre (n-1)
  então ate agora temos bn =b1+ (n-1)a1
  agora vamos deduzir o coeficiente de r
  b1=0r
  b2=0r
  b3=1r
  b4=3r
  observe que para n=1 e e n=2, os coeficientes de r são zero,
  poderiamos escrever entao
  (n-1)(n-2).k como coeficiente de r, só que tem que ser válido quando
  n=3, se n=3, temos
  o coeficiente igual a 1, logo (3-1)(3-2).k=1 2.1.k=1, logo k=1/2
  temos então o coeficiente de r igual a (n)(n-1)/2
  a forma fica então

  bn=b1+(n-1)a1+(n-1)(n-2).r/2
  escreva agora essa expressão com coeficiente binomial
  bn=c(n-1,0)b1+c(n-1,1)a1+c(n-1,2)r
  e compare com a p.a escrita com coeficiente binomial
  an=c(n-1,0)a1+c(n-1,1)r

  as duas juntas
  bn=c(n-1,0)b1+c(n-1,1)a1+c(n-1,2)r
  an=c(n-1,0)a1+c(n-1,1)r
  olhando esse padrão ja daria pra deduzir a formula da p.a de terceira ordem

  cn=c(n-1,0)c1+c(n-1,1)b1+c(n-1,2)a1 +c(n-1,3)r
  onde c1, seria a o primeiro termo da p.a de ordem 3

  e a de quarta ordem
  dn=c(n-1,0)d1+c(n-1,1)c1+c(n-1,2)b1 +c(n-1,3)a1 +c(n-1,4)r

  Em 25/02/08, Rodrigo Renji[EMAIL PROTECTED] escreveu:

  acho que nao mandei o email inteiro da outra vez, aqui vai completo:
  
  
Pedro vou tentar explicar melhor o que eu sei de p.a's de outras
ordens (seguindo uma maneira informal e tentando deduzir a formula
geral)
  
primeiro as notações que vou usar, o coeficiente binomial  n!/k!(n-k)
vou escrever como c(n,k)
  
vou começar fazendo o seguinte, analisando um polinomio do primeiro
grau, por exemplo...
y=2x+1. tomando valores, de x variando de 0, depois 1, depois 2, vamos
ver o que acontece
  
x=0 y=2.0+1 =1
x=1 y=2.1+1=3
x=2 y=2.2+1=5
x=3 y=2.3+1=7
  
vamos alinhar os valores de y em sequencia
1-3---5-7, tirando as diferenças
---2--2---2, ela é sempre constante igual a 2, então temos uma p.a
  
mas o que aconteceria se tomassemos diferenças em um polinomio de grau
2? vamos ver o que acontece?, vamos tomar um polinomio simples de grau
2, por exemplo y=x^2
e tomar valores partindo de zero, e pulando de 1 em 1, vamos lá
  
x=0, y=0
x=1, y=1
x=2, y=4
x=3, y=9
x=4, y=16
x=5, y=25
vamos tomar entao os resultados y, em sequencia
 0-14---916-25
e se tiramos as diferenças, o que acontece?, vamos lá
 0-14916-25
1-35---7--9
aparece nas diferenças, uma sequencia que não é constante, vamos então
tirar a diferença dessa sequencia que apareceu (diferença das
diferenças) (observe que as diferenças são os termos da primeira
sequencia considerada, a que surgiu de y=2x+1)
tomando as diferenças temos entao
1-35---7--9
--2--22---2
uma constante 2, vimos então que, tomando um polinomio de grau 2 (o
y=x^2), e tomando as diferenças, temos uma sequencias onde a diferença
é uma p.a e a segunda diferença é constante, e o que aconteceria com
um polinomio de grau 3? (vou fazer só mais esse caso)
  
exemplo y=x^3, tomando valores, começando

Re: [obm-l] Diferença finita ( de novo)

2008-02-25 Por tôpico Rodrigo Renji
Pedro vou tentar explicar melhor o que eu sei de p.a's de outras
ordens (seguindo uma maneira informal e tentando deduzir a formula
geral)

primeiro as notações que vou usar, o coeficiente binomial  n!/k!(n-k)
vou escrever como c(n,k)

vou começar fazendo o seguinte, analisando um polinomio do primeiro
grau, por exemplo...
y=2x+1. tomando valores, de x variando de 0, depois 1, depois 2, vamos
ver o que acontece

x=0 y=2.0+1 =1
x=1 y=2.1+1=3
x=2 y=2.2+1=5
x=3 y=2.3+1=7

vamos alinhar os valores de y em sequencia
1-3---5-7, tirando as diferenças
---2--2---2, ela é sempre constante igual a 2, então temos uma p.a

mas o que aconteceria se tomassemos diferenças em um polinomio de grau
2? vamos ver o que acontece?, vamos tomar um polinomio simples de grau
2, por exemplo y=x^2
e tomar valores partindo de zero, e pulando de 1 em 1, vamos lá

x=0, y=0
x=1, y=1
x=2, y=4
x=3, y=9
x=4, y=16
x=5, y=25
vamos tomar entao os resultados y, em sequencia
 0-14---916-25
e se tiramos as diferenças, o que acontece?, vamos lá
 0-14916-25
1-35---7--9
aparece nas diferenças, uma sequencia que não é constante, vamos então
tirar a diferença dessa sequencia que apareceu (diferença das
diferenças) (observe que as diferenças são os termos da primeira
sequencia considerada, a que surgiu de y=2x+1)
tomando as diferenças temos entao
1-35---7--9
--2--22---2
uma constante 2, vimos então que, tomando um polinomio de grau 2 (o
y=x^2), e tomando as diferenças, temos uma sequencias onde a diferença
é uma p.a e a segunda diferença é constante, e o que aconteceria com
um polinomio de grau 3? (vou fazer só mais esse caso)

exemplo y=x^3, tomando valores, começando do zero, temos
x=0 y=0
x=1, y=1
x=2, y=8
x=3, y=27
x=4 , y=64
x=5, y=125
pondo em ordem e tirando as diferenças temos

0---18-27-64125
---1---719-37--61
--6---1218--24
-6-6--6
a primeira diferença nao é constante, a segunda diferença não é
constante, porém a terceira diferença é constante

com isso podemos perceber algumas coisas, como, nos casos analisados,
a n diferença de um polinomio de grau n é constante, e como a
diferença de constante é zero, temos a n+1 diferença de um polinomio
de grau n é zero.
dos exemplos, diferença de termos no polinomio de grau 1, 2x+1 ´e constante,
a 2 segunda diferença dos termos de um polinomio de grau 2 é constante
(no caso testado x^2)
a terceira diferença de um polinomio de grau 3 é constante( do caso x^3)

mas como definir essas sequencias?
a sequencia cuja segunda diferença é constante, é uma p.a de ordem 2,
a sequencia cuja terceira diferença é constante é uma p.a de ordem 3,
a sequencia onde a n esima diferença é constante, é uma p.a de ordem n
( sendo as constantes diferentes de zero)

no proximo email uma dedução das primeiras formulas de p.a e
extrapolação pra todos outros casos

Em 25/02/08, Luís Lopes[EMAIL PROTECTED] escreveu:

  Sauda,c~oes,

  Oi Pedro,

  Tudo isto está demonstrado no exercício 56 do
  Manual de Seq. e Séries Vol II.

  []'s
  Luís



  
  From: [EMAIL PROTECTED]
 To: obm-l@mat.puc-rio.br
 Subject: [obm-l] Diferença finita ( de novo)
 Date: Thu, 1 Nov 2001 00:23:32 -0200



 Essa sequecncia foi resolvida Pelo Professor Luís lopes (em 2003)de maneira
 brilhante, muito mais eficaz do que diferença finita.
 Pergunto ao Professor ou os demais da lista.Como demonstrar as fórmulas que
 estão em negritos a abaixo.

 1)Seja a PA de ordem 3

 1,3,19,61,141,271,... a_i

 Vamos gerar outras PAs fazendo a_{i+1} - a_i:

 2,16,42,80,130Delta a_i
 14,26,38,50 Delta^2 a_i
 12,12,12  Delta^3 a_i

 a_i = a_1 + Delta a_1 binom(i-1,1) + Delta^2 a_1
 binom(i-1,2) + Delta^3 a_1 binom(i-1,3)
 a_i = 1 + 2(i-1) + 14(i-1)(i-2)/2 + 12(i-1)(i-2)(i-3)/6
 a_i = 2i^3 - 5i^2 + 3i + 1

 S_n = a_1 binom(n,1) + Delta a_1 binom(n,2) +
 Delta^2 a_1 binom(n,3) + Delta^3 binom(n,4)

 S_n = n[3n^3 - 4n^2 - 3n + 10] / 6

 2) Posso reslver da mesma forma a seguinte questão (arrumando)

 Determine o termo geral da sequência {  3, 0, 5, 34 , 135,
 452} e calcule em seguida a soma dos n primeiros termos.

 
 Receba GRÁTIS as mensagens do Messenger no seu celular quando você estiver
 offline. Conheça o MSN Mobile! Crie já o seu!

=
Instruções para entrar na lista, sair da lista e usar a lista em
http://www.mat.puc-rio.br/~obmlistas/obm-l.html
=


Re: [obm-l] Diferença finita ( de novo)

2008-02-25 Por tôpico Rodrigo Renji
acho que nao mandei o email inteiro da outra vez, aqui vai completo:

Pedro vou tentar explicar melhor o que eu sei de p.a's de outras
ordens (seguindo uma maneira informal e tentando deduzir a formula
geral)

primeiro as notações que vou usar, o coeficiente binomial  n!/k!(n-k)
vou escrever como c(n,k)

vou começar fazendo o seguinte, analisando um polinomio do primeiro
grau, por exemplo...
y=2x+1. tomando valores, de x variando de 0, depois 1, depois 2, vamos
ver o que acontece

x=0 y=2.0+1 =1
x=1 y=2.1+1=3
x=2 y=2.2+1=5
x=3 y=2.3+1=7

vamos alinhar os valores de y em sequencia
1-3---5-7, tirando as diferenças
---2--2---2, ela é sempre constante igual a 2, então temos uma p.a

mas o que aconteceria se tomassemos diferenças em um polinomio de grau
2? vamos ver o que acontece?, vamos tomar um polinomio simples de grau
2, por exemplo y=x^2
e tomar valores partindo de zero, e pulando de 1 em 1, vamos lá

x=0, y=0
x=1, y=1
x=2, y=4
x=3, y=9
x=4, y=16
x=5, y=25
vamos tomar entao os resultados y, em sequencia
 0-14---916-25
e se tiramos as diferenças, o que acontece?, vamos lá
 0-14916-25
1-35---7--9
aparece nas diferenças, uma sequencia que não é constante, vamos então
tirar a diferença dessa sequencia que apareceu (diferença das
diferenças) (observe que as diferenças são os termos da primeira
sequencia considerada, a que surgiu de y=2x+1)
tomando as diferenças temos entao
1-35---7--9
--2--22---2
uma constante 2, vimos então que, tomando um polinomio de grau 2 (o
y=x^2), e tomando as diferenças, temos uma sequencias onde a diferença
é uma p.a e a segunda diferença é constante, e o que aconteceria com
um polinomio de grau 3? (vou fazer só mais esse caso)

exemplo y=x^3, tomando valores, começando do zero, temos
x=0 y=0
x=1, y=1
x=2, y=8
x=3, y=27
x=4 , y=64
x=5, y=125
pondo em ordem e tirando as diferenças temos

0---18-27-64125
---1---719-37--61
--6---1218--24
-6-6--6
a primeira diferença nao é constante, a segunda diferença não é
constante, porém a terceira diferença é constante

com isso podemos perceber algumas coisas, como, nos casos analisados,
a n diferença de um polinomio de grau n é constante, e como a
diferença de constante é zero, temos a n+1 diferença de um polinomio
de grau n é zero.
dos exemplos, diferença de termos no polinomio de grau 1, 2x+1 ´e constante,
a 2 segunda diferença dos termos de um polinomio de grau 2 é constante
(no caso testado x^2)
a terceira diferença de um polinomio de grau 3 é constante( do caso x^3)

mas como definir essas sequencias?
a sequencia cuja segunda diferença é constante, é uma p.a de ordem 2,
a sequencia cuja terceira diferença é constante é uma p.a de ordem 3,
a sequencia onde a n esima diferença é constante, é uma p.a de ordem n
( sendo as constantes diferentes de zero)

no proximo email uma dedução das primeiras formulas de p.a e
extrapolação pra todos outros casos

=
Instruções para entrar na lista, sair da lista e usar a lista em
http://www.mat.puc-rio.br/~obmlistas/obm-l.html
=


Re: [obm-l] Diferença finita ( de novo)

2008-02-24 Por tôpico Rodrigo Renji
a fórmula você pode deduzir assim,
vou chamar o operador delta, de D (nao confundir com derivada), o
operador delta faz Df(x)=f(x+1)-f(x), seja o operador E que faz
Ef(x)= f(x+1), entao podemos escrever  D f(x)= Ef(x)-f(x)  (é possivel
definir operaçãoes analogas a soma, produto , potenciação, com esses
operadores e mostrar que formam um anel , sendo valida tb uma
propriedade similar ao binomio de newton, vou usar ela pra deduzir as
formulas das PA),
 podemos escrever D f(x)= (E-1) f(x)
as diferenças de ordem superior são definidas assim

D^0 f(x)=f(x)

D^(k+1) f(x)= D^k f(x+1) - D^kf(x) para k0 , k natural
definindo tb E^k f(x)= f(x+k)

é válido fazer o seguinte
primeiro
D^n f(x)= (E-1)^n f(x) expandindo esse segundo termo por teoremade
binomio de newton
vou escrever o coeficiente binomial como c(n,k)= n!/k!(n-k)!, temos
D^nf(x)= somatorio [k=0 ate n] de c(n,k) E^(n-k) .(-1)^k f(x)
assim voce tem as diferenças escritas em função de valores sucessivos da função

mas da relação D=E-1, temos D+1=E, tomando potencias n em cada temos
(D+1)^n =E^n, aplicando numa função f(x) temos
E^n f(x)= (D+1)^n f(x) expande por teorema binomial
E^n f(x)= somatorio [k=0 até n] c(n,k) D^k f(x)
f(x+n)= somatorio [k=0 até n] c(n,k) D^k f(x)

se voce fizer x=0 tem

f(n)= somatorio [k=0 até n] c(n,k) D^k f(0)

se fizer x=1 e n=p-1 temos
f(p-1+1)=f(p)= somatorio [k=0 até p-1] c(p-1,k) D^k f(1)

que é a formula que se quer
Em 31/10/01, Pedro[EMAIL PROTECTED] escreveu:


 Essa sequecncia foi resolvida Pelo Professor Luís lopes (em 2003)de maneira
 brilhante, muito mais eficaz do que diferença finita.
 Pergunto ao Professor ou os demais da lista.Como demonstrar as fórmulas que
 estão em negritos a abaixo.

 1)Seja a PA de ordem 3

 1,3,19,61,141,271,... a_i

 Vamos gerar outras PAs fazendo a_{i+1} - a_i:

 2,16,42,80,130Delta a_i
 14,26,38,50 Delta^2 a_i
 12,12,12  Delta^3 a_i

 a_i = a_1 + Delta a_1 binom(i-1,1) + Delta^2 a_1
 binom(i-1,2) + Delta^3 a_1 binom(i-1,3)
 a_i = 1 + 2(i-1) + 14(i-1)(i-2)/2 + 12(i-1)(i-2)(i-3)/6
 a_i = 2i^3 - 5i^2 + 3i + 1

 S_n = a_1 binom(n,1) + Delta a_1 binom(n,2) +
 Delta^2 a_1 binom(n,3) + Delta^3 binom(n,4)

 S_n = n[3n^3 - 4n^2 - 3n + 10] / 6

 2) Posso reslver da mesma forma a seguinte questão (arrumando)

 Determine o termo geral da sequência {  3, 0, 5, 34 , 135,
 452} e calcule em seguida a soma dos n primeiros termos.


=
Instruções para entrar na lista, sair da lista e usar a lista em
http://www.mat.puc-rio.br/~obmlistas/obm-l.html
=


Re: [obm-l] Re: [obm-l] Diferenças finitas

2008-02-16 Por tôpico Rodrigo Renji
Aproveitando pra perguntar, tem alguem da lista que estuda diferenças finitas?

Em 13/02/08, Rodrigo Renji[EMAIL PROTECTED] escreveu:
 faltou calcular o somatorio, que é simples

 soma [n=0, p-1]f(n)=soma [n=0, p-1] [2.3^(n)   -7.n   +1]
  3^n  -7n(n-1)/2 +n|^(p)_(0)= 2.3^p-7(p)(p-1)/2 +p -1

 Em 11/02/08, Rodrigo Renji[EMAIL PROTECTED] escreveu:
  mais comentários sobre esse problema
 
  se a função que fornece a sequencia, é essa função abaixo
  f(n)=2.3^(n)   -7.n   +1
 
  exista maneira de, definir a sequencia , sem dar explicitamente a função
  uma delas é dar a recorrencia
  (E-3)(E-1)^2 f(n)=0
  isto é
  f(n+3)=5f(n+2)-7f(n+1)+3f(n)
 
  com condições iniciais
  f(0)=3
  f(1)=0
  f(2)=5
 
  Em 11/02/08, Rodrigo Renji[EMAIL PROTECTED] escreveu:
   Se quiser ver uns textos que estou escrevendo sobre cálculo finito
   estou enviando eles por link
  
   http://www.4shared.com/dir/5666586/1526a193/meus_textos_atualizados.html
  
   no 4shared, nenhum dos textos é versão final, são apenas esboços ainda...
   estou atualizando eles ainda,
  
   o link que mandei é o link da pasta com os textos, acessando o link
   vai poder ver varios arquivos em formato pdf
  
   no numerosespeciais, sobre numeros de stirling
  
   calculo simbolico, formula de soma de euler maclaurin, series de tg x, 
   cotgx
  
   funçõesfatoriais - potencia fatorial e outras funções
  
   recorrenciaedivisibilidade - aplicação de recorrencia a divisibilidade
  
   numerosespeciais2- aplicação de calculo finito ao estudo de numeros
   poligonais e suas intersecções
  
   definições- principais definições para entender o texto
  
   operadores- definição e teoremas pros operadores do calculo finito
  
   abraços o/
  
   Em 01/11/01, Pedro[EMAIL PROTECTED] escreveu:
Vasculhando os meus livros encontrei a questao do livro:MANUAL DE
PROGRESSÕES de Luís lopes.Questõa 102.
   
- Original Message -
From: Rodrigo Renji [EMAIL PROTECTED]
To: obm-l@mat.puc-rio.br
Sent: Monday, February 11, 2008 6:11 PM
Subject: Re: [obm-l] Diferenças finitas
   
   
uma função simples que interpola os numeros iniciais dados é
   
f(n)=2.3^(n)   -7.n   +1
   
porem concordo com o comentário do bruno, a sequencia nao esta definida
para definir bem ela é necessário dizer a maneira que ela é gerada, o
que facilitaria para achar a fórmula geral
   
uma sequencia finita qualquer, tem infinitas formulas que as interpola
   
sobre links de diferenças finitas, eu estou escrevendo um texto,
depois envio aqui
como deduzi esse f(n) e link para texto
   
abraços
   
Em 11/02/08, Bruno França dos Reis[EMAIL PROTECTED] escreveu:
 Essa questão não da pra resolver da forma como esta posta. Ou melhor,
 qualquer resposta estara certa.

 Vc pode dizer que o termo geral é:
 a_i = 0, i = 7
 e para a_1, a_2, ..., a_6, os valores que vc deu.
 Ta ai, minha sequencia (a_n)_(n Natural) satizfaz seu enunciado.

 Ta vendo? poderiamos ter dito QUAISQUER outros valores para a_i, i = 
 7 e
 teriamos resolvido o exercicio. Se isso foi algum professor seu que te
 propos, faça o favor de lhe dizer para formular melhor suas questões.

 Abraço
 Bruno

 ps: A questao que acredito ser a que vc tem em mente pode ser 
 formulada de
 forma a admitir somente a resposta que vc quer, se vc pedir uma 
 progressao
 aritmetica de ordem minima para satisfazer esses primeiros tantos 
 termos.

 pps: Essas questões de adivinhe a sequencia sempre voltam à lista! 
 Não
 critico quem perguntem aqui, de forma alguma, mas critico as possiveis
 fontes da pergunta: provas, exames que colocam esse tipo de questão...


 On 01/11/2001, Pedro [EMAIL PROTECTED] wrote:
 
 
 
  Amigos da lista, vocês poderiam me indica um site  em portuques  
  sobre :
 DIFERENÇAS FINITAS. Diferença finitas é mesma coisa de progressões
 aritméticas  de  ordem superior.
 
   Eu acho que a questão a seguir sai por diferenças finitas.
 
   Como resolvo essa questão: Determine o termo geral da sequência {  
  3,
  0,
 5, 34 , 135, 452} e calcule em seguida a soma dos n primeiros
 termos.



 --
 Bruno FRANÇA DOS REIS

 msn: [EMAIL PROTECTED]
 skype: brunoreis666
 tel: +33 (0)6 28 43 42 16

 e^(pi*i)+1=0
   
=
Instruções para entrar na lista, sair da lista e usar a lista em
http://www.mat.puc-rio.br/~obmlistas/obm-l.html
=
   
   
=
Instruções para entrar na lista, sair da lista e usar a lista em
http://www.mat.puc-rio.br/~obmlistas/obm-l.html

Re: [obm-l] Re: [obm-l] Diferenças finitas

2008-02-13 Por tôpico Rodrigo Renji
faltou calcular o somatorio, que é simples

soma [n=0, p-1]f(n)=soma [n=0, p-1] [2.3^(n)   -7.n   +1]
 3^n  -7n(n-1)/2 +n|^(p)_(0)= 2.3^p-7(p)(p-1)/2 +p -1

Em 11/02/08, Rodrigo Renji[EMAIL PROTECTED] escreveu:
 mais comentários sobre esse problema

 se a função que fornece a sequencia, é essa função abaixo
 f(n)=2.3^(n)   -7.n   +1

 exista maneira de, definir a sequencia , sem dar explicitamente a função
 uma delas é dar a recorrencia
 (E-3)(E-1)^2 f(n)=0
 isto é
 f(n+3)=5f(n+2)-7f(n+1)+3f(n)

 com condições iniciais
 f(0)=3
 f(1)=0
 f(2)=5

 Em 11/02/08, Rodrigo Renji[EMAIL PROTECTED] escreveu:
  Se quiser ver uns textos que estou escrevendo sobre cálculo finito
  estou enviando eles por link
 
  http://www.4shared.com/dir/5666586/1526a193/meus_textos_atualizados.html
 
  no 4shared, nenhum dos textos é versão final, são apenas esboços ainda...
  estou atualizando eles ainda,
 
  o link que mandei é o link da pasta com os textos, acessando o link
  vai poder ver varios arquivos em formato pdf
 
  no numerosespeciais, sobre numeros de stirling
 
  calculo simbolico, formula de soma de euler maclaurin, series de tg x, cotgx
 
  funçõesfatoriais - potencia fatorial e outras funções
 
  recorrenciaedivisibilidade - aplicação de recorrencia a divisibilidade
 
  numerosespeciais2- aplicação de calculo finito ao estudo de numeros
  poligonais e suas intersecções
 
  definições- principais definições para entender o texto
 
  operadores- definição e teoremas pros operadores do calculo finito
 
  abraços o/
 
  Em 01/11/01, Pedro[EMAIL PROTECTED] escreveu:
   Vasculhando os meus livros encontrei a questao do livro:MANUAL DE
   PROGRESSÕES de Luís lopes.Questõa 102.
  
   - Original Message -
   From: Rodrigo Renji [EMAIL PROTECTED]
   To: obm-l@mat.puc-rio.br
   Sent: Monday, February 11, 2008 6:11 PM
   Subject: Re: [obm-l] Diferenças finitas
  
  
   uma função simples que interpola os numeros iniciais dados é
  
   f(n)=2.3^(n)   -7.n   +1
  
   porem concordo com o comentário do bruno, a sequencia nao esta definida
   para definir bem ela é necessário dizer a maneira que ela é gerada, o
   que facilitaria para achar a fórmula geral
  
   uma sequencia finita qualquer, tem infinitas formulas que as interpola
  
   sobre links de diferenças finitas, eu estou escrevendo um texto,
   depois envio aqui
   como deduzi esse f(n) e link para texto
  
   abraços
  
   Em 11/02/08, Bruno França dos Reis[EMAIL PROTECTED] escreveu:
Essa questão não da pra resolver da forma como esta posta. Ou melhor,
qualquer resposta estara certa.
   
Vc pode dizer que o termo geral é:
a_i = 0, i = 7
e para a_1, a_2, ..., a_6, os valores que vc deu.
Ta ai, minha sequencia (a_n)_(n Natural) satizfaz seu enunciado.
   
Ta vendo? poderiamos ter dito QUAISQUER outros valores para a_i, i = 7 
e
teriamos resolvido o exercicio. Se isso foi algum professor seu que te
propos, faça o favor de lhe dizer para formular melhor suas questões.
   
Abraço
Bruno
   
ps: A questao que acredito ser a que vc tem em mente pode ser formulada 
de
forma a admitir somente a resposta que vc quer, se vc pedir uma 
progressao
aritmetica de ordem minima para satisfazer esses primeiros tantos 
termos.
   
pps: Essas questões de adivinhe a sequencia sempre voltam à lista! Não
critico quem perguntem aqui, de forma alguma, mas critico as possiveis
fontes da pergunta: provas, exames que colocam esse tipo de questão...
   
   
On 01/11/2001, Pedro [EMAIL PROTECTED] wrote:



 Amigos da lista, vocês poderiam me indica um site  em portuques  
 sobre :
DIFERENÇAS FINITAS. Diferença finitas é mesma coisa de progressões
aritméticas  de  ordem superior.

  Eu acho que a questão a seguir sai por diferenças finitas.

  Como resolvo essa questão: Determine o termo geral da sequência {  3,
 0,
5, 34 , 135, 452} e calcule em seguida a soma dos n primeiros
termos.
   
   
   
--
Bruno FRANÇA DOS REIS
   
msn: [EMAIL PROTECTED]
skype: brunoreis666
tel: +33 (0)6 28 43 42 16
   
e^(pi*i)+1=0
  
   =
   Instruções para entrar na lista, sair da lista e usar a lista em
   http://www.mat.puc-rio.br/~obmlistas/obm-l.html
   =
  
  
   =
   Instruções para entrar na lista, sair da lista e usar a lista em
   http://www.mat.puc-rio.br/~obmlistas/obm-l.html
   =
  
 


=
Instruções para entrar na lista, sair da lista e usar a lista em
http://www.mat.puc-rio.br/~obmlistas/obm-l.html
=


[obm-l] numeros pentagonais quadrados

2008-02-12 Por tôpico Rodrigo Renji
Números quadrados são números que satisfazem a recorrência
q(n)=q(n-1)+2n -1

com condição inicial
q(1)=1

números pentagonais são numeros que satisfazem a recorrencia
P(n)=p(n-1)+3n-2

com condição inicial

p(1)=1


existem infinitos números que são pentagonais e quadrados

por exemplo para n=1, P(1) é quadrado  (um numero pentagonal que é quadrado)

para n=81 ,  p(81) é quadrado

ache uma fórmula fechada g(k) tal que g(k) fornece os valores n, tais que
p(n) é quadrado, em sequencia . com k variando nos naturais (inclui zero)

isto é g(0)=1 e P(1) é quadrado
g(1)=81 e P(81) é quadrado

ache a formula fechada de g(k)

=
Instruções para entrar na lista, sair da lista e usar a lista em
http://www.mat.puc-rio.br/~obmlistas/obm-l.html
=


Re: [obm-l] Diferenças finitas

2008-02-11 Por tôpico Rodrigo Renji
uma função simples que interpola os numeros iniciais dados é

f(n)=2.3^(n)   -7.n   +1

porem concordo com o comentário do bruno, a sequencia nao esta definida
para definir bem ela é necessário dizer a maneira que ela é gerada, o
que facilitaria para achar a fórmula geral

uma sequencia finita qualquer, tem infinitas formulas que as interpola

sobre links de diferenças finitas, eu estou escrevendo um texto,
depois envio aqui
como deduzi esse f(n) e link para texto

abraços

Em 11/02/08, Bruno França dos Reis[EMAIL PROTECTED] escreveu:
 Essa questão não da pra resolver da forma como esta posta. Ou melhor,
 qualquer resposta estara certa.

 Vc pode dizer que o termo geral é:
 a_i = 0, i = 7
 e para a_1, a_2, ..., a_6, os valores que vc deu.
 Ta ai, minha sequencia (a_n)_(n Natural) satizfaz seu enunciado.

 Ta vendo? poderiamos ter dito QUAISQUER outros valores para a_i, i = 7 e
 teriamos resolvido o exercicio. Se isso foi algum professor seu que te
 propos, faça o favor de lhe dizer para formular melhor suas questões.

 Abraço
 Bruno

 ps: A questao que acredito ser a que vc tem em mente pode ser formulada de
 forma a admitir somente a resposta que vc quer, se vc pedir uma progressao
 aritmetica de ordem minima para satisfazer esses primeiros tantos termos.

 pps: Essas questões de adivinhe a sequencia sempre voltam à lista! Não
 critico quem perguntem aqui, de forma alguma, mas critico as possiveis
 fontes da pergunta: provas, exames que colocam esse tipo de questão...


 On 01/11/2001, Pedro [EMAIL PROTECTED] wrote:
 
 
 
  Amigos da lista, vocês poderiam me indica um site  em portuques  sobre :
 DIFERENÇAS FINITAS. Diferença finitas é mesma coisa de progressões
 aritméticas  de  ordem superior.
 
   Eu acho que a questão a seguir sai por diferenças finitas.
 
   Como resolvo essa questão: Determine o termo geral da sequência {  3, 0,
 5, 34 , 135, 452} e calcule em seguida a soma dos n primeiros
 termos.



 --
 Bruno FRANÇA DOS REIS

 msn: [EMAIL PROTECTED]
 skype: brunoreis666
 tel: +33 (0)6 28 43 42 16

 e^(pi*i)+1=0

=
Instruções para entrar na lista, sair da lista e usar a lista em
http://www.mat.puc-rio.br/~obmlistas/obm-l.html
=


Re: [obm-l] Re: [obm-l] Diferenças finitas

2008-02-11 Por tôpico Rodrigo Renji
mais comentários sobre esse problema

se a função que fornece a sequencia, é essa função abaixo
f(n)=2.3^(n)   -7.n   +1

exista maneira de, definir a sequencia , sem dar explicitamente a função
uma delas é dar a recorrencia
(E-3)(E-1)^2 f(n)=0
isto é
f(n+3)=5f(n+2)-7f(n+1)+3f(n)

com condições iniciais
f(0)=3
f(1)=0
f(2)=5

Em 11/02/08, Rodrigo Renji[EMAIL PROTECTED] escreveu:
 Se quiser ver uns textos que estou escrevendo sobre cálculo finito
 estou enviando eles por link

 http://www.4shared.com/dir/5666586/1526a193/meus_textos_atualizados.html

 no 4shared, nenhum dos textos é versão final, são apenas esboços ainda...
 estou atualizando eles ainda,

 o link que mandei é o link da pasta com os textos, acessando o link
 vai poder ver varios arquivos em formato pdf

 no numerosespeciais, sobre numeros de stirling

 calculo simbolico, formula de soma de euler maclaurin, series de tg x, cotgx

 funçõesfatoriais - potencia fatorial e outras funções

 recorrenciaedivisibilidade - aplicação de recorrencia a divisibilidade

 numerosespeciais2- aplicação de calculo finito ao estudo de numeros
 poligonais e suas intersecções

 definições- principais definições para entender o texto

 operadores- definição e teoremas pros operadores do calculo finito

 abraços o/

 Em 01/11/01, Pedro[EMAIL PROTECTED] escreveu:
  Vasculhando os meus livros encontrei a questao do livro:MANUAL DE
  PROGRESSÕES de Luís lopes.Questõa 102.
 
  - Original Message -
  From: Rodrigo Renji [EMAIL PROTECTED]
  To: obm-l@mat.puc-rio.br
  Sent: Monday, February 11, 2008 6:11 PM
  Subject: Re: [obm-l] Diferenças finitas
 
 
  uma função simples que interpola os numeros iniciais dados é
 
  f(n)=2.3^(n)   -7.n   +1
 
  porem concordo com o comentário do bruno, a sequencia nao esta definida
  para definir bem ela é necessário dizer a maneira que ela é gerada, o
  que facilitaria para achar a fórmula geral
 
  uma sequencia finita qualquer, tem infinitas formulas que as interpola
 
  sobre links de diferenças finitas, eu estou escrevendo um texto,
  depois envio aqui
  como deduzi esse f(n) e link para texto
 
  abraços
 
  Em 11/02/08, Bruno França dos Reis[EMAIL PROTECTED] escreveu:
   Essa questão não da pra resolver da forma como esta posta. Ou melhor,
   qualquer resposta estara certa.
  
   Vc pode dizer que o termo geral é:
   a_i = 0, i = 7
   e para a_1, a_2, ..., a_6, os valores que vc deu.
   Ta ai, minha sequencia (a_n)_(n Natural) satizfaz seu enunciado.
  
   Ta vendo? poderiamos ter dito QUAISQUER outros valores para a_i, i = 7 e
   teriamos resolvido o exercicio. Se isso foi algum professor seu que te
   propos, faça o favor de lhe dizer para formular melhor suas questões.
  
   Abraço
   Bruno
  
   ps: A questao que acredito ser a que vc tem em mente pode ser formulada de
   forma a admitir somente a resposta que vc quer, se vc pedir uma progressao
   aritmetica de ordem minima para satisfazer esses primeiros tantos termos.
  
   pps: Essas questões de adivinhe a sequencia sempre voltam à lista! Não
   critico quem perguntem aqui, de forma alguma, mas critico as possiveis
   fontes da pergunta: provas, exames que colocam esse tipo de questão...
  
  
   On 01/11/2001, Pedro [EMAIL PROTECTED] wrote:
   
   
   
Amigos da lista, vocês poderiam me indica um site  em portuques  sobre :
   DIFERENÇAS FINITAS. Diferença finitas é mesma coisa de progressões
   aritméticas  de  ordem superior.
   
 Eu acho que a questão a seguir sai por diferenças finitas.
   
 Como resolvo essa questão: Determine o termo geral da sequência {  3,
0,
   5, 34 , 135, 452} e calcule em seguida a soma dos n primeiros
   termos.
  
  
  
   --
   Bruno FRANÇA DOS REIS
  
   msn: [EMAIL PROTECTED]
   skype: brunoreis666
   tel: +33 (0)6 28 43 42 16
  
   e^(pi*i)+1=0
 
  =
  Instruções para entrar na lista, sair da lista e usar a lista em
  http://www.mat.puc-rio.br/~obmlistas/obm-l.html
  =
 
 
  =
  Instruções para entrar na lista, sair da lista e usar a lista em
  http://www.mat.puc-rio.br/~obmlistas/obm-l.html
  =
 


=
Instruções para entrar na lista, sair da lista e usar a lista em
http://www.mat.puc-rio.br/~obmlistas/obm-l.html
=


Re: [obm-l] Diferenças finitas

2008-02-11 Por tôpico Rodrigo Renji
deduzindo então essa fórmula que coloquei pra sequencia

obs: a^n é  a elevado  n

primeiro, seja o operador Delta que vou simbolizar por D
qua aplicado em f(n) faz  Df(n)=f(n+1)-f(n) e com potencias maiores definida
D^p f(n)=D^(p-1) f(n+1) - D^(p-1) f(n)  com D^0 f(n) =f(n)

mostre que se f(n)= c.a^n  então
D^p f(n)= c.(a-1)^p .a^(n)
com isso temos D^p f(0)=c.(a-1)^p

mostre que, se temos um polinomio de grau 1 em n
dn+p, então D^2 (dn +p) =0

agora observe as diferenças da sequencia (que vou simbolizar da maneira abaixo)

sequencia ( 3) - (0)--( 5)- (34
)(135 )---(452)
--(-3)---(5)--(29)(101)(317)
---(8)--(24)--(72)(216)
-(16)--(48)-(144)
-(32)--(96)
---(64)

observe assim que
D^0 f(0)=3
D^1 f(0)=-3
D^2 f(0)=8=2^3
D^3 f(0)=16=2^4
D^4f(0)=32=2^5
D^5f(0)=64=2^6
observe que a partir de D^2 f(0) aparece uma sequencia com padrão exponencial
D^p f(0)=  2^(p+1) =2.2^p
igualando isso com o resultado D^p f(0)=c.(a-1)^p
c.(a-1)^p=2.2^p, basta tomar   c=2 e a-1 =2, logo a=3
logo temos a partir de D^2, uma função do tipo f'''(n)=2.3^n
agora como D^o f(0) e D^1 f(0) diferem do valor encontrado com a função
f'''(n) acima, porem a partir de D^2 f(0)  funciona, pela observação de que
se temos uma função dn+p a segunda diferença sai ser zero
testa uma solução do tipo
f(n)=2.3^n +dn +p
com f(0)= 2+p=3, ache p=1
e com f(1)=6+d+1=0 ache d=-7

dai voce tem
f(n)=2.3^n  -7n +1



Em 11/02/08, Rodrigo Renji[EMAIL PROTECTED] escreveu:
 uma função simples que interpola os numeros iniciais dados é

 f(n)=2.3^(n)   -7.n   +1

 porem concordo com o comentário do bruno, a sequencia nao esta definida
 para definir bem ela é necessário dizer a maneira que ela é gerada, o
 que facilitaria para achar a fórmula geral

 uma sequencia finita qualquer, tem infinitas formulas que as interpola

 sobre links de diferenças finitas, eu estou escrevendo um texto,
 depois envio aqui
 como deduzi esse f(n) e link para texto

 abraços

 Em 11/02/08, Bruno França dos Reis[EMAIL PROTECTED] escreveu:
  Essa questão não da pra resolver da forma como esta posta. Ou melhor,
  qualquer resposta estara certa.
 
  Vc pode dizer que o termo geral é:
  a_i = 0, i = 7
  e para a_1, a_2, ..., a_6, os valores que vc deu.
  Ta ai, minha sequencia (a_n)_(n Natural) satizfaz seu enunciado.
 
  Ta vendo? poderiamos ter dito QUAISQUER outros valores para a_i, i = 7 e
  teriamos resolvido o exercicio. Se isso foi algum professor seu que te
  propos, faça o favor de lhe dizer para formular melhor suas questões.
 
  Abraço
  Bruno
 
  ps: A questao que acredito ser a que vc tem em mente pode ser formulada de
  forma a admitir somente a resposta que vc quer, se vc pedir uma progressao
  aritmetica de ordem minima para satisfazer esses primeiros tantos termos.
 
  pps: Essas questões de adivinhe a sequencia sempre voltam à lista! Não
  critico quem perguntem aqui, de forma alguma, mas critico as possiveis
  fontes da pergunta: provas, exames que colocam esse tipo de questão...
 
 
  On 01/11/2001, Pedro [EMAIL PROTECTED] wrote:
  
  
  
   Amigos da lista, vocês poderiam me indica um site  em portuques  sobre :
  DIFERENÇAS FINITAS. Diferença finitas é mesma coisa de progressões
  aritméticas  de  ordem superior.
  
Eu acho que a questão a seguir sai por diferenças finitas.
  
Como resolvo essa questão: Determine o termo geral da sequência {  3, 0,
  5, 34 , 135, 452} e calcule em seguida a soma dos n primeiros
  termos.
 
 
 
  --
  Bruno FRANÇA DOS REIS
 
  msn: [EMAIL PROTECTED]
  skype: brunoreis666
  tel: +33 (0)6 28 43 42 16
 
  e^(pi*i)+1=0


=
Instruções para entrar na lista, sair da lista e usar a lista em
http://www.mat.puc-rio.br/~obmlistas/obm-l.html
=


Re: [obm-l] Re: [obm-l] Diferenças finitas

2008-02-11 Por tôpico Rodrigo Renji
Se quiser ver uns textos que estou escrevendo sobre cálculo finito
estou enviando eles por link

http://www.4shared.com/dir/5666586/1526a193/meus_textos_atualizados.html

no 4shared, nenhum dos textos é versão final, são apenas esboços ainda...
estou atualizando eles ainda,

o link que mandei é o link da pasta com os textos, acessando o link
vai poder ver varios arquivos em formato pdf

no numerosespeciais, sobre numeros de stirling

calculo simbolico, formula de soma de euler maclaurin, series de tg x, cotgx

funçõesfatoriais - potencia fatorial e outras funções

recorrenciaedivisibilidade - aplicação de recorrencia a divisibilidade

numerosespeciais2- aplicação de calculo finito ao estudo de numeros
poligonais e suas intersecções

definições- principais definições para entender o texto

operadores- definição e teoremas pros operadores do calculo finito

abraços o/

Em 01/11/01, Pedro[EMAIL PROTECTED] escreveu:
 Vasculhando os meus livros encontrei a questao do livro:MANUAL DE
 PROGRESSÕES de Luís lopes.Questõa 102.

 - Original Message -
 From: Rodrigo Renji [EMAIL PROTECTED]
 To: obm-l@mat.puc-rio.br
 Sent: Monday, February 11, 2008 6:11 PM
 Subject: Re: [obm-l] Diferenças finitas


 uma função simples que interpola os numeros iniciais dados é

 f(n)=2.3^(n)   -7.n   +1

 porem concordo com o comentário do bruno, a sequencia nao esta definida
 para definir bem ela é necessário dizer a maneira que ela é gerada, o
 que facilitaria para achar a fórmula geral

 uma sequencia finita qualquer, tem infinitas formulas que as interpola

 sobre links de diferenças finitas, eu estou escrevendo um texto,
 depois envio aqui
 como deduzi esse f(n) e link para texto

 abraços

 Em 11/02/08, Bruno França dos Reis[EMAIL PROTECTED] escreveu:
  Essa questão não da pra resolver da forma como esta posta. Ou melhor,
  qualquer resposta estara certa.
 
  Vc pode dizer que o termo geral é:
  a_i = 0, i = 7
  e para a_1, a_2, ..., a_6, os valores que vc deu.
  Ta ai, minha sequencia (a_n)_(n Natural) satizfaz seu enunciado.
 
  Ta vendo? poderiamos ter dito QUAISQUER outros valores para a_i, i = 7 e
  teriamos resolvido o exercicio. Se isso foi algum professor seu que te
  propos, faça o favor de lhe dizer para formular melhor suas questões.
 
  Abraço
  Bruno
 
  ps: A questao que acredito ser a que vc tem em mente pode ser formulada de
  forma a admitir somente a resposta que vc quer, se vc pedir uma progressao
  aritmetica de ordem minima para satisfazer esses primeiros tantos termos.
 
  pps: Essas questões de adivinhe a sequencia sempre voltam à lista! Não
  critico quem perguntem aqui, de forma alguma, mas critico as possiveis
  fontes da pergunta: provas, exames que colocam esse tipo de questão...
 
 
  On 01/11/2001, Pedro [EMAIL PROTECTED] wrote:
  
  
  
   Amigos da lista, vocês poderiam me indica um site  em portuques  sobre :
  DIFERENÇAS FINITAS. Diferença finitas é mesma coisa de progressões
  aritméticas  de  ordem superior.
  
Eu acho que a questão a seguir sai por diferenças finitas.
  
Como resolvo essa questão: Determine o termo geral da sequência {  3,
   0,
  5, 34 , 135, 452} e calcule em seguida a soma dos n primeiros
  termos.
 
 
 
  --
  Bruno FRANÇA DOS REIS
 
  msn: [EMAIL PROTECTED]
  skype: brunoreis666
  tel: +33 (0)6 28 43 42 16
 
  e^(pi*i)+1=0

 =
 Instruções para entrar na lista, sair da lista e usar a lista em
 http://www.mat.puc-rio.br/~obmlistas/obm-l.html
 =


 =
 Instruções para entrar na lista, sair da lista e usar a lista em
 http://www.mat.puc-rio.br/~obmlistas/obm-l.html
 =


=
Instruções para entrar na lista, sair da lista e usar a lista em
http://www.mat.puc-rio.br/~obmlistas/obm-l.html
=


[obm-l] Recorrencias e divisibilidade

2008-02-07 Por tôpico Rodrigo Renji
Usando recorrencias e conceito de divisibilidade cheguei na seguinte conclusão

seja um polinomio de grau p ,f(x).
Se tivermos
f(0)=f(1)=...=f(p) =0  mod k
(p+1 valores divisiveis por k)

então o polinomio f(x)
apresenta valores divisiveis por k, para todo x natural

isto é se pegarmos um polinomio de grau p, e testarmos os (p+1) valores
iniciais a partir do zero (valores naturais)
e eles forem divisiveis por k, então o polinomio vai ser sempre divisivel por k
quando avaliado em numeros naturais


a mesma coisa acontece com funções do tipo

f(n)=c0.(a0)^n +...+cp.(ap)^n
se os primeiros p+1 valores, forem divisiveis por k, então a função
vai dar sempre valores divisiveis por k

aplicações,: demonstrar problema de divisibilidade de funções usando
computadores e criar exemplos de funções divisiveis por um numero que
quisermos


essas proposições que coloquei acima são verdadeiras?
(escrevi um rascunho de demonstração em um texto)


exemplo de aplicação
mostre que
f(n)=n³+2n, é divisivel por 3

f(0)=0
f(1)=3
f(2)=12
f(3)=33

então é divisivel por 3

(uma mais simples ainda agora, que sai facinho por congruencia)

f(n)=4^n   -1

divisivel por 3, é pois

f(0)=0
f(1)=3
logo a função em n, é divisivel por 3.

=
Instruções para entrar na lista, sair da lista e usar a lista em
http://www.mat.puc-rio.br/~obmlistas/obm-l.html
=


[obm-l] Divisibilidade

2008-02-07 Por tôpico Rodrigo Renji
Usando recorrencias e conceito de divisibilidade cheguei na seguinte conclusão

seja um polinomio de grau p ,f(x).
Se tivermos
f(0)=f(1)=...=f(p) =0  mod k
(p+1 valores divisiveis por k)

então o polinomio f(x)
apresenta valores divisiveis por k, para todo x natural

isto é se pegarmos um polinomio de grau p, e testarmos os (p+1) valores
iniciais a partir do zero (valores naturais)
e eles forem divisiveis por k, então o polinomio vai ser sempre divisivel por k
quando avaliado em numeros naturais


a mesma coisa acontece com funções do tipo

f(n)=c0.(a0)^n +...+cp.(ap)^n
se os primeiros p+1 valores, forem divisiveis por k, então a função
vai dar sempre valores divisiveis por k

aplicações,: demonstrar problema de divisibilidade de funções usando
computadores e criar exemplos de funções divisiveis por um numero que
quisermos


essas proposições que coloquei acima são verdadeiras?
(escrevi um rascunho de demonstração em um texto)


exemplo de aplicação
mostre que
f(n)=n^3+2n, é divisivel por 3

f(0)=0
f(1)=3
f(2)=12
f(3)=33

então é divisivel por 3

(uma mais simples ainda agora, que sai facinho por congruencia)

f(n)=4^n   -1

divisivel por 3, é pois

f(0)=0
f(1)=3
logo a função em n, é divisivel por 3.
(alguns emails meus estão voltando, estou enviando denovo)

=
Instruções para entrar na lista, sair da lista e usar a lista em
http://www.mat.puc-rio.br/~obmlistas/obm-l.html
=


[obm-l] programação e matematica

2008-01-28 Por tôpico Rodrigo Renji
Quais programas vocês acham os mais poderosos (i.e tem mais vantagens)
para trabalhar com matematica? (programas do tipo, pascal, fortran c++, etc)
qual vale mais a pena aprender na sua opnião?
principalmente para testar e fazer programas em teoria dos números

abraços

=
Instruções para entrar na lista, sair da lista e usar a lista em
http://www.mat.puc-rio.br/~obmlistas/obm-l.html
=


Re: [obm-l] programação e matematica

2008-01-28 Por tôpico Rodrigo Renji
Albert, obrigado pela resposta, eu sei programar o básico em
javascript e pascal (mas os compiladores de pascal nao estava
aceitando o número grande que estava colocando nos arrays...) testei
no matlab tb mas o programa travou com o numero que coloquei (o
javascript fez o mesmo em alguns segundos)

agora vou tentar no fortran, mas tenho que aprender ainda...
me indicaram uma linguagem chamada Haskell, algume conhece? e sabe se
vale a pena aprender e usar ele?

Em 28/01/08, albert richerd carnier guedes[EMAIL PROTECTED] escreveu:
 Olha Rodrigo, para mim , trabalhar direto com matematica e fortran,
 tanto que deve ser este o motivo dele existir ainda.
 Com fortran é  muito mais facil criar funções e modularizar.
 Agora tem a questão de gosto também, pois tem gente que se sente bem
 melhor com C ou Pascal, mas minha opinião é que fortran é mais claro
 para isso.
 Abraços.

 Rodrigo Renji escreveu:
  Quais programas vocês acham os mais poderosos (i.e tem mais vantagens)
  para trabalhar com matematica? (programas do tipo, pascal, fortran c++, etc)
  qual vale mais a pena aprender na sua opnião?
  principalmente para testar e fazer programas em teoria dos números
 
  abraços
 
  =
  Instruções para entrar na lista, sair da lista e usar a lista em
  http://www.mat.puc-rio.br/~obmlistas/obm-l.html
  =
 
 

 =
 Instruções para entrar na lista, sair da lista e usar a lista em
 http://www.mat.puc-rio.br/~obmlistas/obm-l.html
 =


=
Instruções para entrar na lista, sair da lista e usar a lista em
http://www.mat.puc-rio.br/~obmlistas/obm-l.html
=


[obm-l] Números poligonais e equações de pell por recorrencia

2008-01-28 Por tôpico Rodrigo Renji
Comecei a estudar números poligonais, pensei que o assunto teriaapenas assuntos 
simples de inicio, achar a formula geral de númerospoligonais gerados por 
recorrencia, mas logo vi que nem todas osproblemas relacionados a esses números 
são tão simples para mim, noproblema de determinar quais números são quadrados 
e triangulares tiveum pouco de dificuldade e acabei vendo que a solução se 
relaciona comas equações chamadas equações de pell
link abaixo com representações do numeros quadrados e triangulares 
(nophotobucket, nao vou deletar as imagens)
http://i178.photobucket.com/albums/w268/rodrigo_renji/Untitled-1.gif
http://s178.photobucket.com/albums/w268/rodrigo_renji/?action=viewcurrent=nu.gif

analisando os numeros que satisfazem as duas condições atraves de umscript em 
javascript, percebi que a diferença entre eles fica cada vezmaior, então pensei 
que poderia ter uma recorrencia de segunda ordemque fosse formula geral desses 
números, (sei que esse assunto já foimuito estudado, mas tentei achar a solução 
do problema antes deprocurar algo para ler sobre)consegui então chegar a 
formula da recorrencia e provar que todos osnúmeros naquela recorrencia 
satisfazem as condições, porém naoconsegui demonstrar o seguinte (nesse 
problema eu peço ajuda)
seja 
f(n)=[(√2)/8][(3+√2]^n-(3-√2]^n]http://i178.photobucket.com/albums/w268/rodrigo_renji/Untitled-5.gif
então nao existe k entre f(n) e f(n+1)tal que
√(1+8k²) é inteiro, para todo n natural
para uma melhor  visualização eu coloquei as formulas num album nophotobucket tb
Fórmula que dá os valores f(n) que satisfazem, f(n)=[1+raiz(1+8k²)]/2,quando 
Raiz(1+8k²) é inteiro, ligada a números que são quadrados etriangulares e 
soluções da equação de 
pellhttp://i178.photobucket.com/albums/w268/rodrigo_renji/Untitled-10.gif
fórmula que dá os valores f(n) tais que Raiz(1+8f(n)²) é 
inteirohttp://i178.photobucket.com/albums/w268/rodrigo_renji/Untitled-5.gif

fórmula que dá os valores inteiros 
f(n)=raiz(1+8k²)http://i178.photobucket.com/albums/w268/rodrigo_renji/Untitled-15.gif

e um pequeno texto que estou escrevendo sobre 
issohttp://www.4shared.com/file/35993332/7282bad0/numerosespeciais2.html?dirPwdVerified=ad9cf664(não
 vou apagar o texto)
mas o problema então, em que preciso de ajuda (ja tentei bastante 
=x)(repetindo)é
seja 
f(n)=[(√2)/8][(3+√2]^n-(3-√2]^n]http://i178.photobucket.com/albums/w268/rodrigo_renji/Untitled-5.gif
então nao existe k entre f(n) e f(n+1)tal que
√(1+8k²) é inteiro, para todo n natural
ficarei muito grato se alguem poder me ajudar (e mais ainda se euconseguir 
entender =x)
abraços
=
Instru��es para entrar na lista, sair da lista e usar a lista em
http://www.mat.puc-rio.br/~obmlistas/obm-l.html
=


[obm-l] Re: Fórmula de soma de Euler-Maclaurin [dedução]

2008-01-05 Por tôpico Rodrigo Renji
http://www.4shared.com/file/33984703/eac6556a/calculosimbolico.html?dirPwdVerified=ad9cf664

atualizei coloquei uma dedução das séries de tangente de x, cotangente
x e x .cotgx
deu um pouco de trabalho deduzir isso passo a passo =x, se alguem
quiser ver ai está o texto
abraços

Em 03/01/08, Rodrigo Renji[EMAIL PROTECTED] escreveu:
 olá

 Colquei num site uma dedução que achei interessante da fórmula de
 euler-maclaurin para somatorio, associando o somatorio a integral, a
 dedução feita usando metodos simbolicos que estava lendo no livro do
 geoge boole, usa conceito de função geradora e números de bernoulli,
 quem quiser ver está no link abaixo( na página também há um link para
 texto em pdf que escrevi)

 http://iishp.5gbfree.com/matematica/calc/simbo.html

 abraços =p


=
Instruções para entrar na lista, sair da lista e usar a lista em
http://www.mat.puc-rio.br/~obmlistas/obm-l.html
=


Re: Res: [obm-l] Produto finito

2007-11-29 Por tôpico Rodrigo Renji
corrigindo
produtorio[k=0 até n] (1+k²)=(-1)^(n+1) = ( somatorio[k=0 até
n+1]s(n+1,k).i^(k) )( somatorio[k=0 até n+1] |s(n+1,k)|.i^(k) )

escrevi uma coisa errada era * em vez de =, assim

produtorio[k=0 até n] (1+k²)=(-1)^(n+1) * ( somatorio[k=0 até
n+1]s(n+1,k).i^(k) )( somatorio[k=0 até n+1] |s(n+1,k)|.i^(k) )

vou postar então o que já pensei sobre esse produtorio

notações
prod[a,b] f(k)= produtório de f(k) com k variando de a até b

no caso o produtório pedido foi
prod[1,n] (1+k²)  , se colocarmos k=0, temos 1+0², que não altera nada
no produtorio por 1 ser elemento neutro, podemos então escrever
prod[0,n] (1+k²) , vou chamar esse produtorio de f(n)
vimos a recorrencia que ele gera
prod[0,n] (1+k²) =prod[0,n-1] (1+k²) * (1+n²), implicando
f(n)=f(n-1)*(1+n²), fazendo n+1 em vez de n temos
f(n+1)=f(n)*(1+(n+1)²), todos termos são diferentes de zero , então
podemos tomar

f(n+1)/f(n)=(1+(n+1)³), chamarei f(n+1)/f(n) de Qf(n)
procuramos então uma função que aplicada o operador Q de (1+(n+1)³) ,
que eu não conheço a priori (=P), se conhecer o problema morre

mas chegamos em=Qf(n)= f(n+1)/f(n)=(1+(n+1)²)
porém podemos fazer o seguinte

seja Df(n)=f(n+1)-f(n), então
f(n+1)/f(n)=a^ D log f(n)_(a), pois
(seja sempre o log na base a)
f(n+1)/f(n)=a^log f(n+1) - log f(n)=a ^log f(n+1) . a^-log
f(n)==f(n+1)*f(n)^(-1)=f(n+1)/f(n)
logo Qf(n)=a^D log f(n)

com isso temos
a^D log f(n)=(1+(n+1)³), implicando
D log f(n)= log (1+ (n+1)²)

seja somatorio de k=0 até n-1 escrito como
soma [0, n-1], pode se mostrar que
soma [0,n-1] Df(k)= f(n)-f(0), usando isso na igualdade de logaritmo acima temos

soma[0, n-1]D log f(k)= log f(n)-log f(0 ), porém como f(n) é o
produtorio e como vimos que ele aplicado em zero, dá 1, temos f(0)=1 e
log 1=0, então a expressão fica como
log f(n)=soma[0, n-1]log (1+ (k+1)²)
tirando o log do primeiro membro, ficamos com
f(n)=a^soma[0, n-1]log (1+ (k+1)²)

continua






Em 29/11/07, albert richerd carnier guedes[EMAIL PROTECTED] escreveu:
 Rodrigo Renji escreveu:
  Cheguei em outro resultado doido pra esse produto, mas nem sei se esta 
  certo
 

  produtorio[k=0 até n] (1+k²)=(-1)^(n+1) = ( somatorio[k=0 até 
  n+1]s(n+1,k).i^(k) )( somatorio[k=0 até n+1] |s(n+1,k)|.i^(k) )

  onde s(n,k) são numeros de stirling do primeiro tipo com sinal |s(n,k)| 
  sendo o módulo desses números, i o número complexo.
 
  A conclusão partiu de (1+k²)=(k+i)(k-i) depois separar o produtorio em dois,
  depois usar a propriedade do produtorio que resultou poder ser escrito
  como somas de potencias com coeficientes em números de stirling, nada
  simples eu acho
 
  abraços
 
  Em 28/11/07, albert richerd carnier guedes[EMAIL PROTECTED] escreveu:
 
  Rodrigo Cientista escreveu:
  Caro Nehab,
 
 
  uma dúvida: os termos, individualmente, me parecem ser
 
  negativos, certo? (à exceção do primeoro que é=0), sendo assim,
  calcularíamos o fatorial de números negativos? exite isso? se sim, fatorial
  de número par seria positivo, e de número ímpar seria negativo, os mais
  geralmente, -N! = (-1)^N *
  N!
 
 
  ***
 
  Carlos
 
  Nehab
 
  Tue, 27 Nov 2007 01:45:00 -0800
  Oi, Albert (e Ponce)
  Faltou aplicar o
 
  fatorial em cada parcela do produtório...
 
  Nehab
 
  - Mensagem original
 
  
 
  De: Rogerio Ponce [EMAIL PROTECTED]
  Para:
 
  obm-l@mat.puc-rio.br
 
  Enviadas: Terça-feira, 27 de Novembro de 2007
 
  3:36:56
 
  Assunto: Re: [obm-l] Produto finito
 
  Ola' Albert,
  voce deve ter se
 
  enganado com alguma coisa no texto.
 
  Do jeito que esta' , o produto e' sempre
 
  zero.
 
 
  []'s
  Rogerio Ponce
 
 
 
  Em 27/11/07, albert richerd carnier
 
  guedes[EMAIL PROTECTED] escreveu:
 
 
 
  Olá. É a primeira vez que estou escrevendo para esta lista.
 
  Alguém sabe qual
 
  é o valor do produto finito
 
 
  P = ( 1 - 1^2 )( 1 - 2^2 )( 1 -3^2 )... ( 1 -
 
  N^2 )em função de N.
 
 
  Eu sei que ele possue o valor entre (N+1)! e
 
  (N+1)!N!.
 
 
  Agradeço qualquer
 
  sugestão.
 
  =
  Instruções
 
  para entrar na lista, sair da lista e usar a lista
  em
 
  http://www.mat.puc-rio.br/~obmlistas/obm-l.html
  =
 
 
  =
 
  Instruções
 
  para entrar na lista, sair da lista e usar a lista
  em
 
  http://www.mat.puc-rio.br/~obmlistas/obm-l.html
  =
 
 
 
  Abra sua conta no Yahoo! Mail, o único sem limite de espaço para
  armazenamento!
 
  http://br.mail.yahoo.com/
 
  =
  Instruções
 
  para entrar na lista, sair da lista e usar a lista
  em
 
  http://www.mat.puc-rio.br/~obmlistas/obm-l.html

Re: Res: [obm-l] Produto finito

2007-11-29 Por tôpico Rodrigo Renji
fica então dessa forma a responsabilidade, par o calculo da soma de um logaritmo
vendo que essa função satisfaz a recorrencia encontrada acima

f(n+1)=a^soma[0, n]log (1+ (k+1)²)=a^soma[0, n-1]log (1+ (k+1)²).
a^log (1+(n+1)²)=
f(n)*(1+(n+1)²)
então f(n+1)/f(n)=(1+(n+1)²), satisfaz a recorrência

agora sobre como aparece os números de stirling nesse problema

analisando sua solução do outro produtorio albert, tentei fazer esse
de forma analoga
prod[0,n](1+k²)
fatorando 1+k², com ajuda dos complexos
(k+i)(k-i)
prod[0,n](k+i)(k-i)=prod[0,n](k+i) *prod[0,n](k-i)=
prod[0,n](i+k) *prod[0,n](-1)(i-k)=
colocando o (-1) pra fora do produtorio
(-1)^(n+1).prod[0,n](i+k) *prod[0,n](i-k)
nos dois produtorio termo (abrindo de maneira informal)
 (i)(i+1)(i+2) (i+n) * (i)(i-1)(i-2)... (i-n) (-1)^(n+1)
temos duas potencias fatoriais multiplicadas, potencias fatoriais de
base complexa

a da esquerda vou escrever
(i)^(n+1,-1)
e da direita
(i)^(n+1,1) para potencias fatoriais de passo -1 e 1 (respectivamente
representando os produtorios a partir da esquerda)
dai temos isso então

(i)^(n+1,-1)* (i)^(n+1,1) * (-1)^(n+1)
porém é possivel escrever potências fatorias como soma de potencias
normais, atraves dos numeros de stirling, temos então
(i)^(n+1,-1)=soma[0,n+1] |s(n+1,k)| i^k
e

(i)^(n+1,1)=soma[0,n+1] s(n+1,k) i^k
logo o produtorio toma forma de

soma[0,n+1] |s(n+1,k)| i^k * soma[0,n+1] s(n+1,k) i^k. (-1)^(n+1)

onde s(n,k) são números de stirling do primeiro tipo (com sinal
alternado, se quiserem posso postar a demonstração e definição depois)
e |s(n,k) | o modulo deles, é isso (que não ajudou em nada =P)
abraços









Em 29/11/07, Rodrigo Renji[EMAIL PROTECTED] escreveu:
 corrigindo
 produtorio[k=0 até n] (1+k²)=(-1)^(n+1) = ( somatorio[k=0 até
 n+1]s(n+1,k).i^(k) )( somatorio[k=0 até n+1] |s(n+1,k)|.i^(k) )

 escrevi uma coisa errada era * em vez de =, assim

 produtorio[k=0 até n] (1+k²)=(-1)^(n+1) * ( somatorio[k=0 até
 n+1]s(n+1,k).i^(k) )( somatorio[k=0 até n+1] |s(n+1,k)|.i^(k) )

 vou postar então o que já pensei sobre esse produtorio

 notações
 prod[a,b] f(k)= produtório de f(k) com k variando de a até b

 no caso o produtório pedido foi
 prod[1,n] (1+k²)  , se colocarmos k=0, temos 1+0², que não altera nada
 no produtorio por 1 ser elemento neutro, podemos então escrever
 prod[0,n] (1+k²) , vou chamar esse produtorio de f(n)
 vimos a recorrencia que ele gera
 prod[0,n] (1+k²) =prod[0,n-1] (1+k²) * (1+n²), implicando
 f(n)=f(n-1)*(1+n²), fazendo n+1 em vez de n temos
 f(n+1)=f(n)*(1+(n+1)²), todos termos são diferentes de zero , então
 podemos tomar

 f(n+1)/f(n)=(1+(n+1)³), chamarei f(n+1)/f(n) de Qf(n)
 procuramos então uma função que aplicada o operador Q de (1+(n+1)³) ,
 que eu não conheço a priori (=P), se conhecer o problema morre

 mas chegamos em=Qf(n)= f(n+1)/f(n)=(1+(n+1)²)
 porém podemos fazer o seguinte

 seja Df(n)=f(n+1)-f(n), então
 f(n+1)/f(n)=a^ D log f(n)_(a), pois
 (seja sempre o log na base a)
 f(n+1)/f(n)=a^log f(n+1) - log f(n)=a ^log f(n+1) . a^-log
 f(n)==f(n+1)*f(n)^(-1)=f(n+1)/f(n)
 logo Qf(n)=a^D log f(n)

 com isso temos
 a^D log f(n)=(1+(n+1)³), implicando
 D log f(n)= log (1+ (n+1)²)

 seja somatorio de k=0 até n-1 escrito como
 soma [0, n-1], pode se mostrar que
 soma [0,n-1] Df(k)= f(n)-f(0), usando isso na igualdade de logaritmo acima 
 temos

 soma[0, n-1]D log f(k)= log f(n)-log f(0 ), porém como f(n) é o
 produtorio e como vimos que ele aplicado em zero, dá 1, temos f(0)=1 e
 log 1=0, então a expressão fica como
 log f(n)=soma[0, n-1]log (1+ (k+1)²)
 tirando o log do primeiro membro, ficamos com
 f(n)=a^soma[0, n-1]log (1+ (k+1)²)

 continua






 Em 29/11/07, albert richerd carnier guedes[EMAIL PROTECTED] escreveu:
  Rodrigo Renji escreveu:
   Cheguei em outro resultado doido pra esse produto, mas nem sei se esta 
   certo
  
 
   produtorio[k=0 até n] (1+k²)=(-1)^(n+1) = ( somatorio[k=0 até 
   n+1]s(n+1,k).i^(k) )( somatorio[k=0 até n+1] |s(n+1,k)|.i^(k) )
 
   onde s(n,k) são numeros de stirling do primeiro tipo com sinal |s(n,k)| 
   sendo o módulo desses números, i o número complexo.
  
   A conclusão partiu de (1+k²)=(k+i)(k-i) depois separar o produtorio em 
   dois,
   depois usar a propriedade do produtorio que resultou poder ser escrito
   como somas de potencias com coeficientes em números de stirling, nada
   simples eu acho
  
   abraços
  
   Em 28/11/07, albert richerd carnier guedes[EMAIL PROTECTED] escreveu:
  
   Rodrigo Cientista escreveu:
   Caro Nehab,
  
  
   uma dúvida: os termos, individualmente, me parecem ser
  
   negativos, certo? (à exceção do primeoro que é=0), sendo assim,
   calcularíamos o fatorial de números negativos? exite isso? se sim, 
   fatorial
   de número par seria positivo, e de número ímpar seria negativo, os mais
   geralmente, -N! = (-1)^N *
   N!
  
  
   ***
  
   Carlos
  
   Nehab
  
   Tue, 27 Nov 2007 01:45:00 -0800

Re: [obm-l] Res: [obm-l] Res: [obm-l] provas de convergência: sequência de fibonacci e análogas

2007-11-29 Por tôpico Rodrigo Renji
Alguém pode enviar algo sobre a série dos reciprocos da sequencia de fibonacci?
(convergencia  e irracionalidade )

abraços
Em 29/11/07, Nicolau C. Saldanha[EMAIL PROTECTED] escreveu:
 On Nov 29, 2007 11:37 AM, Rodrigo Cientista
 [EMAIL PROTECTED] wrote:
  Vou colocar oq considero a minha prova para a convergência:
 
  Inicialmente fiz algumas observações (usando a sequência de Lucas, mas pode 
  ser generalizado):
 
  a)(an)^2= (an-1)*(an+1) +- 5 (é +5 se n é par, e -5 se n é ímpar)  obs: 
  com +- quero dizer + ou -
  Note que eu não sei se isto é verdade para toda a sequência, já que uma 
  observação não é prova. Entretanto, se eu conseguisse provar que este fato 
  é verdade, eu poderia dividir todos os termos da equação por (an)*(an-1), 
  chegando à seguinte expressão:
 

 Isto é verdade trocando 5 por outra constante para qq solução de
 a_(n+1) = a_n + a_(n-1).
 Há várias maneiras de ver isso. A mais óbvia é usar a fórmula a_n = A
 phi^n + B phib^n.
 Outra é ver que

 [[a_(n+2),a_(n+1)],[a_(n+1),a_n]] = [[1,1],[1,0]] *
 [[a_(n+1),a_n],[a_n,a_(n-1)]]
 donde, tirando determinantes,

 a_(n+2)*a_n - (a_(n+1))^2 = - (a_(n+1)*a_(n-1) - (a_n)^2)


  (an)/(an-1) = (an+1)/(an) +-5/[(an)*(an-1)]
 
  Usando-se limites, vemos que quando n-- infinito, +-5/[(an)*(an-1)]-- 0, 
  e (an)/(an-1)-- (an+1)/(an), e por indução vemos que (an)/(an-1)-- razão 
  áurea.
 
  Usando a própria definição da sequência:
 
  (an)^2= (an-1)*(an+1) +- 5, sabe-se que an+1=an + an-1
 
  (an)^2= (an-1)*(an + an-1) +- 5 == (an)^2 - (an-1)^2= (an-1)*(an) +- 5 ==
 
  == (an + an-1)(an - an-1) = (an-1)*(an) +- 5, sabe-se que (an - an-1) = 
  an-2 ==
 
  == (an + an-1)(an-2) = (an-1)*(an) +- 5, sabe-se que an = an-1 + an-2 ==
 
  == (an + an-1)(an-2) = (an-1)*(an-1 + an-2 ) +- 5 ==
 
  == (an-2)*(an) + (an-1)*(an-2) = (an-1)^2 + (an-1)*(an-2) +- 5 == 
  (an-1)^2 = (an-2)*(an) +- 5
 
  comparando-se a expressão original com esta,
  (an)^2= (an-1)*(an+1) +- 5
  (an-1)^2 = (an-2)*(an) +- 5
 
  ou mais geralmente:
 
  (ai)^2 = (ai-1)*(ai+1) +- 5, com i=2,3,4,5,6...,n
 
  provando por indução sobre n
 
  Como já foi previamente dito, agora podemos resolver o limite, a saber:
 
  LIM (an)/(an-1) =LIM{ (an+1)/(an) +-5/[(an)*(an-1)] } n-- infinito
 
  LIM (an)/(an-1) =LIM(an+1)/(an) + LIM{ +-5/[(an)*(an-1)] } n-- infinito
 
  LIM{ +-5/[(an)*(an-1)] } n-- infinito é ZERO
 
  NO LIMITE, (an)/(an-1) = (an+1)/(an) + 0
 
  Assim a prova está completa!
 

 Infelizmente considero a sua demonstração incompleta (além de ser
 desnecessariamente complicada).
 Você demonstrou que
 lim ( (a_(n+1)/a_n) - (a_n/a_(n-1)) ) = 0
 Isto NÃO implica na existência de
 lim a_(n+1)/a_n
 Para ver isso, considere c_n = log(n).
 Temos
 lim c_(n+1) - c_n = 0
 mas
 lim c_n = +infinito.

 Em outras palavras, se o termo geral de uma série tende a zero isto
 não garante a convergência da série.

 N.

 =
 Instruções para entrar na lista, sair da lista e usar a lista em
 http://www.mat.puc-rio.br/~obmlistas/obm-l.html
 =


=
Instruções para entrar na lista, sair da lista e usar a lista em
http://www.mat.puc-rio.br/~obmlistas/obm-l.html
=


Re: [obm-l] provas de convergência: sequência de fibonacci e análogas

2007-11-28 Por tôpico Rodrigo Renji
Uma série que converge mais ainda não consegui ver a demonstração, que
está relacionada com a sequencia de fibonacci é a série dos reciprocos
do números de fibonacci, me falaram que ela converge para um número
irracional
1/1 +1/1+1/2+1/3+1/5+1/8+...
onde os termos do denominador são dados por
f(n+2)=f(n+1)+f(n)
com f(0)=1=f(1)

javascript:v=[];k=[];v[0]=eval(prompt(Entre com o termo
inicial,));v[1]=eval(prompt(Entre com o segundo
termo,));s=eval(prompt(Entre com o termo
desejado,));i=1;for(w=1;ws;w++){v[w+1]=v[w]+v[w-1]};k[0]=1/v[0];for(l=1;ls;l++){k[l]=k[l-1]+1/(v[l])};alert(k[s-1])

para que seja reciproca da sequencia de fibonacci os primeiros termos
devem ser 1 e 1, a terceira entrada deve ser o termo na sequencia dos
reciprocos que se deseja, se as condições iniciais forem alteradas a
sequencia muda e não é mais a sequencia reciproca de fibonacci, para
usar o script acima, só é necessário copiar ele e colar na barra de
endereços do navegador (internet explorer, firefox e outros), onde se
escreve os sites

abraços


eu fiz um script para testar para alguns números, script em javascript

Em 28/11/07, Nicolau C. Saldanha[EMAIL PROTECTED] escreveu:
 Não entendi.

 A seq de Fibo tende para +infinito então ela diverge (trivialmente).

 Pela sua mensagem suspeito que você esteja querendo provar que existe
 o limite lim a_(n+1)/a_n.
 Se for isso, segue facilmente da fórmula

 a_n = A phi^n + B phib^n

 onde phi = (1+sqrt(5))/2, phib = (1-sqrt(5))/2.

 Como phi  1 e -1  phib  0 temos lim a_n/(A phi^n) = lim ( 1 +
 (B/A)*(phib/phi)^n ) = 1 desde que A seja diferente de 0.
 Assim  lim a_(n+1)/a_n = lim (A phi^(n+1))/(A phi^n) = phi.

 On Nov 27, 2007 9:58 PM, Rodrigo Cientista
 [EMAIL PROTECTED] wrote:
  Alguém conheceria uma prova de convergência da sequência de fibonacci? ou 
  sequências com a mesma regra de formação (a de lucas, por exemplo: 
  1,3,4,7,11,18...)
 
  Dei uma prova de convergência feia  a partir da sequência de lucas (mas o 
  mesmo argumento vale para a sequência de fibonacci e qualquer outra)
 
  Repare que achar a razão áurea (pelo menos pelo método tradicional***) não 
  prova a convergência da sequência
 
  ***seja an = an-1 + an-2 a regra de formação; SE a sequência das razões 
  an/an-1converge para um limite L, então quando n-- infinito, an/an-1 -- L
 
  na verdade, no limite an/an-1 = L,  como an+1 = an + an-1, an/an-1 = (an + 
  an-1)/an = 1+an-1/an ==  L = 1 + 1/L == L^2 - L - 1 = 0 == L = (1 +ou- 
  5^1/2)/2,
 
  desprezando-se o caso do sinal negativo (pois an é sempre maior que an-1 e 
  no caso negativo L seria  1)
 
  Mas tudo isso baseado na suposição, gostaria de ver uma prova da 
  convergência mais bonita... (a minha é muito grande pra esse espaço)
 
 
Abra sua conta no Yahoo! Mail, o único sem limite de espaço para 
  armazenamento!
  http://br.mail.yahoo.com/
 
  =
  Instruções para entrar na lista, sair da lista e usar a lista em
  http://www.mat.puc-rio.br/~obmlistas/obm-l.html
  =
 

 =
 Instruções para entrar na lista, sair da lista e usar a lista em
 http://www.mat.puc-rio.br/~obmlistas/obm-l.html
 =


=
Instruções para entrar na lista, sair da lista e usar a lista em
http://www.mat.puc-rio.br/~obmlistas/obm-l.html
=


Re: [obm-l] Res: [obm-l] provas de convergência: sequência de fibonacci e análogas

2007-11-28 Por tôpico Rodrigo Renji
Rodrigo, você esta falando da forma geral dos termos da sequência de fibonacci?
se for ela pode ser deduzida assim
a sequencia de fibonacci satizfas a recorrencia
f(n+2)=f(n+1)+f(n)
com condições iniciais f(0)=1=f(1) (ou f(1)=f(2)=1)

um meio é chutar uma solução do tipo f(n)=b^n
ficando com
b^(n+2)=b^(n+1)+b^(n)
b^n .b² =b^n. b + b^n, queremos b^n diferente de zero
dai temos
b²=b+1
b²-b-1=0
então b=[1+ou -raiz(5)]/2

logo as soluções ficam
f(n)=c1.b1^n +c2.b2^n, onde b1 e b2 são as soluções da equação do
segundo grau acima c1 e c2 são constantes que devem ser determinadas
pelas condições iniciais da recorrencia, que no caso seriam
f(0)=1=f(1), tendo essas informações se chega na formula geral da
sequencia de fibonacci

tenho outro meio de chegar na mesma resposta usando operadores Delta
vou definir assim
Df(n)=f(n+1)-f(n) e o operador expansão Ef(n)=f(n+1)
E²f(n)=f(n+2)
é possivel fazer o seguinte
f(n+2)=f(n+1)+f(n)
E²f(n)=Ef(n)+f(n)
(E²-E-1)f(n)=0
que pode ser fatorado
(E-b1)(E-b2)f(n)=0
as soluções são f(n)=c1.b1^n+c2.b2^n
pois os operadores (E-b1)(E-b2), ZERAM esse tipo de função

abraços



Em 28/11/07, Rodrigo Cientista[EMAIL PROTECTED] escreveu:
 Nicolau, realmente eu estava me referindo à sequência das razões a_n/a_(n-1)

 Algo que eu não consegui entender é: vc se baseia na suposição de que o 
 limite existe, e caso ele exista é phi, isso que não entra na minha cabeça!

 Supondo que o limite existe, ele é igual a phi, mas eu não sei se ele existe, 
 então não entendi como usar a suposição da sua existência na prova de sua 
 própria existência. Eu não deveria, por exemplo, supor que ele não existe e 
 identificar a contradição decorrente dessa suposição (uma forma de prova)? Eu 
 nunca tinha visto a fórmula que você apresentou... chegou-se a essa fórmula 
 sem supor a existência do limite?

 Me perdôe se a pergunta é tôla, sou apenas um amador...

 Aguardo comentários



  Não entendi.
 
  A seq de Fibo tende para +infinito então ela diverge (trivialmente).
 
  Pela sua mensagem suspeito que você esteja querendo provar que existe
  o limite lim a_(n+1)/a_n.
  Se for isso, segue facilmente da fórmula
 
  a_n = A phi^n + B phib^n
 
  onde phi = (1+sqrt(5))/2, phib = (1-sqrt(5))/2.
 
  Como phi  1 e -1  phib  0 temos lim a_n/(A phi^n) = lim ( 1 +
  (B/A)*(phib/phi)^n ) = 1 desde que A seja diferente de 0.
  Assim  lim a_(n+1)/a_n = lim (A phi^(n+1))/(A phi^n) = phi.
 
  On Nov 27, 2007 9:58 PM, Rodrigo Cientista
  [EMAIL PROTECTED] wrote:
   Alguém conheceria uma prova de convergência da sequência de fibonacci? ou 
   sequências com a mesma regra de formação (a de lucas, por exemplo: 
   1,3,4,7,11,18...)
  
   Dei uma prova de convergência feia  a partir da sequência de lucas (mas 
   o mesmo argumento vale para a sequência de fibonacci e qualquer outra)
  
   Repare que achar a razão áurea (pelo menos pelo método tradicional***) 
   não prova a convergência da sequência
  
   ***seja an = an-1 + an-2 a regra de formação; SE a sequência das razões 
   an/an-1converge para um limite L, então quando n-- infinito, an/an-1 -- 
   L
  
   na verdade, no limite an/an-1 = L,  como an+1 = an + an-1, an/an-1 = (an 
   + an-1)/an = 1+an-1/an ==  L = 1 + 1/L == L^2 - L - 1 = 0 == L = (1 
   +ou- 5^1/2)/2,
  
   desprezando-se o caso do sinal negativo (pois an é sempre maior que an-1 
   e no caso negativo L seria  1)
  
   Mas tudo isso baseado na suposição, gostaria de ver uma prova da 
   convergência mais bonita... (a minha é muito grande pra esse espaço)
  
  
Abra sua conta no Yahoo! Mail, o único sem limite de espaço para 
   armazenamento!
   http://br.mail.yahoo.com/
  
   =
   Instruções para entrar na lista, sair da lista e usar a lista em
   http://www.mat.puc-rio.br/~obmlistas/obm-l.html
   =
  
 
  =
  Instruções para entrar na lista, sair da lista e usar a lista em
  http://www.mat.puc-rio.br/~obmlistas/obm-l.html
  =
 

 =
 Instruções para entrar na lista, sair da lista e usar a lista em
 http://www.mat.puc-rio.br/~obmlistas/obm-l.html
 =


  Abra sua conta no Yahoo! Mail, o único sem limite de espaço para 
 armazenamento!
 http://br.mail.yahoo.com/

 =
 Instruções para entrar na lista, sair da lista e usar a lista em
 http://www.mat.puc-rio.br/~obmlistas/obm-l.html
 =


=
Instruções para entrar na lista, sair da lista e usar a lista em

Re: Res: [obm-l] Produto finito

2007-11-28 Por tôpico Rodrigo Renji
Cheguei em outro resultado doido pra esse produto, mas nem sei se esta certo
produtorio[k=0 até n] (1+k²)=(-1)^(n+1). somatorio[k=0 até n+1]
s(n+1,k).i^(k).somatorio[k=0 até n+1] |s(n+1,k)|.i^(k)
onde s(n,k) são numeros de stirling do primeiro tipo com sinal
|s(n,k)| sendo o módulo desses números, i o número complexo

a conclusão partiu de (1+k²)=(k+i)(k-i) depois separar o produtorio em dois,
depois usar a propriedade do produtorio que resultou poder ser escrito
como somas de potencias com coeficientes em números de stirling, nada
simples eu acho

abraços

Em 28/11/07, albert richerd carnier guedes[EMAIL PROTECTED] escreveu:
 Rodrigo Cientista escreveu:
 Caro Nehab,

uma dúvida: os termos, individualmente, me parecem ser
 negativos, certo? (à exceção do primeoro que é=0), sendo assim,
 calcularíamos o fatorial de números negativos? exite isso? se sim, fatorial
 de número par seria positivo, e de número ímpar seria negativo, os mais
 geralmente, -N! = (-1)^N *
 N!

***

Carlos
 Nehab
Tue, 27 Nov 2007 01:45:00 -0800
Oi, Albert (e Ponce)
Faltou aplicar o
 fatorial em cada parcela do produtório...
Nehab

- Mensagem original
 
De: Rogerio Ponce [EMAIL PROTECTED]
Para:
 obm-l@mat.puc-rio.br
Enviadas: Terça-feira, 27 de Novembro de 2007
 3:36:56
Assunto: Re: [obm-l] Produto finito

Ola' Albert,
voce deve ter se
 enganado com alguma coisa no texto.
Do jeito que esta' , o produto e' sempre
 zero.

[]'s
Rogerio Ponce



Em 27/11/07, albert richerd carnier
 guedes[EMAIL PROTECTED] escreveu:

 Olá. É a primeira vez que estou escrevendo para esta lista.
Alguém sabe qual
 é o valor do produto finito

P = ( 1 - 1^2 )( 1 - 2^2 )( 1 -3^2 )... ( 1 -
 N^2 )em função de N.

Eu sei que ele possue o valor entre (N+1)! e
 (N+1)!N!.

Agradeço qualquer
 sugestão.
=
Instruções
 para entrar na lista, sair da lista e usar a lista
 em
http://www.mat.puc-rio.br/~obmlistas/obm-l.html
=

 =
Instruções
 para entrar na lista, sair da lista e usar a lista
 em
http://www.mat.puc-rio.br/~obmlistas/obm-l.html
=


 Abra sua conta no Yahoo! Mail, o único sem limite de espaço para
 armazenamento!
http://br.mail.yahoo.com/

=
Instruções
 para entrar na lista, sair da lista e usar a lista
 em
http://www.mat.puc-rio.br/~obmlistas/obm-l.html
=

 Só para não criar um buraco no assunto, a solução do produto

 P = ( 1 - 2^2 ).( 1 - 4^2 ).( 1 - 5^2 ) ... ( 1 - N^2 )

sempre começa em
 2, pois se começar em 1 fica tudo 0.

Ele é bem mais fácil de achar.
Se
 tivermos a_n = ( 1 - n^2 ), podemos colocar na forma

a_n = ( 1 - n )( 1 + n
 )

e teremos o produto

P = ... [( 1 - n ).( 1 + n )] ...[( 1 - N ).( 1 + N
 )]

e isso dá para separar em dois produtos mais fáceis

P_1 = ( 1 - 2 ) ...
 ( 1 - n ) ... ( 1 - N ) = (-1)^{N-1}.(N-1)!
P_2 = ( 1 + 2 ) ... ( 1 + n )
 ... ( 1 + N ) = ( N + 1 )!/2

E teremos

P = P_1 . P_2 = (-1)^{N-1}.(N-1)!(
 N +1 )!/2


Pouca gente fala sobre produtos finitos, mas eu gosto muito
 deles.
Não sei pra que servem, mas acho muito legais.

 =
 Instruções para entrar na lista, sair da lista e usar a lista em
 http://www.mat.puc-rio.br/~obmlistas/obm-l.html
 =

=
Instruções para entrar na lista, sair da lista e usar a lista em
http://www.mat.puc-rio.br/~obmlistas/obm-l.html
=


Re: [obm-l] Produto finito

2007-11-27 Por tôpico Rodrigo Renji
Cheguei no resultado (tem que calcular um somatorio =x), acho que vão
achar trivial mas lá vai (nao deve acrescentar nada talvez...)
seja somatorio de n=0 até k-1 de uma função f(n), escrita como
soma[0,k-1]f(n)
e o logaritmo na base a escrito como logx_(a)
então uma função que satisfaz a relação

f(k)=a^ (soma[n=0,k-1]log[1+(n+1)^2]_(a))
´e solução da recorrencia
abraços

Em 27/11/07, Marcelo Salhab Brogliato[EMAIL PROTECTED] escreveu:
 Olá Albert,

 P_n = (1 + 1^2)(1 + 2^2)(1 + 3^2)...(1 + n^2)

 usando recorrencias, temos:
 P_1 = (1 + 1^2) = 2
 P_(n+1) = P_n * (1 + (n+1)^2)

 deste modo: P_(n+1) = P_n + P_n*(n+1)^2
 assim: dP_n = P_(n+1) - P_n = P_n * (n+1)^2

 temos que resolver: dP_n = P_n * (n+1)^2
 ou entao: P_(n+1)/P_n = 1 + (n+1)^2 = n^2 + 2n + 2

 vou tentar no papel e, se eu conseguir, eu mando ;)

 abraços,
 Salhab




=
Instruções para entrar na lista, sair da lista e usar a lista em
http://www.mat.puc-rio.br/~obmlistas/obm-l.html
=


Re: [obm-l] Produto finito

2007-11-27 Por tôpico Rodrigo Renji
a recorrencia você não acharia assim

produtorio de g(k) com k variando de a até p vou representar por
prod[a,p]g(k)
no caso temos
prod[0,n](1+k²)
podemos fazer
prod[0,n]1+k²=f(n)
por propriedade do produtorio temos
prod[0,n]1+k²=(prod[0,n-1]1+k²)*(1+n²)  (aqui abri o ultimo termo)
sendo prod[0,n]1+k²=f(n), então (prod[0,n-1]1+k²)=f(n-1)
logo
f(n)=f(n-1)*(1+n²), com condição inicial f(1)=2, como todos termos
diferentes de zero, podemos dividir por f(n-1) em ambos lados, ficando
com
f(n)/f(n-1)=(1+n²) , mudando de n para n+1
f(n+1)/f(n)=1+(n+1)²
agora o problema é saber se existe, e se existe qual é a função cujo
quociente acima dá 1+(n+1)²

abraçs

Em 27/11/07, ralonso[EMAIL PROTECTED] escreveu:
 Acho que dah para fazer por indução e formar uma equação
 de diferenças, mas ainda, para ser sincero não pensei com calma,
 veja:


 P = ( 1 + 1^2 )( 1 + 2^2 )( 1 + 3^2 )... ( 1 + N^2 )


 P_1  = ( 1 + 1^2 )
 P_2  = ( 1 + 1^2 )( 1 + 2^2 )
  = (1 + 2^2) +  1^2 (1+2^2)
   = (1 + n^2) + (n-1)^2 P_n/P_(n-1)


 Assim P_n = (1 + n^2) + (n-1)^2 P_n / P_(n-1)
 O grande problema neste caso é que essa equação de diferenças é
 meio complicada de resolver para achar P_n :)



 [EMAIL PROTECTED] wrote:
 Rogerio Ponce escreveu:
 Ola' Albert,
voce deve ter se enganado com alguma coisa no texto.
Do jeito
 que esta' , o produto e'  sempre zero.

[]'s
Rogerio Ponce



Em 27/11/07,
 albert richerd carnier guedes[EMAIL PROTECTED] escreveu:
 Olá. É a primeira vez que estou escrevendo para esta lista.
Alguém sabe qual
 é o valor do produto finito

P = ( 1 - 1^2 )( 1 - 2^2 )( 1 -3^2 )... ( 1 -
 N^2 )em função de N.

Eu sei que ele possue o valor entre (N+1)! e
 (N+1)!N!.

Agradeço qualquer
 sugestão.
=
Instruções
 para entrar na lista, sair da lista e usar a lista
 em
http://www.mat.puc-rio.br/~obmlistas/obm-l.html
=

 =
Instruções
 para entrar na lista, sair da lista e usar a lista
 em
http://www.mat.puc-rio.br/~obmlistas/obm-l.html
=





 =
 Instruções para entrar na lista, sair da lista e usar a lista em
 http://www.mat.puc-rio.br/~obmlistas/obm-l.html
 =







=
Instruções para entrar na lista, sair da lista e usar a lista em
http://www.mat.puc-rio.br/~obmlistas/obm-l.html
=


Re: [obm-l] Conjuntos finitos

2007-10-29 Por tôpico Rodrigo Renji
Marcelo, obrigado , a fórmula está estranha pois foi eu  formulei
hehehe, e que eu estava pensando no seguinte, se temos um conjunto de
números com uma relação de ordem  finito ele possui um máximo e se
formos tirando o máximo do conjunto em cada etapa, se ele é finito
chega uma  hora que ele se torna o conjunto vazio
ex:
{0, 2.2 ,3}
tira o máximo (3)
{ 0, 2.2}
tira o maximo (2.2)
{0}
tira o maximo 0
{}

acho que o mesmo acontece quando vamos tirando o minimo do conjunto
não é? se ele é um conjunto de numeros com relação de ordem e finito,
chega uma hora que ficamos com o conjunto vazio (só pra explicar o que
eu estava/estou pensando sobre essa questão).  A principio estava
pensando que valia pra qualquer conjunto finito, mas é obvio que não
( conjunto {a} a sendo a letra do alfabeto , é um conjunto finito não
é?, tendo apenas um elemento, ou essa nomenclatura é apenas para
conjuntos númericos ?)


esses conceitos eu estou usando para tentar definir formalmente
somatorio sobre um conjunto finito de numeros com relação de ordem.

abraços
Em 27/10/07, Marcelo Salhab Brogliato[EMAIL PROTECTED] escreveu:
 Olá Rodrigo,

 achei a formulação um tanto estranha... nao acho correto dizer: Existe n
 tal que S(n) = vazio... pois n está definido na questão..
 acredito que deveria ser: Se S(n) = vazio, entao |S(n)| = n ?

 |S(0)| = |S|
 |S(1)| = |S(0) - {max S(n)}| .. como {max S(n)} E S(0), e |{max S(n)}| = 1,
 entao: |S(1)| = |S| - 1
 por inducao: |S(k)| = |S| - k

 vamos supor que |S|  n, entao |S(n)|  0, absurdo! Pois |S(n)| = 0 por
 hipótese..
 vamos supor que |S|  n, entao |S(|S|)| = 0... assim: |S(n)| = 0
 vamos supor que |S| = n, entao |S(n)| = 0... assim: |S(n)| = 0

 logo, podemos concluir que S é finito, e que a cardinalidade S é menor ou
 igual a n...

 abraços,
 Salhab



 On 10/27/07, Rodrigo Renji [EMAIL PROTECTED] wrote:
 
  Seja
  S um conjunto
  defino
  (n natural)
 
  S(n+1)=S(n)-{max S(n)}
  S(0)=S
 
  (se S(n) possui máximo) [prestar atenção nessa condição]
 
  Se existe n, tal que s(n)=vazio
  então n é finito e tem n elementos?
 
  e se um conjunto é finito vale a propriedade acima?
  (relaçao de se e somente se).
 
 
 =
  Instruções para entrar na lista, sair da lista e usar a lista em
  http://www.mat.puc-rio.br/~obmlistas/obm-l.html
 
 =
 



=
Instruções para entrar na lista, sair da lista e usar a lista em
http://www.mat.puc-rio.br/~obmlistas/obm-l.html
=


Re: [obm-l] Expansão de termos -proposta de problema

2007-10-29 Por tôpico Rodrigo Renji
Puxa eu tive maior trabalho pra fazer isso (em um intervalo entre
aulas na faculdade) mas acabei chegando na mesma recorrência. Sobre os
números de stirling do segundo tipo eu estudei um pouco deles e
demonstrei como se transformar esses fatores tem algum tempo... eu li
no livro do Knuth, concrete mathematics, foi lá que aprendi um pouco
disso, uma coisa que acho interessante é o seguinte

é possivel transformar o produto dos fatores

x(x-h)(x-2h). ... . (x-h(n-1)) em potencias usando numeros de stirling
do primeiro tipo tb
eu acho que a formula é a seguinte
somatorio k=0 até n de [n,k] h^( n-k) .x^k, acho que h, pode ser real
ou complexo.
([n,k]) numeros de stirling do primeiro tipo

e a outra transformar potencia
x^n em soma de termos do tipo  x(x-h)(x-2h). ... . (x-h(n-1))

acho que fica da forma
x^n=somatorio k=0 até n de {n,k} h^(n-k) .x^(n,h)

onde x^(n,h)=x(x-h)(x-2h). ... . (x-h(n-1))
e {n,k} são numeros de stirling do segundo tipo
no ingles são chamados de factorial power, eu chamo de potencias fatoriais

tem relaçoes que acho bonitas com x^(n,h), primeiro que x^(n,0)=x^n
segundo n^(n,1)=n!
somatorio x^(n,1)=x^(n+1,1)/n+1
seja Df(x)=f(x+1)-f(x) então Dx^(n,1)=nx^(n-1,1)

com os numeros de stirling é possivel transformar a potencia em
potencia fatorial, depois calcular o somatorio de maneira mais simples

um outro exemplo de relação onde aparecem numeros de stirling
enesima derivada de f(e^x), os coeficientes que aparecem são numeros
de stirling do segundo tipo, eu cheguei num resultado envolvendo os
dois numeros

seja um operador T (pode ser numero) que satisfaz
T^0 g(x)=g(x)
T[T^n g(x)]=T^(n+1)g(x)
que T possa passar para dentro de somatorios

seja um operador P que satisfaz (pode ser numeros)
P^0 g(x)=g(x)
P[P^n g(x)]=P^(n+1)g(x) e
Pkg(x)=kPg(x) para qualquer número P
com aplicação de T e P comutando
se temos uma função de duas variaveis g(k,x) e g(x)
g(k,x) definida pelo menos para numeros naturais
e
g(x)=g(0,x) e com T aplicando em g(k,x) resultando
Tg(k,x)=kg(k,x)+g(k+1,x)

então
T^n g(x)= soma k=0 até n de {n,k} P^(n-k)g(k,x)
e com algumas outras relações temos

g(n,x)= soma k=0 ate n [n,kP^(n-k)T^k g(x)

essas relações, acho, que contem as outras postadas aqui

abraços =P

Em 29/10/07, Marcelo Salhab Brogliato[EMAIL PROTECTED] escreveu:
 Apenas para complementar um pouco sua postagem, segue um exercício
 interessante:
 Calcule a derivada e a integral de x(x-1)(x-2)...(x-n+1).. [dica: use
 números de Stirling]

 Também te proponho o inverso...
 vc fez: x(x-1)(x-2) = x^3 - 3x + 2x ... isto é: encontrarmos os coeficientes
 de x, x^2, x^3...
 te proponho:
 x = x
 x^2 = x(x-1) + x
 x^3 = x(x-1)(x-2) + 3x(x-1) + x

 encontrar os coeficientes de x, x(x-1), x(x-1)(x-2), ...
 estes coeficientes sao chamados de numeros de Stirling de 2a. ordem

 abraços,
 Salhab





 On 10/29/07, Marcelo Salhab Brogliato [EMAIL PROTECTED] wrote:
  Olá Rodrigo,
 
  são os números de Stirling
 (http://en.wikipedia.org/wiki/Stirling_number ).
 
  vamos mostrar algumas coisas legais...
  digamos que:
  x(x-1)(x-2)...(x-n+1) = Sum {k=1..n} S[n, k].x^k
  então:
  x(x-1)(x-2)...(x-n+1)(x-n) = Sum{k=1..n+1} S[n+1, k].x^k
 
  pegando a primeira e multiplicando por (x-n), temos:
  x(x-1)(x-2)...(x-n+1)(x-n) = ( Sum {k=1..n} S[n, k].x^k ).(x-n)
  Sum{k=1..n+1} S[n+1, k].x^k = ( Sum {k=1..n} S[n, k].x^k ).(x-n)
  Sum{k=1..n+1} S[n+1, k].x^k = ( Sum {k=1..n} S[n, k].x^(k+1) ) - ( Sum
 {k=1..n} nS[n, k].x^k )
  Sum{k=1..n+1} S[n+1, k].x^k = ( Sum {k=2..n+1} S[n, k-1].x^k ) - ( Sum
 {k=1..n} nS[n, k].x^k )
  Sum{k=2..n} S[n+1, k].x^k + S[n+1, n+1].x^(n+1) + S[n+1, 1].x = ( Sum
 {k=2..n} (S[n, k-1] - nS[n, k]).x^k ) + S[n, n].x^(n+1) - nS[n, 1].x
 
  portanto:
  S[n+1, 1] = -nS[n, 1]
  S[n+1, k] = S[n, k-1] - nS[n, k] ... k=2, 3, ..., n
  S[n+1, n+1] = S[n, n]
 
  x=x... entao: S[1,1] = 1
 
  x(x-1) = x^2 - x
  S[2, 1] = - 1.S[1, 1] = -1 ... ok!
  S[2, 2] = S[1, 1] = 1 ... ok!
 
  x(x-1)(x-2) = x^3 - 3x + 2x
  S[3, 1] = -2.S[2, 1] = -2.(-1) = 2 ... ok!
  S[3, 2] = S[2, 1] - 2.S[2, 2] = -1 - 2.1 = -3 ... ok!
  S[3, 3] = S[2, 2] = 1 ... ok!
 
  x(x-1)(x-2)(x-3) = x^4 - 6x^3 + 11x^2 - 6x
  S[4, 1] = -3.S[3, 1] = -3.2 = -6  ok!
  S[4, 2] = S[3, 1] - 3.S[3, 2] = 2 - 3.(-3) = 11 ... ok!
  S[4, 3] = S[3, 2] - 3.S[3, 3] = -3 - 3.1 = -6 ... ok!
  S[4, 4] = S[3, 3] = 1 ... ok!
 
  e assim por diante.. :)
 
  abraços,
  Salhab
 
 
 
 
  On 10/27/07, Rodrigo Renji [EMAIL PROTECTED] wrote:
   Tente encontrar uma formula para os coeficientes da potência que
   aparecem na expansão de
  
   x(x-1)(x-2). ... (x-n)
  
   i.e
   x=x
   x(x-1)=x²-x
  
   x(x-1)(x-2)=x³-3x+2x
  
  
   x(x-1)(x-2)(x-3)=x^4 -6x³+11x²-6x
  
   etc...
   (a fórmula existe, é uma recorrência de duas variáveis)
  
  
 =
   Instruções para entrar na lista, sair da lista e usar a lista em
   http://www.mat.puc-rio.br/~obmlistas/obm-l.html

Re: [obm-l] Conjuntos finitos

2007-10-29 Por tôpico Rodrigo Renji
Olá Marcelo \o/

vou tentar explicar o que eu quis dizer sobre somatório em conjuntos
finitos com relação de ordem, mas primeiro vou falar sobre minha
opnião sobre o somatorio comum que aparece na matemática

costuma-se definir
somatorio k=0 até n f(k)= f(0)+...+f(n),

só que eu me sinto meio desconfortavel usando esses pontinhos, acho
meio informal, apesar que dá a idéia do que é o somatório, da uma
noção intuitiva do que acontece, porém eu prefiro não usar esse modo e
sim, definir da seguinte maneira

somatorio k=0 até n f(k)= [somatorio k=0 até n-1 f(k)] +f(n)
se n0, n natural e se n=0

somatorio k=0 até 0 f(k)=f(0)
, i,e repito o processo tirando o ultimo termo do somatorio até chegar
no termo minimo, o mesmo faço para definir somatorio com limite
inferior inteiro e superior inteiro


somatorio k=a até a+p f(k)= [somatorio k=a até a+p-1 f(k)] +f(a+p)
se p0, p natural e se p=0

somatorio k=a até a f(k)=f(a)

com a inteiro. se a inteiro então a+p=b um número inteiro maior ou igual à a
então fica definido para mim o somatorio com limites nos inteiros.

para provar alguns teoremas de somatorio me parece util definir
somatorio k=b até a f(k) =0  se ab (i.e se o limite superior é menor
que o limite inferior)

com essas definições é possivel demonstrar algumas propriedades de
somatorios como

somatorio k=a até b f(k) =somatorio k=a+p até b+p f(k-p)
somatorio k=a até b f(k)=somatorio k=-b até -a f(-k)
somatorio k=a até b f(k)=somatorio k=a até s f(k) +somatorio k=s+1 até b f(k)
se s=a e b=s+1. propriedade que chamo de abertura de somatorios
ela é equivalente a definição que postei acima de somatorio, a partir
de uma se prova outra e vice e versa (porém essa de abertura é mais
geral de certo modo)

mas ai estava pensando em definir somatorios em outros  conjuntos
finitos, por exemplo, definir  formalmente somatorio sobre primos em
um intervalo etc...
na verdade eu me divirto um pouco fazendo essas coisas hehe, as vezes
procurar algo já pronto para mim é o mesmo que contar o final de um
filme, depois eu volto a postar mais sobre, e as idéias que estou
tendo sobre esse assunto

abraços

Em 29/10/07, Marcelo Salhab Brogliato[EMAIL PROTECTED] escreveu:
 Olá Rodrigo,

 pensando sobre o que você disse... pelo que sei a notação de conjuntos é
 geral..
 { a } é um conjunto finito cuja cardinalidade é 1... e a é qualquer
 coisa.. hehe (bem informal)
 sobre retirar os elementos, acredito que você pode fazer assim:
 Seja A um conjunto tal que |A| = n.
 Como A é finito e enumerável, vamos definir uma bijecao f : I_n - A
 onde I_n = { 1, 2, 3, ..., n }.
 façamos: A_0 = A ... A_k = A_(k-1) \ { f(k) }
 deste modo, temos: A_1 = A_0 \ { f(1) } ... A_2 = A_1 \ { f(2) }..
 vamos chegar em A_n = {} ...

 Agora, não entendi o que você quer definir sobre o somatório em conjuntos
 finitos com
 relação de ordem... :))

 um abraço,
 Salhab




 On 10/29/07, Rodrigo Renji [EMAIL PROTECTED] wrote:
 
  Marcelo, obrigado , a fórmula está estranha pois foi eu  formulei
  hehehe, e que eu estava pensando no seguinte, se temos um conjunto de
  números com uma relação de ordem  finito ele possui um máximo e se
  formos tirando o máximo do conjunto em cada etapa, se ele é finito
  chega uma  hora que ele se torna o conjunto vazio
  ex:
  {0, 2.2 ,3}
  tira o máximo (3)
  { 0, 2.2}
  tira o maximo (2.2)
  {0}
  tira o maximo 0
  {}
 
  acho que o mesmo acontece quando vamos tirando o minimo do conjunto
  não é? se ele é um conjunto de numeros com relação de ordem e finito,
  chega uma hora que ficamos com o conjunto vazio (só pra explicar o que
  eu estava/estou pensando sobre essa questão).  A principio estava
  pensando que valia pra qualquer conjunto finito, mas é obvio que não
  ( conjunto {a} a sendo a letra do alfabeto , é um conjunto finito não
  é?, tendo apenas um elemento, ou essa nomenclatura é apenas para
  conjuntos númericos ?)
 
 
  esses conceitos eu estou usando para tentar definir formalmente
  somatorio sobre um conjunto finito de numeros com relação de ordem.
 
  abraços
  Em 27/10/07, Marcelo Salhab Brogliato[EMAIL PROTECTED] escreveu:
   Olá Rodrigo,
  
   achei a formulação um tanto estranha... nao acho correto dizer: Existe
 n
   tal que S(n) = vazio... pois n está definido na questão..
   acredito que deveria ser: Se S(n) = vazio, entao |S(n)| = n ?
  
   |S(0)| = |S|
   |S(1)| = |S(0) - {max S(n)}| .. como {max S(n)} E S(0), e |{max S(n)}| =
 1,
   entao: |S(1)| = |S| - 1
   por inducao: |S(k)| = |S| - k
  
   vamos supor que |S|  n, entao |S(n)|  0, absurdo! Pois |S(n)| = 0 por
   hipótese..
   vamos supor que |S|  n, entao |S(|S|)| = 0... assim: |S(n)| = 0
   vamos supor que |S| = n, entao |S(n)| = 0... assim: |S(n)| = 0
  
   logo, podemos concluir que S é finito, e que a cardinalidade S é menor
 ou
   igual a n...
  
   abraços,
   Salhab
  
  
  
   On 10/27/07, Rodrigo Renji [EMAIL PROTECTED] wrote:
   
Seja
S um conjunto
defino
(n natural)
   
S(n+1)=S(n)-{max S(n)}
S(0)=S
   
(se S(n

[obm-l] Expansão de termos -proposta de problema

2007-10-27 Por tôpico Rodrigo Renji
Tente encontrar uma formula para os coeficientes da potência que
aparecem na expansão de

x(x-1)(x-2). ... (x-n)

i.e
x=x
x(x-1)=x²-x

x(x-1)(x-2)=x³-3x+2x


x(x-1)(x-2)(x-3)=x^4 -6x³+11x²-6x

etc...
(a fórmula existe, é uma recorrência de duas variáveis)

=
Instruções para entrar na lista, sair da lista e usar a lista em
http://www.mat.puc-rio.br/~obmlistas/obm-l.html
=


[obm-l] Conjuntos finitos

2007-10-27 Por tôpico Rodrigo Renji
Seja
S um conjunto
defino
(n natural)

S(n+1)=S(n)-{max S(n)}
S(0)=S

(se S(n) possui máximo) [prestar atenção nessa condição]

Se existe n, tal que s(n)=vazio
então n é finito e tem n elementos?

e se um conjunto é finito vale a propriedade acima?
(relaçao de se e somente se).

=
Instruções para entrar na lista, sair da lista e usar a lista em
http://www.mat.puc-rio.br/~obmlistas/obm-l.html
=


Re: [obm-l] Qual Triangulo?

2007-10-24 Por tôpico Rodrigo Renji
troquei as letras apenas
p=a ;q=b ;c=r. eu já tinha resolvido em outro lugar  copiei e colei aqui

a² + b² + c² = ab + bc +ac


chamo
a=a
b=a+k
c=a+p
sem perda de generalidade


substituindo ficamos com

a²+(a+k)²+(a+p)²=a(a+k)+(a+k)(a+p)+a(a+p)
expandindo temos

a²+a²+2ak+k²+a²+2ap+p²=a²+ak+a²+ap+ak+a²+ap+kp

anulando os a² em ambos os lados
2ak+k²+2ap+p²=ak+ap+ak+ap+kp

anulando 2ap e 2ak de ambos os lados
k²+p²=kp

k²-pk+p²=0

tomando como equação em k

delta =p²-4p²=-3p²

k=[p+- raiz(-3p²)]/2

como delta =-3p²
e p² é sempre maior ou igual a zero
só teremos valor real na equação se p=0, porém se p=0
k=0
logo
a=b=c


equilatero, letra a


Em 24/10/07, Thelio Gama[EMAIL PROTECTED] escreveu:
 Essa aqui ta difícil, nenhum dos feras da minha turma resolveu. Gostaria da
 ajuda dos senhores. Obrigado.



 Se p, q e r sao os comprimentos dos lados de um triangulo e se p² + q² + r²
 = pq + qr + pr, entao o triangulo é:

 a)   Equilatero

 b)   Escaleno

 c)   Reto

 d)   Obtuso

 e)N.R.A.



 Thelio

=
Instruções para entrar na lista, sair da lista e usar a lista em
http://www.mat.puc-rio.br/~obmlistas/obm-l.html
=


[obm-l] pequeno teorema sobre numeros de stirling parte 1

2007-10-24 Por tôpico Rodrigo Renji
Descobri esse pequeno teorema e quero compartilhar com o pessoal da
lista, lá vai ele

vou provar aqui um pequeno teorema que descobri esse dias que fala de
numeros tem como corolario a correlação entre números de stirling e
potencias fatoriais


o somatório de f(k), com k variando de k=0 até k=n, eu vou escrever como
soma[k=0,n]f(k).


seja um operador T qualquer com as seguintes propriedades
Tsoma[k=0,n](ck)f(k,x)=soma[k=0,n](ck)Tf(k,x)
isto é, comuta com somatório (ck são números)
e

T[T^(n)g(x)=T^(n+1)g(x).
com
T^(0)g(x)=g(x), para qualquer n natural.



e, seja uma função de duas variáveis g(k,x), definida pelo menos para
k inteiro não negativo. tal que

g(x)=g(0,x)
e

Tg(k,x)=kg(k,x)+g(k+1,x)

agora definição para números de stirling
são numeros que satisfazem a recorrencia

[n,k]=k[n-1,k] + [n-1,k-1]

se nk e k1
[n,n]=1 para qualquer n natural
[n,1]=1 para qualquer n natural
e [n+1,0]=0 para qualquer n natural

temos então o teorema

T^(n)[g(x)]=soma[k=0,n][n,k]g(k,x).

demonstração por indução sobre n

para n=0 temos

T^(0)[g(x)]=g(x)=soma[k=0,0].[0,k].g(k,x)=[0,0]g(0,x)=g(x)
pelas definições do operador e da função g(k,x) e números de stirling

hipotese da indução
T^(n)[g(x)]=soma[k=0,n][n,k]g(k,x).


vamos demonstrar agora para
(n+1)
T^(n+1)[g(x)]=soma[k=0,n+1][n+1,k]g(k,x).
partindo da hipotese e aplicando o operador T nela

T[T^(n)][g(x)]=T^(n+1)[g(x)]=Tsoma[k=0,n][n,k]g(k,x) =soma[k=0,n][n,k]Tg(k,x)

pela definição do operador e suas propriedades, agora aplicando a
definição do operador na função g(k,x) temos
=soma[k=0,n][n,k][kg(k,x)+g(k+1,x)]=
usando a linearidade do somatorio ficamos com
=soma[k=0,n][n,k]kg(k,x)+soma[k=0,n][n,k]g(k+1,x)=

continua

=
Instruções para entrar na lista, sair da lista e usar a lista em
http://www.mat.puc-rio.br/~obmlistas/obm-l.html
=


[obm-l] Re: pequeno teorema sobre numeros de stirling parte 1

2007-10-24 Por tôpico Rodrigo Renji
=soma[k=0,n][n,k]kg(k,x)+soma[k=0,n][n,k]g(k+1,x)=
aparece um termo k dentro do somatorio, se abrirmos o primeiro termo
do somatorio verificamos que ele é zero, então podemos escrever

=soma[k=1,n][n,k]kg(k,x)+soma[k=0,n][n,k]g(k+1,x)=

fazendo uma mudança de variavel no somatorio, subtraindo 1 dos limites
ficamos com
(apenas no primeiro somatorio)

=soma[k=0,n-1][n,k+1](k+1)g(k+1,x)+soma[k=0,n][n,k]g(k+1,x)=
abrindo o ultimo termo do segundo ficamos com

=soma[k=0,n-1][n,k+1](k+1)g(k+1,x)+soma[k=0,n-1][n,k]g(k+1,x)+[n,n]g(n+1,x)=
juntando agora os dois somatorios do meio em um mesmo por linearidade

=soma[k=0,n-1][n,k+1](k+1)g(k+1,x)+[n,k]g(k+1,x)+[n,n]g(n+1,x)=
colocando g(k+1,x) em evidencia

=soma[k=0,n-1][[n,k+1](k+1)+[n,k]]g(k+1,x)+[n,n]g(n+1,x)=
usando a definição de números de stirling para escrever

=soma[k=0,n-1][n+1,k+1]g(k+1,x)+[n,n]g(n+1,x)=

[n,n]=1=[n+1,n+1] pela definição de numeros de stirling, tomando uma
mudança de variavel no somatorio, somando +1 aos limites

=soma[k=1,n][n+1,k]g(k,x)+[n+1,n+1]g(n+1,x)=

juntando o termo [n+1,n+1]g(n+1,x)= ao somatorio

=soma[k=1,n+1][n+1,k]g(k,x)
lembrando que [n+1,0]=0 podemos escrever finalmente

=T^(n+1)g(x)=soma[k=0,n+1][n+1,k]g(k,x)
c.e.d



dois corolários
primeiro
toda potencia pode ser escrita como soma de potencias fatoriais

faça x=T
então
xg(k,x)=kg(k,x)+g(k+1,x) e g(0,x)=1
g(k,x) será a potencia fatorial x^n =T^n,

x^n=soma[k=0,n][n,k]g(k,x)


segundo
tome
g(x)=f(e^x)
e g(k,x)=e^(kx).f^(k)(e^x)
com o operador T como derivada, que vou simbolizar ela aplicada por
[g(x)]^(n), temos então

[f(e^x)]^(n)=soma[k=0,n][n,k]e^(kx).f^(k)(e^x)

f^(k)(e^x) é a k-esima derivada da função [nao sendo da composta
inteira, apenas da exterior]

só isso =x

Em 24/10/07, Rodrigo Renji[EMAIL PROTECTED] escreveu:
 Descobri esse pequeno teorema e quero compartilhar com o pessoal da
 lista, lá vai ele

 vou provar aqui um pequeno teorema que descobri esse dias que fala de
 numeros tem como corolario a correlação entre números de stirling e
 potencias fatoriais


 o somatório de f(k), com k variando de k=0 até k=n, eu vou escrever como
 soma[k=0,n]f(k).


 seja um operador T qualquer com as seguintes propriedades
 Tsoma[k=0,n](ck)f(k,x)=soma[k=0,n](ck)Tf(k,x)
 isto é, comuta com somatório (ck são números)
 e

 T[T^(n)g(x)=T^(n+1)g(x).
 com
 T^(0)g(x)=g(x), para qualquer n natural.



 e, seja uma função de duas variáveis g(k,x), definida pelo menos para
 k inteiro não negativo. tal que

 g(x)=g(0,x)
 e

 Tg(k,x)=kg(k,x)+g(k+1,x)

 agora definição para números de stirling
 são numeros que satisfazem a recorrencia

 [n,k]=k[n-1,k] + [n-1,k-1]

 se nk e k1
 [n,n]=1 para qualquer n natural
 [n,1]=1 para qualquer n natural
 e [n+1,0]=0 para qualquer n natural

 temos então o teorema

 T^(n)[g(x)]=soma[k=0,n][n,k]g(k,x).

 demonstração por indução sobre n

 para n=0 temos

 T^(0)[g(x)]=g(x)=soma[k=0,0].[0,k].g(k,x)=[0,0]g(0,x)=g(x)
 pelas definições do operador e da função g(k,x) e números de stirling

 hipotese da indução
 T^(n)[g(x)]=soma[k=0,n][n,k]g(k,x).


 vamos demonstrar agora para
 (n+1)
 T^(n+1)[g(x)]=soma[k=0,n+1][n+1,k]g(k,x).
 partindo da hipotese e aplicando o operador T nela

 T[T^(n)][g(x)]=T^(n+1)[g(x)]=Tsoma[k=0,n][n,k]g(k,x) =soma[k=0,n][n,k]Tg(k,x)

 pela definição do operador e suas propriedades, agora aplicando a
 definição do operador na função g(k,x) temos
 =soma[k=0,n][n,k][kg(k,x)+g(k+1,x)]=
 usando a linearidade do somatorio ficamos com
 =soma[k=0,n][n,k]kg(k,x)+soma[k=0,n][n,k]g(k+1,x)=

 continua


=
Instruções para entrar na lista, sair da lista e usar a lista em
http://www.mat.puc-rio.br/~obmlistas/obm-l.html
=